Sie sind auf Seite 1von 95

1 | P a g e

DOCTRINE OF SEPARATE LEGAL PERSONALITY


Palay, Inc. v. Clave
Facts:
Onstott as the President of Palay Inc. executed in favour of Dumpit a
Contract to Sell of a parcel of land in Antipolo. Paragraph six of the contract
provided for automatic extrajudicial rescission upon default without need of
notice and with forfeiture of all instalments paid.
Dumpit made the last instalment sometime in September 1967, six years
after, he wrote petitioner offering to update all his overdue accounts and
seeking consent to transfer his right to Dizon. Palay replied that the contract
had long been rescinded and that the lot had been resold.
Dumpit questioned the validity of the rescission with the NHA which ruled in
his favor, NHA ordered Palay Inc. and Onstott as president, jointly and
severally, to refund to Dumpit what the latter had paid with interest.
Onstott controverted that he should not have been made jointly and
severally liable with Palay since a corporation has a personality distinct and
separate from the persons composing it.

Issue:
Whether the doctrine of piercing of veil of corporate fiction has application
to the case at bar

Held:
NO. We find no badges of fraud on petitioners part. They had literally
relied, albeit mistakenly, on paragraph 6 of the contract when it rescinded
the same and sold it to a third person.
In the case, petitioner Onstott was made liable because he was then the
controlling stockholder of the corporation. However no sufficient proof
exists on the record that said petitioner used the corporation to defraud
private respondent. He cannot, therefore, be made personally liable just
because he appears to be the controlling stockholder. Mere ownership of a
single stockholder is not in itself a sufficient ground for disregarding the
separate corporate personality.

Note:
General rule: a corporation may not be made to answer for acts or liabilities
of its stockholders or those of the legal entities to which it may be
connected and vice versa.

Exception: The veil of corporate fiction may be pierced when it is used:
1. as a shield to further an end subversive of justice; or
2. for purposes that could not have been intended by the law that
created it; or
3. to defeat public convenience, justify wrong, protect fraud or defend
crime; or
4. to perpetuate fraud or confuse legitimate issues; or
5. to circumvent the law or perpetuate deception; or
6. as an alter ego, adjunct or business conduit for the sole benefit of
the stockholders.

Cruz v. Dalisay
Facts:
Sheriff Dalisay was charged with malfeasance in office, corrupt practices and
serious irregularities. It was alleged that the sheriff attached and/ or levied
the money belonging to complainant Cruz when he was not the judgment
debtor in the final judgment of the NLRC sought to be enforced, but rather
the company known as Qualitrans Limousine Service, Inc. a duly
registered company.
In his comment Sheriff Dalisay argued that he attached the money on the
basis of an affidavit executed by Cruz stating that he is the owner/ president
of said corporation.
Prior to the termination of the proceedings, complainant executed an
affidavit of desistance stating that he is no longer interested in prosecuting
the case.

Issue:
Whether or not the Sheriff erred in enforcing a judgment against Cruz who
is not the judgment debtor in the case.

Held:
YES. The tenor of the NLRC judgment and the implementing writ are clear
enough; it is directed against Qualitrans Limousine Inc. ordering it to
reinstate the discharged employees and pay them full backwages.
2 | P a g e

Respondent however chose to pierce the veil of corporate entity usurping
a power belonging to the court and assumed improvidently that since the
complainant is the owner/ president of the corporation, they are one and
the same.
It is well- settled doctrine both in law and in equity that as a legal entity, a
corporation has a personality distinct and separate from its individual
stockholders and members. The mere fact that one is a president of a
corporation does not render the property he owns or possesses the
property of the corporation; since the president as individual and the
corporation, are separate entities.

Indophil Textile Mill Workers Union v. Calica
Facts:
Petitioner Indophil and private respondent Indophil Textile Mills, Inc.
executed a collective bargaining agreement
On November 3, 1987, Indophil Acrylic Manufacturing Corporation was
formed and registered with the SEC.
Subsequently, Acrylic applied for registration with the Board of Investments
for incentives. The application was approved on a preferred non-pioneer
status.
In 1988, Acrylic became operational and hired workers according to its own
criteria and standards.
Sometime in July, 1989, the workers of Acrylic unionized and a duly certified
CBA was executed.
In 1990 or a year after the workers of Acrylic have been unionized and a
CBA executed, the petitioner union claimed that Acrylic is part of the
Indophil bargaining unit.
The petitioner's contention was opposed by private respondent which
submits that it is a juridical entity separate and distinct from Acrylic.
The existing impasse led the petitioner and private respondent to enter into
a submission agreement.
After the parties submitted their respective position papers and replies, the
Voluntary Arbitrator rendered its award that Acrylic is not an extension or
expansion of Indophil Textile Mills, Inc. interpreting the CBA provision
relative to the issue.
Hence this petition.

Issue:
Whether or not the respondent arbitrator erred in interpreting that Acrylic
is not an extension of Indophil

Held:
NO. Acrylic is not an extension or expansion of private respondent, the rank-
and-file employees working at Acrylic should not be recognized as part of,
and/or within the scope of the petitioner, as the bargaining representative
of private respondent.
The fact that the businesses of private respondent and Acrylic are related,
that some of the employees of the private respondent are the same persons
manning and providing for auxiliary services to the units of Acrylic, and that
the physical plants, offices and facilities are situated in the same compound,
are not sufficient to justify the piercing of the corporate veil of Acrylic.

EPG Construction Company, Inc. v. Court of Appeals
Facts:
EPG and UP entered into a contract for the construction of the UP Law
Library. In the contract, the parties stipulated that the Contractors
guarantees the completion of the work, and that the contractor shall repair
at his own cost for one year from the completion all the work covered by
the contract.
Upon completion on January 1983, the building was formally turned over to
UP. Sometime in July 1983, UP complaint to the petitioner that 6 air
conditioning units on the 3
rd
floor were not cooling properly. After
inspection, EPG agreed to shoulder the expenses for its repair.
The repair however was never undertaken; UP contracted another company
for the repair of the air conditioning units and sought reimbursement from
EPG which the latter refused. UP then sued EPG and its president De
Guzman.
The court ruled for UP which on appeal was sustained.

Issue:
Whether or not De Guzman is liable

Held:
NO. It is a doctrine well- established and obtains both in law and in equity
that a corporation is a distinct legal entity separate and apart from the
individual stockholders or members who compose it, and is not affected by
3 | P a g e

the personal rights, obligations and transactions of its stockholders or
members.
Mere ownership by a single stockholder or by another corporation of all or
nearly all of the capital stock of a corporation is not of itself a sufficient
ground for disregarding the separate corporate personality of the
corporation.
The general manager of a corporation therefore should not be made
personally answerable for the payment of the employees back wages unless
he had acted maliciously or in bad faith, in which event he may be made
personally liable for his own act. That exception is not applicable to the case
at bar because it had not been proved that De Guzman acted maliciously or
in bad faith when as President of EPG, he sought to protect its interests and
resisted UPs claims. Whatever damage was caused to UP as a result of his
acts is the sole responsibility of EPG even though De Guzman was its
principal officer and controlling stockholder.

Boyer- Roxas v. Court of Appeals
Facts:
The subject properties belonged to Eugenia Roxas. After her death her heirs,
among them the petitioners formed a corporation, the Eugenia Roxas Inc.
with the inherited properties as the capital of the corporation. Its primary
purpose was to engage in agriculture, its AOI was subsequently amended to
allow it to engage in the resort business.
Accordingly, the corporation used the subject properties to put up a resort
known as the Hidden Valley Springs Resort.
Eugenia Roxas Inc. filed an action for ejectment from the resort against
Rebecca and Guillermo Roxas, alleging that the property is owned by the
corporation.
Petitioners on the other hand contend that they are the heirs of Eugenia
Roxas and as such they are co- owners of the subject properties and that
their stay in the resort was consented by Eufrocino who then controlled the
management of the corporation.
Petitioners contended that the veil of corporate fiction should be pierced
considering that petitioners are owners of the aliquot part of the properties
of private respondent.

Issue:
Whether the veil of corporate fiction should be pierced

Held:
NO. Eugenia Roxas Inc. is a bona fide corporation. As such, it has a juridical
personality of its own separate from the stockholders or members
composing it.
Properties registered in the name of the corporation are owned by it as an
entity separate and distinct from its members. While share of stocks
constitute personal property, they do not represent property of the
corporation. The corporation has property of its own which consists chiefly
of real estate. A share of stock only typifies an aliquot part of the
corporations property, or the right to share in its proceeds to that extent
when distributed according to law and equity but its holder is not the owner
of the any part of the capital of the corporation, nor is he entitled to the
possession of any definite portion of its property or assets.
The corporation transacts it business only through its officers or agents
whatever authority these officers of agents may have is derived from the
board of directors or other governing body unless conferred by the charter
of the corporation.
An officers power as an agent of the corporation must be sought from the
statute, charter, the by- laws or in a delegation of authority to such officer
from the acts of the board of directors, formally expressed or implied from a
habit or custom of doing business.
The petitioners stay within the questioned properties was merely by
tolerance of the respondent corporation in deference to the wishes of
Eufricino Roxas, who during his lifetime controlled and managed the
corporation. Eufrocino Roxas actions could not have bound the corporation
forever.

Note:
The separate personality of the corporation may be disregarded only when
the corporation is used as a cloak or cover for fraud or illegality, or to work
injustice, or where necessary for the protection of the creditors.

Asionics Philippines, Inc. v. NLRC
Facts:
Boaquina and Gayola were employers of API as control clerk and production
operator. During the third quarter of 1992 API negotiated with FFW
(Federation of Free Workers), the bargaining agent of its employees.
4 | P a g e

The negotiations resulted into a deadlock which prompted FFW to file a
notice of strike. As a result API was forced to suspend operations. Boaquina
and Gayola were among those asked to take a leave from worked.
A CBA was then concluded. Despite such, the business activity of API
remained critical, it was forced to implement retrenchment which affected
a number of employees including Boaquina and Gayola.
The two, together with the others joined Lakas Union. The Union staged a
strike which was declared illegal. Further the NLRC ruled that the
respondents were validly terminated in accordance with the retrenchment
policy of API and as such ordered the corporation to pay together with
Frank Yih who was its president and at the same time the majority
stockholder of the company.

Issue:
Whether or not a stockholder/ director/ president of a corporation can be
held liable for the obligation of the corporation absent any proof and finding
of bad faith.

Held:
NO. It is basic that a corporation is invested by law with a personality
distinct and separate from those of the persons composing it as well as from
that of any other legal entity to which it may be related.
Mere ownership by a single stockholder or by another corporation of all or
nearly all of the capital stock of the corporation is not of itself sufficient
ground for disregarding the separate corporate personality.
Nothing on record is shown to indicate that Frank Yih has acted in bad faith
or with malice in carrying out the retrenchment program of the company.
His having been held by the NLRC to be solidarilly and personally liable with
API is thus legally unjustified.


Francisco Motors Corporation v. Court of Appeals
Facts:
FMC filed a complaint against Spouses Manuel to recover the balance of the
jeep body purchased by the latter from the former. The cost of the repair
was added to the Spouses outstanding obligation.
In their answer, the Spouses interposed a counterclaim for unpaid legal
services by Gregorio Manuel in the amount of P50,000, which was not paid
by the incorporators, directors and officers of FMC. The lower court ruled
for the petitioner as regards the claim of money but also allowed the
counterclaim. The CA sustained the ruling of the lower court.
Manuel argues that he previously represented the members of Francisco
family in the intestate proceedings of the late Trinidad and that his services
were not paid; that said family members were also the incorporators,
directors and officers of FMC.

Issue:
Whether or not the corporation should be held liable for the fees owed by
the incorporators, directors and officers of the corporation in their personal
capacities

Held:
NO. Given the facts and circumstance of the case, the doctrine of piercing
the corporate veil has no relevant application. Respondent court erred in
permitting the lower courts resort to this doctrine.
The rationale behind piercing a corporations identity in a given case is to
remove the barrier between the corporation from the persons composing it
to thwart the fraudulent and illegal schemes of those who use the corporate
personality as a shield for undertaking certain proscribed activities.
However in the case at bar, instead of holding certain individuals or persons
liable for an alleged corporate act, the situation has been reversed, it is the
petitioner as a corporation which is being ordered to answer for the
personal liability of certain individual directors, officers, and incorporators
concerned. Hence, it appears to the court that the doctrine has been turned
upside down because of its erroneous invocation.
Note that according to Manuel his services were solicited as counsel for the
members of Francisco family in the estate proceedings. These proceedings
did not involve any business of the petitioner corporation.
When directors and officers of a corporation are unable to compensate a
party for personal obligation, it is far fetched to alleged that the corporation
is perpetuating fraud or promoting injustice, and thereby held liable
therefor by piercing its corporate veil.
The claim for legal fees against the concerned individuals could not properly
be directed against the corporation without violating basic principles
governing corporations.

5 | P a g e

Complex Electronics Employees Association v. NLRC
Facts:
Complex Electronics was engaged in the manufacture of electronic
products, its customers were foreign based companies with different
product lines.
Lite- On Taiwan, one of its clients, required Complex to reduce its selling
price, however Complex answered that it was not feasible as it was already
incurring losses. After sometime, Complex was forced to close down the
Lite- On line.
The displaced workers of the line, through the CEEA pushed for
retrenchment pay, Complex refused.
The machineries and equipment were pulled out from the company
premises and were transferred to Ionics Circuit Inc.
CEEA filed a complaint with the NLRC, it impleaded Ionics as party
defendant because the officers and management personnel of Complex and
President Qua were its officers as well.
Ionics contended that it was an entity separate and distinct from Complex.
The LA rendered judgment against Complex, Ionics and President Qua.
When the case reached the SC, CEEA presented documentary evidence
which consisted of a newspaper clipping allegedly proving that both
companies, Ionics and Complex are one and the same.
CEEA claimed that business has not ceased at Complex but that it was
merely transferred to Ionics, a runaway shop, thus justifying the reason for
piercing the veil of corporate fiction.

Issues:
1. Whether or not Ionics may be held liable together with Complex on
the ground that both companies have the set of board of Directors
2. Whether or not Lawrence Qua should be made personally liable to
CEEA since he was the principal player in the closure of the
company.

Held:
NO TO BOTH ISSUES
1. A runaway shop is defined as an industrial plain moved by its
owners from one location to another to escape union labor
regulations or state laws, the term is also used to describe a plant
removed to a new location in order to discriminate against
employees at the old plant because of their union activities.
At the time the labor dispute arose at Complex, Ionics was already
existing as an independent company. It cannot therefore be said
that the temporary closure in Complex and its subsequent transfer
of business to Ionics was for anti- union purposes.
Ionics may be engaged in the same business as that of Complex, but
this fact alone is not enough reason to pierce the veil of corporate
fiction. To disregard the separate juridical personality of the
corporation, the wrongdoing must be clearly and convincingly
established.
2. It is settled that in the absence of malice or bad faith, a stockholder
or an officer of the corporation cannot be made personally liable for
corporate liabilities.
In the present case, while it may be true that the equipment,
materials and machineries were transferred to Ionics during the
night, but it was because the laborers were already vandalizing the
equipments. Such pull out was made pursuant to the demand of
Complex customers who were already alarmed by the labor dispute.
Lim v. Court of Appeals
Facts:
Petitioner Rufina Lim is the surviving spouse of the late Pastor Lim whose
estate is the subject of a probate proceedings.
Private respondent corporations: Auto Truck Corporation, Alliance
Marketing Corp., Speed Distributing Inc., Active Distributing Inc., and Action
Company, whose properties were included in the inventory of the estate,
filed a motion for exclusion from the estate which was granted by the RTC.
Lim filed a verified amended petition claiming that Pastor Lim personally
owned during his lifetime the following business entities; that all their
capital, assets and equity were personally owned by the late Lim. Hence, the
alleged stockholder and officers appearing in the respective AOI of the
above business entities were mere dummies and that they were listed only
for purposes of registration with the SEC.
The RTC ruled in favor of Lim, CA reversed.

Issue:
1. May a corporation, in its universality, be the proper subject of and
be included in the inventory of the estate of a deceased person?
6 | P a g e

2. Whether or not the corporations are the mere alter egos or
instrumentalities of Pastor Lim

Held:
NO TO BOTH ISSUES
1. The real properties included in the inventory of the estate of the
late Pastor Lim are in the possession of and are registered in the
name of private respondent corporations, which under the law
possess a personality separate and distinct from their stockholders,
and in the absence of any cogency to shred the veil of corporate
fiction, the presumption of conclusiveness of said titles in favor of
the corporations should stand undisturbed.
2. Mere ownership by a single stockholder or by another corporation
of all or nearly all of the capital stock of a corporation is not of itself
a sufficient reason for disregarding the fiction of separate corporate
personalities.

Note:
The tests in determining the applicability of the doctrine of piercing the veil
of corporate fiction are as follows:
1. Control, not mere majority or complete stock control, but complete
domination not only of the finances but of policy and business
practice in respect to the transaction attacked so that the corporate
entity as to this transaction had at the time no separate mind, will
or existence of its own;
2. Such control must have been used by the defendant to commit
fraud or wrong, to perpetuate the violation of statutory or other
positive legal duty, or dishonest and unjust act in contravention of
plaintiffs legal right; and
3. The aforesaid control and breach of duty must proximately cause
the injury or unjust loss complained of. The absence of any of these
elements prevents piercing the corporate veil.

Land Bank of the Philippines v. Court of Appeals
Facts:
On various dates, Land Bank extended a series of credit accommodations to
ECO using the trust funds of PVTA. The proceeds of the credit
accommodations were received on behalf of ECO by appellate Ooate.
ECO defaulted to pay its obligation; it proposed a Plan of Payment whereby
the former would set up a financing company which would absorb the loan
obligations.
Land bank approved the same but averred that it will not participate in any
manner whatsoever. ECO revised the plan, but the bank rejected the same.
Land Bank sent a letter to PVTA to hear its comment; the letter stated that
the failure of PVTA to answer would be tantamount to approval of the
banks intention to file a suit against ECO. No response was made.
A suit was filed and judgment was rendered against ECO, Ooate was
absolved from any liability.

Issue:
1. Whether or not the corporate veil of ECO Management Corporation
should be pierced
2. Whether or not Emmanuel Ooate should be held jointly and
severally liable with ECO Management Corporation for the loans
incurred from Land Bank

Held:
NO TO BOTH ISSUES
1. The burden is on the petitioner to prove that the corporation and its
stockholders are, in fact, using the personality of the corporation as
a means to perpetuate fraud and/ or escape a liability and
responsibility demanded by law. In order to disregard the separate
juridical personality of a corporation, the wrongdoing must be
clearly and convincingly established.
In the absence of any malice or bad faith, a stockholder or officer of
a corporation cannot be made personally liable for any corporate
liabilities
2. The mere fact that Ooate owned the majority of the shares of ECO
is not a ground to conclude that Ooate and ECO are one and the
same. Mere ownership by a single stockholder of all or nearly all of
the capital stock of a corporation is not by itself a sufficient reason
for disregarding the fiction of separate corporate personality.

PNB v. Ritratto Group Inc.
Facts:
7 | P a g e

PNB- IFL, a subsidiary company of PNB extended a letter of credit in favor or
Ritratto Group secured by real estate mortgages.
Ritratto Group failed to pay the entire obligation; as a result, PNB-IFL
through its attorney- in fact PNB, notified the former of the foreclosure of
all the real estate mortgages and that subject properties were to be sold at
a public auction.
Rittrato filed for injunction against PNB; further, it prayed that PNB be
ordered to re- compute the rescheduling of the interest. PNB moved for
dismissal on the ground of failure to state a cause of action and absence of
any privity between the petitioner and respondent. The injunction was
granted; the RTC ruled that a suit against PNB is a suit against PNB- IFL. CA
affirmed.

Issue:
Whether a corporate entity may be disregarded where a corporation is a
mere alter ego or business conduit of a person or where the corporation is
so organized and controlled and its affairs are so conducted as to make it
merely an instrumentality, agency, conduit or adjunct of another
corporation

Held:
NO. The general rule is that as a legal entity, a corporation has a personality
separate and distinct from its individual stockholders or members, and is
not affected by the personal rights, obligations and transactions of the
latter. The mere fact that a corporation owns all of the stocks of another
corporation, taken alone is not sufficient to justify their being treated as one
entity.
If used to perform a legitimate function, a subsidiarys separate existence
must be respected, and the liability of the parent corporation as well as the
subsidiary will be confined to those arising in their respective business.
When a borrower failed to pay credit accommodations granted by a
subsidiary of a banking corporation, the suit against the parent company to
direct it re- compute the rescheduling of the interest to be paid and enjoin
the foreclosure initiated by the parent company as attorney- in- fact of the
subsidiary will not prosper because the corporations are separate and
distinct from each other. Aside from the fact that the lender is a wholly-
owned subsidiary, there is no showing that it is a mere instrumentality of
the parent company. The parent- subsidiary relationship between the two
corporations is not the significant relationship involved in this case since the
parent company was not sued because it is the parent company of the
lender. Rather, it was sued because it acted as attorney- in- fact of the
lender in initiating the foreclosure proceedings. A suit against the agent
cannot without compelling reasons, be considered a suit against the
principal.

China Banking Corporation v. Dyne Sem Electronics
Facts:
Dynetics and Lim borrowed P9M from China Bank which was evidenced by 6
PNs.
The borrowers failed to pay when the obligations became due, this
prompted China Bank to file a complaint for sum of money. Summons were
not served on Dynetics because it closed down. On the other hand, Lim
denied that he promised to pay the obligation jointly and severally with
Dynetics.
An amended complaint was filed impleading Dyne Sem and its stockholder
on the ground that it was allegedly formed to be Dyneticss alter ego, the
argument was bolstered by the acquisition of some of the machineries of
Dynetics by Dyne Sem.
The court held that Dyne Sem is not an alter ego of Dynetics thus it is not
liable under the PNs.
The CA sustained the RTC.

Issue:
Whether or not the doctrine of piercing the veil of corporate fiction is
applicable in the case

Held:
NO. a corporation could not be made party defendant to a collection simply
because summons could not be served on the debtor- corporation on the
mere grounds that the businesses of the two corporation are interrelated
and they have common directors absent sufficient showing that the
corporate entity was purposely used as a shield to defraud creditors and
third persons of their rights.
Likewise, the acquisition of some of the machineries and equipment of the
seller corporation was not proof that the buyer- corporation was formed to
defraud creditor of the seller corporation. No merger took place between
8 | P a g e

the seller and the buyer. What took place was a sale of the assets of the
former to the latter. Merger is legally distinct from a sale of assets. Thus,
were one corporation sells or otherwise transfers all of its assets to another
corporation for value, the latter is not by that fact alone, liable for the debts
and liabilities of the transferor.

Spouses Nisce v. Equitable PCI Bank
Facts:
Natividad Nisce requested that $20,000 of her deposit with PCI Bank be
transferred to PCI Capital in Hong Kong. PCI Capital is the subsidiary of PCI
Bank.
PCI Capital issued a Certificate of Deposit as proof of receipt of the transfer.
Sometime in June 1991, Natividads two sons were stranded in Hong Kong.
She called the bank and requested for the partial release of her dollar
deposit to her sons; however, she was informed that according to computer
records, no such dollar account existed.
In the meantime, in 1994, the Equitable Banking Corporation and the PCI
Bank were merged under the corporate name Equitable PCI Bank.
In 1996, the Spouses Nisce obtained a 20M and 13M loan secured by real
estate mortgages with Equitable PCI Bank. Petitioners failed to comply with
their obligations, which prompted the bank to extra judicially foreclose the
mortgages.
Natividad offer to settle their loan account with Equitable by offsetting the
peso equivalent of her dollar account with PCI Capital. In 2001, PCI Bank,
now Equitable Bank, claimed that it merely acted as a conduit in facilitating
the transfer of the funds.
RTC issued the injunction. CA on the other hand ruled in favor of Equitable,
that the bank had a clear right to seek the remedy of foreclosure; that legal
compensation cannot take place because the parties are not mutual
creditors and debtors of each other.

Issue:
Whether or not the claim of an investor against a subsidiary of another
corporation which subsequently became the acquired corporation in a
merger be enforced against the surviving corporation.

Held:
NO. PCI Capital is a subsidiary of PCI Bank now, Equitable PCI Bank. PCI
Capital has an independent and separate juridical personality from that of
Equitable, its parent company; hence, any claim against the subsidiary is not
a claim against the parent company and vice versa.
In the case at bar, PCI Bank, which had been merged with Equitable owned
all of the stocks of PCI Capital, however that fact taken alone is not
sufficient to justify their being treated as one entity. If used to perform
legitimate functions, a subsidiarys separate existence shall be respected
and the liability of the parent corporation, as well as the subsidiary shall be
confined to those arising in their respective business.
Petitioners failed to adduce evidence to justify the piercing of the veil of
corporate entity and render respondent Bank liable for the $20,000 deposit
of petitioner Natividad.

Yamamoto v. Nishino Leather Industries Inc.
Facts:
WAKO was organized by Yamamoto under Philippine Laws. Yamamoto and
Nishino entered into a MOA under which they agreed to enter a joint
venture wherein Nishino would acquire 70% of the authorized capital stock
of WAKO.
Eventually, Nishino and his brother Yoshinobu acquired more than 70% of
the authorized capital stock of WAKO, reducing Yamamotos investment
therein to, according to his claim, 10%.
The Corporate name of WAKO was later changed to NLII.
In a letter made by the corporate lawyer, Yamamoto was allowed to take
out the equipment which were part of his investment in the corporation but
he was stopped by the respondents prompting him to file a complaint for
replevin.
In their answer, respondents claim that the equipment were corporate
property. The RTC ruled in favor of Yamamoto, CA reversed.

Issue:
Whether the advice in the letter allowing Yamamoto to retrieve the
equipment which were part of his investment bound the corporation.

Held:
NO. Where the lawyer of the controlling stockholder of the corporation
advised another stockholder that he could obtain possession of certain
9 | P a g e

corporate properties by way of return of his equity investment but the
lawyer acted without board approval, the advice is not binding on the
corporation even though it had the approval of the controlling stockholder.
The doctrine of piercing the veil of corporate fiction can not be invoked on
the sole ground that the presence of other stockholders in the corporation
was only for the purpose of complying with the statutory minimum
requirements on number of directors.

Delima v. Gois
Facts:
Delima filed a case for illegal dismissal against Golden Union Aquamarine
Corporation, Prospero and Susan Gois. The Labor Arbiter found Golden
Union liable to pay backwages. A vehicle was attached by the sheriff for the
satisfaction of the claim. Gois filed an Affidavit of Third Party claim, claiming
that the attachment of the vehicle was irregular because said vehicle was
registered in her name and not Goldens although, she let the corporation
use the same, she being a stockholder of the corporation; she further
alleged that she was not a party to the illegal dismissal case filed by Delima.
The LA denied her claimed on the ground that she was named as one of the
respondents and summons were validly served in her. On appeal, the NLRC
affirmed. CA reversed.

Issue:
Whether or not a stockholders property may answer for the debt of a
corporation

Held:
NO. the vehicle principally used in the business of the corporation but
registered under the name of the stockholder president can not be
garnished to satisfy the debt of the corporation. The rule is that obligations
incurred by the corporation, acting through its directors, officers and
employees, are its sole liabilities. Thus, property belonging to a corporation
cannot be attached to satisfy the debt of the a stockholder and vice versa,
the latter having only an indirect interest in the assets and business of the
former.

Excellent Quality Apparel Inc. v. Win Multi Rich Builders, Inc.
Facts:

17. PEA- PTGWO v. NLRC
Facts:
The Gonzales family owned both the Pantranco North Express Inc. (PNEI)
and Macris Realty Corporation. PNEI is engaged in public transportation
services to the public, its terminal stood on four lands registered under
Macris Realty Corporation.
The family failed to pay its obligations, this allowed the creditors to take
over both companies.
Sometime in 1978, PNEI and Macris were transferred to Natl Investment
Development Corporation (NIDC), a subsidiary of PNB. Macris name was
changed to National Realty Development Corporation (NAREDECO) and was
merged with the National Warehousing Corporation to form PNB-
MADECOR, the new PNB subsidiary.
Mega Prime acquired the shares of PNB over PNB-MADECOR among those
acquired by Mega Prime was PNEI.
PNEI was sold to North Express Transport which was sequestered by the
PCGG after the EDSA Revolution and was sold to the private sector through
the Asset Privatization Trust (APT).
Eventually, PNEI closed down and labour claims were filed against said
company. On July 5, 2002, the Labour Arbiter issued a writ of execution to
satisfy the claim of PNEIs former employees. The sheriffs were instructed to
proceed against PNB, PNB- MADECOR and Mega Prime. The sheriff levied
the real property on which the terminal stood; said properties were
subsequently sold to satisfy the claims of the labourers.
PNB questioned the sale on the ground that it has a right over the subject
properties since Mega Prime was indebted to it, and the sale of the said
properties would unduly prejudice the bank in the event that Mega Prime
fails to satisfy its obligations.

Issue:
Whether said properties can be attached to satisfy the claims against PNEI

Held:
NO. the labourers of the PNEI, a corporation which has ceased operations,
can not enforce their claims against PNB just because it acquired PNEI at the
time it was suffering financial reverses nor against PNB MADECOR just
because it is the owner of PNEI properties and a subsidiary of PNB nor
10 | P a g e

against Mega Prime just because the latter acquired the shares of PNB over
PNB- MADECOR. PNB- MADECOR and Mega Prime are corporations with
personalities separate and distinct from that of PNEI. The general rule is
that a corporation has a personality separate and distinct from those of its
stockholders and other corporations to which it may be connected.
Moreover, these corporations are registered as separate entities and,
absent any valid reason, their separate identities should be maintained and
should not be treated as one. Neither can the legal personality of PNEI be
merged with PNB simply because the latter acquired the former. Settled the
rule that where one corporation sells or otherwise transfers all of its assets
to another corporation for value, the latter is not by that fact alone, liable
for the debts and liabilities of the transferor. The execution sale on the
Pantranco properties to satisfy the labourers claims is null and void.
However, only PNB- MADECOR or its successors or assigns has the right to
annul the sale. PNB is not the real party in interest to question the sale just
because Mega Prime is indebted to it. PNBs right over the Pantranco
properties are only inchoate which could ripen into substantial interest only
if Mega Prime does not pay its indebtedness to PNB.

CLAIM FOR MORAL DAMAGES

Filipinas Broadcasting Network v. Ago Medical Center
Facts:
One of the broadcasters of Filipinas uttered libellous remarks over a radio
broadcast against Ago Medical and Educational Center. the broadcaster said
that Ago is the dumping ground of intellectual misfits; because of such
remark Ago medical sued Filipinas.

Issue:
Can a corporation be entitled to moral damages?

Held:
IT DEPENDS, IN THIS CASE YES. A juridical personality is generally not
entitled to moral damages because, unlike a natural person, it can not
experience physical suffering or such sentiments as wounded feelings,
serious anxiety, mental anguish or moral shock. Nevertheless, if a
corporations claim for moral damages falls under Section 7 Article 2219 of
the Civil Code which expressly authorizes recovery of moral damages in
cases of libel, slander, or any other form of defamation, then moral
damages may be awarded. This is because Article 2219 does not qualify
whether the plaintiff is natural or juridical person. Therefore a juridical
person such as a corporation can validly complain for libel or for any other
form of defamation and claim for moral damages.

MERALCO v. TEAM Electronics
Facts:
TEAM owns several buildings. They are leased by companies. Lessees were
ejected from the premises of TEAM and the latter occupied the same.
Meralco inspected the area however and found out that there are several
cables tampered with.
This caused Meralco to assess deficiency payment and directed their claims
against TEAM. TEAM refused to pay since according to the period of
assessment, it was the former lessee who occupied the premises who is
liable not TEAM. TEAM refused to pay.
Meralco disconnected the services without notice. The respondent paid
under protest.
The corporation subsequently filed a case for moral damages alleging
debased reputation by disconnection of electricity and thus resulting to
injurious acts towards the reputation of the corporation.

Issue:
Whether the company is entitled to moral damages

Held:
NO. The company is not entitled to moral damages absent proof by the
corporation that indeed their good name was debased by the acts of the
person complained. Corporations usually are not entitled to moral damages,
except when their good name is besmirched or debased resulting in insult
and humiliation to the business world. But the claimant must allege and
prove the same. Absent proof, the corporation cannot pursue and win a
claim for moral damages. Proof of such is imperative to justify the award.



11 | P a g e


DOCTRINE OF PIERCING THE CORPORATE VEIL

Villa Rey Transit, Inc. v. Eusebio Ferrer
Facts:
Jose Villarama was the operator of Villa Rey Transit; he sold two certificates
of public convenience to Pantranco with the condition among others that
the seller (Villarama) shall no for a period of 10 years from the date of sale,
apply for any TPU service identical or competing with the buyer.
Barely three months after, another Villa Rey Tansit was organized with
Natividad, Joses wife, and his brother in law, among the incorporators.
Villa Rey bought five certificate of public convenience from Public Service
Commission (PSC); it likewise bought equipment from Valentin Fernando.
Before the PSC could approve the sail, the sheriff levied two of the five
certificates pursuant to a writ of execution issued by the then CFI of
Pangasinan, in another civil case in favor of Eusebio Ferrer against Valentin
Fernando. Ferrer sold the certificates to Pantranco; Villa Rey filed an action
to annul the sale.
Pantranco filed a third party complaint against Villarama, it alleged that
Villarama and Villa Rey are one and the same, and as such, Villa Rey was
disqualified from operating the two certificates by virtue of the agreement
between Villarama and Pantranco, which by nature prohibits Villarama from
competing with Pantranco.

Issue:
Whether Villa Rey and Villarama are one and the same

Held:
YES. Villa Rey Transit is an alter ego of Jose Villarama, hence the restrictive
clause in the contract entered into by the latter and Pantranco is likewise
enforceable and binding against the said corporation. The rule is that a
seller or promisor may not make use of a corporate entity as a means of
evading the obligation of his covenant. Where the corporation is
substantially the alter ego of the covenantor to the restrictive agreement, it
can be enjoined from competing with the covenantee.





AC Ransom Labor Union- CCLU v. NLRC
Facts:
AC Ransom Labor Union filed a case for unfair labor practice against AC
Ransom Phil. Corporation.
During the pendency of the case, Rosario Industrial Corporation was
established by the same officers and Stockholders of AC Ransom; further it
was engaged in the same line of business as AC Ransom. It was alleged that
the assets were legitimately sold to Rosario together with the machineries.
NLRC found AC Ransom guilty and was ordered to pay employees unpaid
wages. However, it was granted clearance by the SEC to cease operations,
by virtue of which, AC Ransom did not pay the claims of the workers.
Writ of execution against AC Ransom were issued but to no avail. The union,
filed another writ of execution praying that the veil of corporate fiction be
pierced.

Issue:
Whether or not the veil of corporate fiction may be pierced

Held:
YES. Sale of corporate assets to another corporation organized previously by
the same officers of the vendor and engaged in the same line of business,
using the machineries of the vendor in the same factory, is an instance
where corporate veil should be pierced, vis a vis claim of the laborers for
backwages.

Shoemart Inc. v. NLRC
Facts:
Majority of the employees of Moris Industries, Inc. were members of Moris
Industries Workers Union (Union for brevity). The Union affiliated itself with
PAFLU.
The union through a letter invited Moris to enter into negotiations for a
collective bargaining agreement. However, after two days, Moris suddenly
closed shop and ceased operations, claiming that such closure was due to
business reverses.
As a result, the Union filed a complaint for unfair labor practice; Shoemart
was impleaded together with its president Henry Sy, it was argued that
12 | P a g e

since Shoemart is the exclusive buyer of all of Moris products and both are
housed in one building, and more importantly that the incorporators and
directors of Moris are the major stockholders of Shoemart, the two
therefore should be treated as one and the same.
Shoemart and Mr. Sy moved for the dismissal of the complaint on the
ground of lack of jurisdiction, there being no employment relationship
between Shoemart and the Union members; that its corporate relationship
was separate and distinct from that of Moris.

LA ruled that since Moris is owned by the Sy family, Shoemart is liable for
the claims against Moris. NLRC affirmed.

Issue:
Whether or not Shoemart is equally liable with Moris for unfair labor
practice, illegal termination of employment and non payment of the
benefits to the employees.

Held:
YES, THE SEPARATE JURIDICAL PERSONALITIES MAY BE DISREGARDED. The
Court took into account the close relationship between the parties. An
examination of the Incorporation Papers of Shoe mart and Moris show that
except for one all incorporators and directors of Moris are major
stockholders of Shoe Mart. Both are housed in one building, and that the
former is the exclusive buyer of the latter.
In disregarding their separate juridical personalities, Shoe mart can not be
compelled to open a manufacturing company to accommodate the
displaced employees of Moris considering the obvious difference and
diversity in skills, experience and orientation between the two corporations.

Reynoso v. Court of Appeals
Facts:
Reynoso was assigned as the resident manager of the franchise company of
Commercial Credit Corporation (CCC) in Quezon City (CCC-QC).
CCC formed CCC-Equity Corporation (CCC- Equity), a wholly owned
subsidiary, to which CCC transferred its 30% equity in CCC- QC, together
with two seats in the latters Board of Directors.
Under the new set up, employees including Reynoso from CCC-QC became
the employees of CCC- Equity. While Reynoso continued to be the manager
of CCC-QC, he drew his salaries and allowandes from CCC- Equity,
furthermore, he together with the rest of the employees from CCC- Qc
became qualified members of the CCC Employees Pension Plan.
Reynoso was dismissed from CCC- Equity for alleged embezzlement. A
Complaint for sum of money was filed by against him, it was alleged that
Reynoso embezzled the funds of CCC- QC and to use the same to purchase a
house. Reynoso for his part, filed a counter claim for damages.
The then CFI, dismissed the complaint and granted the counterclaim. The
judgment however was unsatisfied; the court then issued an alias writ of
execution against General Credit Corporation (GCC), the new name of CCC.
The CA nullified the lower courts finding.

Issue:
Whether the subsidiary may be considered as separate and distinct from the
mother corporation in the case at bar.

Held:
YES, BY WAY OF EXCEPTION. The defense of separateness will be
disregarded where the business affairs of the subsidiary corporation are so
controlled by the mother corporation to the extent that it becomes an
instrument or agent.
A subsidiary is considered a mere instrumentality of the parent company; if
the laters determines the personnel, administrative, finance policies, hires
the employees and funds the operations of the former.
The manager of subsidiary could therefor enforce his claim against the
parent company even though his employment is with the subsidiary.

Lipat v. Pacific Banking Corporation
Facts:
The Lipats owned Belas Export Trading (BET) a single proprietorship,
managed by their daughter; apart from which the spouses also owned
Mystical Fashions in the United States.
Estelita Lipat executed SPA appointing Teresita, her daughter as the
attorney- in- fact to obtain loans and execute mortgage on properties they
co- own. Teresita obtained a loan secured by a real estate mortgage over
the spouses property; said property also secured additional loans. Further,
Teresita also issued promissory notes and entered into Trust Receipt
agreement in behalf of BET.
13 | P a g e

Later on, BET was incorporated into a family corporation names Belas
Export Corporation (BEC) which was engaged in the same kind of business,
utilized the same machineries and equipment.
The loans were restructured in the name of BEC. BEC defaulted in its
obligations; the mortgaged was foreclosed and sold at public auction. A
certificate of sale was issued to Trinidad as the highest bidder.
The Lipats filed an action to annul the mortgage, on the ground that the
promissory notes, trust receipts and export bills were all ultra vires acts of
Teresita as they were executed without board resolution, and that assuming
said acts were valid, the same were the corporations sole obligation, it
having a personality distinct and separate from spouses Lipat.
RTC ruled that since BEC was a family corporation, it was a mere alter
extension of the spouses personality and business and a mere alter ego or
business conduit of the Lipats established for their own benefit. CA affirmed
and found that the corporations by laws even allowed Teresita to secure
loans without a board resolution.

Issue:
Whether the loan of a single proprietorship can be considered as loan of the
corporation

Held:
YES. When the corporation is the mere alter ego or business conduit of a
person, the separate personality of the corporation may be disregarded.
This is commonly referred to as the instrumentality rule or the alter ego
doctrine, which the courts have applied in disregarding the separate
juridical personality of corporations. Where the owner of the business
operating as a single proprietorship authorized her daughter to constitute a
mortgage on the proprietors property to secure a loan and the single
proprietorship was eventually formed into a corporation, the loans incurred
by the single proprietor should be considered obligations of the corporation
where the proprietor and her husband are the majority shareholders of the
corporation; both firms were managed by their daughter, engaged in the
same business and held office in the same building owned by the spouses;
and the corporation benefited from said loans, then the corporation should
be treated as a conduit of and having merely succeeded in the single
proprietorship. Thus, the obligations under the mortgage contract secured
under the name of the corporation cannot be evaded on the pretext that it
was signed for the benefit of the and under the name of the single
proprietorship only.

Times Transportation Company, Inc. v. Sotelo
Facts:
Times Employees Union (TEU) held a strike as a response to Times alleged
attempt to forma rival union and its dismissal of the employees identified to
be active union members. Times allegedly terminated the workers, and
while the Secretary of Labor issued a return to work order, the employees
were no longer admitted back to work.
In the middle of the labor dispute, Mencorp Transport Systems Inc. acquired
ownership over Times certificate of public convenience.
Mencorp is controlled and operated by Mrs. Mendoza, daughter of Santiago
Rondaris, the majority stockholder of Times.
The Labor Arbiter ordered reinstatement and payment of backwages by
Mencorp.

Issue:
Whether the veil of corporate fiction may be pierced

Held:
YES. Piercing the corporate veil is warranted of in the middle of a labor
dispute, a corporation sold its franchise to a company controlled by the
daughter of the controlling shareholder of the assignor corporation where
daughter is also a director. It is evident that the transaction was made in
order to remove the corporations remaining assets from the reach of any
judgment that may be rendered in the unfair labor practice case filed
against it.

Apex Mining Co., Inc. v. Southeast Mindanao Gold Mining Corp.
Facts:
In 1983, Camilo Banad with the natives established the Balite Communal
Portal Mining Corp.; in the same year, it entered into an operating
agreement with Apex.
Marcoper Mining Corp (MMC) was granted a permit, EP 133, which
embraced the areas claimed by Apex. The terms of the permit prohibit the
assignment thereof except in favor of an agent and the corporation, as
holder of the mining claim.
14 | P a g e

Marcoper likewise filed for the cancellation of the mining claims of Apex.
The Bureau of Mines and Geo. Science (BMG), dismissed MMCs petition.
When the case reached the DENR, it ruled that MMCs EP 133 is valid. The
MR was denied. The SC held that proper process should have been
followed, that application for permit should have been filed with the Bureau
of Forest Development (BFD) and not with the BMC.
In 1991, DENR issued DAO 66, declaring a portion of the contested area
open to all small scale mining purpose.
In 1994 a certain Villaflor filed a petition for the cancellation of EP 133. In
the same year and prior to the permits expiration, MMC assigned EP 133 to
Southeast Mindanao Mining Corp (SEM)- a domestic corporation alleged to
be a 100% owned subsidiary of MMC.
The lower court ruled the EP 133 is valid.
On appeal the CA invoked the doctrine of piercing the corporate veil to
legitimize the prohibited transfer, it stressed that SEM is just a business
conduit of MMC; hence the distinct legal personality should not be
recognized.

Issue:
Whether the veil of corporate fiction may be pierced

Held:
NO. Where the terms of the mining claim prohibit the assignment thereof
except in favor of an agent and the corporation, as holder of the mining
claim, transferred its right to a whollyowned subsidiary, the separate legal
personality of the subsidiary can not be pierced so that the latter shall be
considered a mere conduit or agent of the parent company to legitimize the
prohibited transfer. The doctrine of piercing the corporate veil cannot be
used as a vehicle to commit prohibited acts because these acts are the ones
which the doctrine seeks to prevent. To allow the subsidiary to avail itself of
this doctrine and to approve the validity of the assignment is tantamount to
sanctioning illegal act which is what the doctrine precisely seeks to forestall.

GCC v. Alson Development and Investment Corporation
Facts:
General Credit Corporation (GCC) established CCC franchise companies in
different parts of the country.
GCC applied for and was granted licenses from the CB and SEC to enable it
to engage in quasi- banking activities. Subsequently, CCC Equity Corporation
(EQUITY) was organized by GCC; to take over the operations and
management of the various franchise companies.
Alson Development and Investment Corporation (Alsons) and the Alcantara
family owned shares in the GCC franchise companies which they eventually
sold to EQUITY. EQUITY issued promissory notes as payment. Later on, the
Alcantara family assigned its rights over the PNs to Alsons.
EQUITY failed to pay the PNs upon maturity, this prompted Alsons to file a
complaint for sum of money against EQUITY, GCC was impleaded as party-
defendant on the ground that EQUITY was organized as a tool and mere
conduit of GCC.
RTC ruled in favour of Alsons, holding EQUITY and GCC solidarily liable. CA
affirmed.

Issue:
Whether Alsons could enforce payment of the PN of EQUITY against GCC

Held:
YES. Commonality of directors, officers and stockholders and even sharing
of office between GCC and EQUITY; certain financing and management
arrangements between the two, allowing GCC to handle the funds of the
latter; virtual domination if not control wielded by GCC over the finances,
business policies and practices of EQUITY; and the establishment of EQUITY
by GCC to circumvent CB rules are circumstances which justify the
conclusion that EQUITY is just an instrumentality of GCC. Thus, Alsons could
enforce payment of the PN even against GCC.

Note:
Whether the separate personality of the corporation should be pierced
hinges on obtaining facts appropriately pleaded or proved.
There are three basic areas where piercing the veil of corporate fiction may
be allowed; these are:
1. DEFEAT OF PUBLIC CONVENIENCE, as when the corporate
fiction is used as vehicle for the evasion of an existing
obligation;
2. FRAUD TEST, when the corporate entity is used to justify a
wrong, protect fraud or defend a crime; or
15 | P a g e

3. ALTER EGO TEST, where a corporation is merely a farce
since it is a mere alter ego or a business conduit of a person,
or where the corporation is so organized and controlled and
its affairs are so conducted as to make it merely an
instrumentality, agency or conduit or adjunct of another
corporation


Sta. Monica Industrial v. The DAR Regional Director
Facts:
Trinidad owned five parcels of land in Bulacan. De Guzman was the
agricultural leasehold tenant of Trinidad. The two signed a leasehold
contract, by virtue of which a Certificate of Land Transfer was issued in
favour of De Guzman.
De Guzman filed a petition for the issuance of patent in his name with the
Regional Director which was granted.
A year later, Sta. Monica filed a petition claiming that the said landholding
was sold to the corporation. Sta. Monica was organized by Trinidad, the
family owned and controlled the same.
Sta. Monicas petition was dismissed; MR was likewise denied.

Issue:
Whether the doctrine of corporate fiction should be applied

Held:
YES. The sale of agricultural land covered by the agrarian reform law by the
owner to a corporation owned and controlled by the same owner and his
family is null and void. The corporate vehicle can not be used to shield the
owner from the agricultural claims of the tenant- beneficiary. The veil of
corporate fiction should be pierced when it is used to subvert public policy.

Spouses Violago v. BA Finance Group
Facts:
Avelino Violago, President of Violago Motor Sales Corporation (VMSC) sold a
car to his cousin Pedro and his wife.
The spouses and Avelino signed a PN under which they bound themselves to
pay jointly and severally to the order of VMSC, the spouses likewise
executed a chattel mortgage over the car as security.
VMSC, though Avelino endorsed the PN to BA Finance without recourse.
After receiving a certain amount VMSC assigned its rights over the PN and
CM in favour of BA Finance. Meanwhile, the spouses remitted the down
payment to VMSC.
The spouses were unaware that the same car had already been sold to
another cousing of Avelino, Esmeraldo. Despite the spouses demand for the
vehicle and Avelinos repeated assurances, there was no delivery made, this
prompted the spouses to not pay any monthly amortization to BA Finance.
BA Finance for its part, filed a complaint against the spouses; the RTC ruled
in its favour.
In the meantime, Esmeraldo conveyed the vehicle to a certain Jose Olvido,
the latter executed a CM over the vehicle in favour of Lopez as security for a
loan, the PN of which was later endorsed to BA Finance.
CA nullified the RTC decision.

Issue:
Whether the veil of corporate fiction may be invoked and sustained despite
the fraud and deception of Avelino

Held:
NO. the facts of the case show that Avelino clearly defrauded the
petitioners. His actions were the proximate cause of petitioners loss; he can
not now hide behind the separate corporate personality of VMSC to escape
from the liability for the amount adjudged by the trial court in favour of the
petitioners.

NOTE:
CONTROL TEST is one of the tests applied to determine the applicability of
the doctrine of piercing the veil of corporate fiction; it is allowed when the
following requisites are complied with:
1. CONTROL, not mere majority or complete stock control, but
complete domination, not only of finances but of policy and
business practice in respect to the transaction attacked so that
the corporate entity as to this transaction had at the time no
separate mind, will or existence of its own;
2. Such control must have been used by the defendant to commit
fraud or wrong, to perpetuate the violation of a statutory or
16 | P a g e

other positive legal duty, or dishonest and unjust acts in
contravention of plaintiffs legal rights; and
3. The aforeasaid control and breach of duty must proximately
cause the injury or unjust loss complained of.

Siain Enterprises Inc. v. Cupertino Realty Inc. Co.
Facts:
Cua Le Leng was the president and majority stockholder of three affiliated
companies Siain Enterprises Inc. (SIAIN), Yuyek Manufacturing Corporation
and SIAIN Transport Inc.
As President of SIAIN, Cua Le Leng obtained a loan from Cupertino Realty
Corporation (CUPERTINO) which was secured by a PN. The proceeds of the
loan were divided among the three affiliated companies despite the fact
that it was obtained in the name of SIAIN.
As additional security SIAIN executed a Real Estate Mortgage (REM).
Contract of pledge was likewise executed over the properties registered in
the name of the two affiliate companies in the event of non payment.
Dispute arose between SIAIN and CUPERTINO as to whether the proceeds of
the loan were released.
In the meantime, CUPERTINO was already making arrangements with the
Notary Public for the extrajudicial sale of the mortgaged properties.
SIAIN filed a complaint with a prayer to restrain the Notary Public from
proceeding with the public auction. Petitioner insisted that it never received
the proceeds of the loan, CUPERTINO claimed otherwise

Issue:
Whether or not ), Yuyek Manufacturing Corporation and SIAIN Transport
Inc. may be held liable for the loan obligation incurred in the name of SIAIN
Enterprise

Held:
YES. It is clear from the records among others, that the three corporations
are characterized by oneness of operations vested in the person of their
common president, Cua Le Leng.
The conjunction of identity of three companies has been indubitably shown:
1. SIAIAN and Yuyek have a common set of incorporators, stockholders
and board of directors;
2. They have the same internal book keeper and accountant;
3. They have the same office address;
4. They have the same majority stockholders and President
5. In relation to SIAIN Transport, Cua Le Leng ha the unlimited
authority without authority from the Board to use the funds of
SIAIN Transport to pay the obligation incurred by petitioner
corporation.
When two corporations are characterized by oneness of operations vested
in the person of their common president and unity in the keeping and
maintenance of their corporate books and records through their common
accountant and bookkeeper, loans obtained by these corporations together
with their controlling shareholders shall be treated as one. Thus, even
though the mortgage of record is only one of the corporations, their
properties may be foreclosed to answer for all the loan obligations of the
corporations, their controlling shareholding and all those who received
certain amount of the loan for the account of the former.

GOVERNMENT CORPORATION

Ramon A. Gonzales v. PNB
Facts:
Petitioner instituted several cases against the bank questioning several
transactions entered into by the bank. In the course of the hearing, the
issue as regards his capacity to sue was raised; in response he acquired a
share of stock from Cong. Montano.
The court denied petitioner's prayer that he be allowed to examine and
inspect the books of the bank regarding the transaction entered by the bank
on the grounds that the right of a stockholder to inspect the record of the
business transactions of a corporation granted under Section 51 of the
former Corporation Law (Act No. 1459, as amended) is not absolute, but is
limited to purposes reasonably related to the interest of the stockholder,
must be asked for in good faith for a specific and honest purpose and not
gratify curiosity or for speculative or vicious purposes; that such
examination would violate the confidentiality of the records of the
respondent bank as provided in Section 16 of its charter, Republic Act No.
1300, as amended; and that the petitioner has not exhausted his
administrative remedies.

Issue:
17 | P a g e

whether or not petitioner, as stockholder, may inspect and examine the
books and records of the bank

Held:
NO. The Philippine National Bank, as a rule, is not governed by the
Corporation Code, but its charter. Thus the right of the stockholder to
examine corporate books under the Corporation Code does not apply to
PNB since under its charter, the records of PNB can only be assessed by
specified individuals.

Note:
Under PNB's charter, specifically Sections 15 and 16, the National Bank shall
be subject to inspection by the Department of Supervision and Examination
of the Central Bank. And that The Superintendent of Banks and the Auditor
General, or other officers designated by law to inspect or investigate shall
not reveal to any person other than the President of the Philippines, the
Secretary of Finance, and the Board of Directors the details of the inspection
or investigation, nor shall they give any information relative to the funds in
its custody, its current accounts or deposits belonging to private individuals,
corporations, or any other entity, except by order of a Court of competent
jurisdiction.

Francisca Baluyot v. Paul Holganza
Facts:
During a spot audit conducted by a team of auditors from the PNRC
headquartes a cash shortage of P154,350.13 was discovered in its Bohol
Chapter. Baluyot as the head of the Chapter was held accountable.
Holganza, as member of the board of directors of the chapter filed a
complaint against the petitioner in the Office of the Ombudsman charging
the latter with malversation.
Baluyot, in her counter affidavit claimed that the Ombudsman had no
jurisdiction over the action, since the Ombudsman had authority only over
GOCCs, which the PNRC was not.

Issue:
Whether or not the PNRC is a GOCC or a private corporation.

Held:
THE PNRC IS A GOCC WITH AN ORIGINAL CHARTER UNDER RA NO. 95.
The test to determine whether a corporation is government owned or
controlled, or private in nature is simple. Is it created by its own charter for
the exercise of a public function, or by incorporation under the general
corporation law? Those with special charters are government corporations
subject to its provisions, and its employees are under the jurisdiction of the
Civil Service Commission, and are compulsory members of the Government
Service Insurance System.
Clearly then, public respondent has jurisdiction over the matter.

Dante Liban v. Richard Gordon
Facts:
Petitioners as officers of the Board of Directors of the QC Red Cross Chapter
filed with the court a petition to declare Gordon as Having Forfeited his seat
in the Senate.
While Gordon was a senator he was elected as Chairman of the PNRC.
Petitioners alleged that by accepting the chairmanship, Gordon has ceased
to be a member of the Senate as provided in Section 13, Article 6of the
Constitution which provides that:

Section 13: No Senator or member of the House may hold any other office
or employment in the Government..including GOCCs durign his term
without forfeiting his seat.

The petitioners cite Camporedondo v. NLRC which held that the PNRC is a
GOCC. Petitioners then claim that respondent has automatically forfeited
his seat in the senate pursuant to Flores v. Drilon which held that incumbet
national legislators lose their elective posts upon their appointment to
another government office.

Issue:
Whether the PNRC is a GOCC

Held:
PNRC IS A PRIVATE ORGANIZATION PERFORMING PUBLIC FUNCTIONS.
The PNRC was created pursuant to the Geneva Convention. Its mission is to
bring timely, effective and compassionate humanitarian assistance to the
most vulnerable without regard of nationality, race, religion, gender, social
18 | P a g e

status or political affiliation . To achieve such, autonomy is fundamental.
The PNRC must be autonomous, neutral and independent in order to
conduct its activities. It must not appear to be an instrument or agency that
implements government policy.
In ruling that the PNRC is a private organization, the court put into
consideration the following:

1. The PNRC does not have government assets and does not receive
appropriation from the Congress. It is financed primarily by contributions
through solicitation.
2. The Government has no control over the PNRC. Under its charter only 6
of the 30 members of the Board are appointed by the President. Four fifths
of the Board are elected by the private sector.
3. Senator Gordon was elected as Chairman by a private sector controlled
PNRC Board.
4. The PNRC chairman is not appointed by the President or by any
subordinate government official. Thus, the PNRC Chairman is not an official
or employee of the Philippine Government.
The Camporedondo ruling which stated that PNRC is a GOCC failed to take
into consideration the definition of a GOCC under the Administrative Code:
GOCC- ...owned by the Government directly or through its instrumentality
either wholly or in the case of stock corporations, to the extent of at least
51% of its capital stock.
As regards the issue that the PNRC Charter is violative of the Constitutional
Proscription against the Creation of Private Corporations by Special Law, the
court held that the charter in so far as it creates the PNRC as a private
corporation and grants it corporate powers is void in accordance with the
Constitutional Proscription. As such, the PNRC should incorporate under the
Corporation Code and register with the SEC.

Note:
The dissenting opinion said that a GOCC may either be a stock or non-stock
corporation, or that it may be further categorized, suggesting that the
definition provided in the Administrative Code is broad enough to admit of
other distinctions as to the kinds of GOCCs.

The dissenting opinion said that PNRC is at the very least a government
instrumentality, citing Section 2(10) of Executive Order No. 292

Instrumentality refers to any agency of the National Government not
integrated within the department framework, vested with special functions
or jurisdiction by law, endowed with some if not all corporate powers,
administering special funds, and enjoying operational autonomy, usually
through a charter. This term includes regulatory agencies, chartered
institutions and government-owned or controlled corporations.

Engr. Feliciano v. Commission on Audit
Facts:
A special audit team from COA audited the accounts of Leyte Metropolitan
Water District (LMWD). Feliciano as General Manager of LMWD refused to
pay the auditing fees, he further requested COA to refund all the auditing
fees previously paid to COA. Petitioner's contention is that LMWD is a
private corporation, thus it cannot be subjected to auditing by COA.

Issue:
Whether a Local Water District created under PD 198 is a GOCC subject to
the audit jurisdiction of COA.

Held:
LWD IS A GOCC WITH AN ORIGINAL CHARTER.
Congress can not enact a law creating a private corporation with a special
charter. Such would be unconstitutional. Private corporation may exist only
under a general law.

INCORPORATION

Mariano Albert v. University Publishing Co.,Inc.
Facts:
On September 1949, Albert sued University Publishing Co., Inc. Plaintiff
alleged that the corporation through its President, Aruego entered into a
contract with him; that defendant would pay P30,000 for te exclusive right
to published his revised commentaries on the Revised Penal Code. The court
ruled in his favor. Plaintiff now moves for execution against Aruego because
it was discovered that no such entity as University Publishing. Aruego now
claims that he is not a party to the case.

19 | P a g e

Issue:
Whether the judgment may be executed against Aruego supposed President
of University Publishing Co., Inc., as the real defendant.

Held:
YES. Where a corporation through its president entered into a contract but
the corporation failed to comply with its obligation under said contract,
judgment may be enforced against the President who participated all
through out the hearings when it turned out that the corporation is not duly
registered with the SEC. The real defendant in a suit against a corporation is
the person who has control of its proceedings.

MSCI- NACUSIP Local Chapter v. National Wages Productivity Commission
and Monomer Sugar Central, Inc.
Facts:
Asturias Sugar Central Inc. (ASCI) executed a MOA with Monomer Trading
Industries, Inc. (MTII) whereby by deed of assignment MTII shall acquire the
assets of ASCI and that a new corporation in place of MTII shall be
establishes which will be the new assignee of the assets of ASCI.
By virtue of the MOA, Monomer Sugar Central Inc, (MSCI) was organized
and incorporated.
Sometime in 1991, MSCI claiming to be a distressed employer, applied for
exemption from the coverage of Wage Order No RO V1-01. MSCI claimed
that its paid up capital is only 5M The petitioner, opposed claiming that
MSCI is not distressed.
The Board denied MSCI's application because the loss reported constituted
only 5.25% of the corporation's paid up capital which is not sufficient to
meet the required 25%.
The Board held that the paid-up capital of MSCI on the aforesaid dates was
actually P64,688,528.00. This conclusion is anchored on the belief of the
Board that the value of the assets of ASCI, party to the Memorandum of
Agreement, transferred to MSCI on March 28, 1990 should be taken into
consideration in computing the paid-up capital of MSCI to reflect its true
financial structure. Moreover, the loans or advances extended by MTII, the
other party to the Agreement, to MSCI should allegedly be treated as
additional investments to MSCI and must therefore be included in
computing respondent's paid-up capital.
Commission reversed and granted the exemption.

Issue:
What is the correct paid up capital of MSCI

Held:
Paid-up capital is that portion of the authorized capital stock which has
been both subscribed and paid. MSCI was organized and incorporated on
February 15, 1990 with an authorized capital stock of P60 million, P20
million of which was subscribed. Of the P20 million subscribed capital stock,
P5 million was paid-up.

Note:
Not all funds or assets received by the corporation can be considered paid-
up capital. Such must form part of the authorized capital stock of the
corporation, subscribed and then actually paid up.

Samahan ng mga Optometrists sa Pilipinas Ilocos Sur- Abra Chapter v.
Acebedo Internation Corporation
Facts:
Acebedo filed an application with the Office of the Mayor of Candon, Ilocos
Sur for the issuance of a permit for the operation of a branch of Acebedo
Optical in the municipality. Petitioner opposed on the ground that Acebedo
is a juridical entity not qualified to practice optometry.
The committee created by the Mayor of Candon to pass on Acebedos
application denied the same and ordered the closure of Acebedo optical
shops. Acebedo appealed but its appeal was dismissed by the trial court on
the ground that it was practicing optometry. On appeal, the Court of
Appeals held that Acebedo was not operating as an optical clinic nor
engaged in the practice of optometry, although it employed licensed
optometrists. Acebedo simply dispensed optical and ophthalmic
instruments and supplies.

Issue:
May corporations, engaged in the business of selling optical wares, supplies,
substances and instruments which, as an incident to and in the ordinary
course of the business hire optometrists, be said to be practicing the
profession of optometry which, by legal mandate, may only be engaged in
by natural persons possessed of specific legal qualifications?
20 | P a g e


Held:
NO. The fact that private respondent hires optometrists who practice their
profession in the course of their employment in private respondent's optical
shops, does not translate into a practice of optometry by private
respondent itself. Private respondent is a corporation created and organized
for the purpose of conducting the business of selling optical lenses or
eyeglasses, among others. The clientele of private respondent understably,
would largely be composed of persons with defective vision and thus need
the proper lenses to correct the same and enable them to gain normal
vision. The determination of the proper lenses to sell to private
respondent's clientele entails the employment of optometrists who have
been precisely trained for that purpose. Private respondent's business is not
the determination itself of the proper lenses needed by persons with
defective vision. Private respondent's business, rather, is the buying and
importing of eyeglasses and lenses and other similar or allied instruments
from suppliers thereof and selling the same to consumers.chanro


Dr. Alfafara et. al. v. Acebedo Optical Co., Inc.
Facts:
Petitioners sought to enjoin Respodent and its agents as well as its
employees from practicing optometry in Cebu.
After hearing, judgment was rendered in favor of the petitioners. The trial
court found that the hiring of licensed optometrists by the respondent was
unlawful because it resulted to the practice of optometry by the
respondent.

Issue:
Whether Acebedo Optical is guilty of practicing optometry?

Held:
NO. R.A. No. 1998 does not prohibit corporations from employing licensed
optometrists. What it prohibits is the practice of optometry by individuals
who do not have a license to practice. The prohibition is addressed to
natural persons who are required to have a valid certificate of registration
as optometrist and who must be of good moral character. This Court
affirmed the ruling of the appeals court and explained that even under R.A.
No. 8050 (Revised Optometry Law) there is no prohibition against the hiring
by corporations of optometrists. The fact that Acebedo hired optometrists
who practiced their profession in the course of their employment in
Acebedos optical shops did not mean that it was itself engaged in the
practice of optometry.

Islamic Directorate of the Philippines v. Court of Appeals
Facts:
Sometime in 1971 Islamic leaders of all Muslim major tribal groups in the
Philippines organized and incorporated the Islamic Directorate of the
Philippines (IDP).
In 1972, Martial Law was declared. Most of the Board of Trustees flew to
the Middle East to escape political persecution.
Two groups emerged, the IDP-Abbas group and the Carpizo Group, both
claimed to be the legitimate IDP and both have elected their respective
Board members. SEC however ruled that the election of both the Carpizo
and Abbas group of board members are null and void, because before any
election of the members of the Board of Trustees could be conducted, there
must be an approved by-laws of the corporation.
No by- laws were adopted.
The Carpizo group signed an alleged Board resolution authorizing the sale of
a parcel of land to Iglesia ni Kristo for 22M. Tamano group filed a petition
with the SEC to nullify said sale.
Meanwhile, INC filed for specific performance against Carpizo group to
compel the latter to clear the property of squatters and to compel mortagee
Ligon to deliver the TCT. Tamano group sought to intervene. INC opposed
the motion arguing, inter alia, that the issue sought to be litigated by way of
intervention is an intra-corporate dispute which falls under the jurisdiction
of the SEC. RTC ruled that the Tamano Group had no juridical personality, it
then rendered partial judgmennt ordering the Carpizo Group to comply with
its obligation under the Deed of Sale. RTC likewise ordered Ligon to deliver
the TCT. Ligon filed a case with the SC and Tamano group likewise filed its
own petition with the SC, however when the latter petition was pending,
the SC denied Ligon's petition. Does the decision in Ligon's petition works to
bar the petition filed by Tamano?

Issue:
Whether res judicata applies to the case at bar
21 | P a g e

Whether or not the sale was valid

Held:
NO. A juridical person can not be considered essentially a formal party to a
case where it was not duly represented by its legitimate governing board. It
can not be said that the corporation is represented by a legitimate board in
an action involving the validity of the sale of its corporate property when
the corporation has two sets of board of directors both claiming to be the
legitimate board and the issue on who is the legitimate board is still pending
at the time of the sale. Any decision in a court suit does not become final
and executory insofar as the corporation is concerned because it was
effectively denied its day in court for want of legitimate representation.
NO. where the only assets of the corporation are two parcels of land, their
sale without the approval of at least 2/3 of the members is invalid under
Section 40 of the Corporation Code

International Express Travel and Tour Services, Inc. v. Court of Appeals
Facts:
IETTSI wrote a letter to the Philippine Football Federation (PFF) through its
President, Henri Kahn, offering its services as a travel agency to the latter.
The offer was accepted. IETTSI secured the airline tickets for the trips of the
athletes. PFF made two partial payments; thereafter no further payments
were made despite repeated demands. This prompted IETTSI to file an
action for collection; IETTSI sought to hold Kahn liable for the unpaid
balance on the ground that he allegedly guaranteed the said obligation.
The trial court ruled in favor of IEETSI; only Kahn elevated the case to the
CA; which reversed the trial court on the basis of the doctrine of corporation
by estoppel.

Issue:
1. Whether the Philippine Football Federation has a corporate
existence of its own
2. Whether Kahn should be made personally liable for the unpaid
obligations of Philippine Football Federation
3. Whether the appellate court properly applied the doctrine of
corporation by estoppel

Held:
1. NO IT HAS NO CORPORATE EXISTENCE OF ITS OWN. Both RA 3135 (The
Revised Charter of the Philippine Amateur Athletic Federation) and PD
604 recognized the juridical existence of national sports associations.
This may be gleaned from the powers and functions granted to these
associations. The powers and functions granted to national sports
associations indicate that these entities may acquire a juridical
personality. However, while national sports associations may be
accorded corporate status, such does not automatically take place by
the mere passage of law. It is a basic postulate that before a corporation
may acquire juridical personality, the State must give its consent either
in the form of a special law or a general enabling act. The Federation did
not come into existence upon the passage of these laws. These laws
merely recognized the existence of national sports associations and
provided the manner by which these entities may acquire juridical
personalities. Section 11 of RA 3135 and Section 8 of PD 604 require
that before an entity may be considered as a national sports
association, such entity must be recognized by the accrediting
organization. Henri Kahn failed to prove this fact of recognition. A copy
of the constitution and by laws does not prove such recognition.
Accordingly, the Philippine Football Federation is not a national sports
association within the purview of the aforementioned laws and does
not have corporate existence of its own.

2. YES. HENRI KAHN SHOULD BE MADE LIABLE FOR THE UNPAID
OBLIGATIONS OF THE UNINCORPORATED FEDERATION. It is settled
principle in corporation law that any person acting or purporting to act
on behalf of a corporation which has no valid existence assumes such
privileges and becomes personally liable for the contract entered into or
for other acts performed as such agent. As President of the federation,
Henri Kahn is presumed to have known about the corporate existence
or non-existence of the Federation.

3. NO. the court cannot subscribe to the position taken by the appellate
court that even assuming that the Federation was defectively
incorporated, IETTSI cannot deny the corporate existence of the
corporation because it had contracted and dealt with the Federation in
such a manner as to recognize and in effect admit its existence. The
doctrine of corporation by estoppel is mistakenly applied by the
22 | P a g e

appellate court to IETTSI. The application of the doctrine applies to a
third party only when he tries to escape liabilities on a contract from
which he has benefited on the irrelevant ground of defective
incorporation. Herein, IETTSI is not trying to escape liability from the
contract but rather is the one claiming from the contract.


CORPORATE NAME

Republic Planters Bank v. Court of Appeals
Facts:
Canlas and Yamaguchi were officers of Worldwide Garment Manufacturing.
By virtue of a Board Resolution the two were authorized to apply for credit
facilities with Planters Bank. Nine PNs were issued in the name of
Worldwide.
On December 220, 1982, Worldwide changed its corporate name to Pinch.
The notes were not paid upon maturity, this prompted the bank to file an
action for recovery which was originally brought against Worldwide but was
after amended to Pinch.
The CA ruled that change of corporate name extinguished the corporate
personality of the original corporation.

Issue:
Whether or not a change in corporate name extinguishes the personality of
the original corporation
Held:
NO. a change in corporate name does not make it a new corporation and
does not affect its properties, rights and liabilities.



Industrial Refractories Corporation v. Court of Appeals
Facts:
Refractories Corporation of the Philippines (RCP) was organized on October
1976. On June 1977, it registered its corporate and business name with the
Bureau of Domestic Trade.
Industrial Refractories Corp. of the Philippines (IRCP) on the other hand, was
incorporated on August 1979 originally under the name "Synclaire
Manufacturing Corporation". It amended its Articles of Incorporation on
August 1985 to change its corporate name to "Industrial Refractories Corp.
of the Philippines".
Both RCP and IRCP were the only local supplier of monolithic gunning mix.
RCP filed with the SEC a petition to compel IRCP to change its corporate
name.
The SEC decided in favor of RCP.
SEC En Banc modified the appealed decision, IRCP was ordered to delete or
drop from its corporate name only the word "Refractories". IRCP elevated
the decision to the Court of Appeals which then rendered the decision,
denying to give due course the petition filed by IRCP, it ruled that the
corporate names of IRCP and RCP are confusingly or deceptively similar, and
that RCP has established its prior right to use the word "Refractories" as its
corporate name.

Issue:
Whether the corporate names of IRCP and RCP are confusingly similar.

Held:
YES. Refractories Corporation of the Philippines (RCP) is confusingly similar
with Industrial Refractories Corporation of the Philippines (IRCP). Being the
prior registrant, the RCP has acquired the right to use the word
Refractories as part of its corporate name.

PC Javier &Sons Inc. v. Court of Appeals
Facts:
PC Javier & Sons applied with First Summa Bank for a loan accommodation
under the Industrial Guarantee Loan Fund (IGLF).
The corporation through Pablo Javier was advised that its loan application
was approved and that the same shall be forwarded to the Central Bank for
processing.
The CB released the loan.
To secure the loan, Javier executed CM over some machinery in favor of the
bank. In the meantime, the bank changed its name to PAIC Savings and
Mortgage Bank Inc.
Thereafter, the corporation failed to pay; this prompted the Bank to move
for the extrajudicial foreclosure of the mortgages.
23 | P a g e

PC Javier filed an action to restrain the extrajudicial foreclosure on the
ground that it First Summa and PAIC Bank are separate entities.

Issue:
Whether the debtor should be formally notified of the corporate creditors
change of name

Held:
NO. There is no such requirement under the law or any regulation ordering
a bank that changes its corporate name to formally notify all its debtors.
This being the case, the court cannot impose on the bank that changes its
corporate name to notify its debtors of such change absent any law,
circular or regulation requiring it. Formal notification is therefore
discretionary on the bank.


RESIDENCE AND NATIONALITY OF A CORPORATION

Hyatt Elevators and Escalator Corp. v. Goldstar Elevators
Facts:
HYATT is a domestic corporation with address at the 6
th
Floor, Dao I
Condominium, Salcedo St., Legaspi Village, Makati, as stated in its Articles of
Incorporation.
HYATT filed a Complaint for unfair trade practices and damages against LG
Industrial Systems Co. Ltd. (LGISC) and LG International Corporation (LGIC)
in RTC Mandaluyong, alleging among others, that LGISC and LGIC, in the
middle of negotiations, terminated the proposal to the prejudice of HYATT.
In the course of the proceedings, HYATT filed a motion for leave of court to
substitute LGISC to LG OTIS, it alleged that subsequent to the filing of the
complaint, it learned that LGISC transferred all its organization, assets and
goodwill, as a consequence of a joint venture agreement with Otis Elevator
Company of the USA, to LG Otis Elevator Company (LG OTIS, for brevity). It
was further alleged that GOLDSTAR was being utilized by LG OTIS and LGIC
in perpetrating their unlawful and unjustified acts against HYATT, because
GOLDSTAR is being managed and operated by the same Korean officers of
defendants LG-OTIS Elevator Company and LG International Corporation.
GOLDSTAR filed for a MTD on the ground that the venue was improperly
laid.

Issue:
Whether the venue was improper

Held:
YES. The residence of a corporation is the place where its principal office is
located, as stated in its Articles of Incorporation even though the
corporation has closed its office therein and relocated to another place.

Unchuan v. Lozada
Facts:
Sisters Anita and Peregrina who were based in the US sold to their nephew
Antonio several parcel of lots which were located in the Philippines. Dr.
Lozada, their brother who was an American citizen agreed to advance the
purchase price for Antonio.
Antonio and Dr. Lozada agreed that said subject properties would be the
capital of Damasa Corporation. According to their agreement, Antonio and
Dr. Lozada are to hold 60% and 40% of the shares in said corporation,
respectively.
The Deed of Sale was later notarized and authenticated and was sent to
Antonio in the Philippines. Upon receipt of said documents, Antonio
recorded the sale with the RD.
Pending registration of the deed, Unchuan caused the annotation of
adverse claim on the lots. Marissa claimed that Anita donated an undivided
share in the lots to her; she likewise raised the issue that it was Dr. Lozada
an American citizen who paid for said lots and that Antonio was merely a
dummy of the former.
The court declare Antonio to be the absolute owner of the subject
properties.
On Motion for Reconsideration, the court reversed and cancelled the titles
in Antonios name.

Issue:
Whether the sale violated the public policy prohibiting aliens from owning
lands


Held:
24 | P a g e

NO. the sale to a foreigner of parcels of lands does not violate the public
policy prohibiting aliens from owning lands in the Philippines when the
foreigner advanced the money for the payment thereof but at no point
were the lots registered in his name nor was it contemplated that the lots
be under his control for they are actually to be included as capital of a
private corporation to be formed with a Filipino individual who will own
60% of the corporation with the foreigner holding 40% thereof. Under RA
7042, a corporation organized under the laws of the Philippines of which at
least 60% of the outstanding capital stock and entitled to vote is owned and
held by citizens of the Philippines, is considered a Philippine national. As
such, the corporation ,ay acquire disposable lands in the Philippines.

CORPORATION BY ESTOPPEL

Pioneer Surety & Insurance Corporation v. Court of Appeals
Facts:
Japan Domestic Airlines (JDA) and Jacob Lim entered into a contract of sale
of 2 aircraft DC-3A for US$ 109,000.
Petitioner executed a surety bond in favor of JDA on behalf of Lim as
principal in the name of Southern Airlines.
Border Machinery & Heavy Equipments Corp, Francisco and Modesto
Cervantes, Costancio Maglana contributed funds to the endeavor and
guaranteed the obligation.

Issue:
Whether or not a partnership is formed when funds were invested in a
corporation that was not created.

Held:
NO. Where someone convinced other parties to contribute funds for the
formation of a corporation which was never formed, there is no partnership
among them, and the latter cannot be held liable to share in the losses of
the proposed corporation.

People v. Garcia
Facts:
Complainants in this case applied to a recruitment corporation named
Ricorn Philippine International Shipping Lines, Inc. Garcia and Botero
represented themselves as president and vice-president of the corporation.
After the complainants sent the required documents for their respective job
applications, they returned to the purported main office of the corporation
but they discovered that Ricorn abandoned its office.
After further investigation, they found out that Ricorn was not registered in
SEC and was not also licensed in DOLE.
During trial, accused Garcia testified that he knew that the group
representing Ricorn was not registered ion SEC and DOLE.

Issue:
Whether or not a person representing himself as a corporation can be liable
as a general partner.

Held:
YES. The persons who illegally recruited workers for overseas employment
by representing themselves to be officer of a corporation which they knew
had not been incorporated are liable as general partners for all debts,
liabilities and damages incurred or arising as a result thereof.

Lozano v. Delos Santos
Facts:
Lozano alleged that he was the president of the Kapatirang Mabalacat-
Angeles Jeepney Drivers' Association, Inc. (KAMAJDA) while respondent
Anda was the president of the Samahang Angeles-Mabalacat Jeepney
Operators' and Drivers' Association, Inc. (SAMAJODA).
Upon the request of the Sangguniang Bayan of Mabalacat, Pampanga,
Lozano and Anda agreed to consolidate their respective associations and
form the Unified Mabalacat-Angeles Jeepney Operators' and Drivers'
Association, Inc. (UMAJODA); they also agreed to elect one set of officers
who shall be given the sole authority to collect the daily dues from the
members of the consolidated association.
Elections were held on October 29, 1995 and both petitioner and private
respondent ran for presidency.
Petitioner won; private respondent protested and, alleging fraud, refused
to recognize the results of the election; private respondent also refused to
abide by their agreement and continued collecting the dues from the
25 | P a g e

members of his association despite several demands to desist. Petitioner
was thus constrained to file the complaint to restrain private respondent
from collecting the dues.
Private respondent moved to dismiss the complaint for lack of jurisdiction,
claiming that jurisdiction was lodged with the Securities and Exchange
Commission (SEC).

Issue:
1. Whether or not the case is classified as an intra-corporate
controversy thus falling within the jurisdiction of the SEC?
2. Whether or not the proposed consolidated corporation may be
considered a corporation by estoppel

Held:
1. THE SEC HAS NO JURISDICTION OVER THE COMPLAINT. There is no
intracorporate nor partnership relation between petitioner and private
respondent. The controversy between them arose out of their plan to
consolidate their respective jeepney drivers' and operators' associations
into a single common association. This unified association was,
however, still a proposal. The KAMAJDA and SAMAJODA to which
petitioner and private respondent belong are duly registered with the
SEC, but these associations are two separate entities. It is between
members of separate and distinct associations. Petitioner and private
respondent have no intracorporate relation much less do they have an
intracorporate dispute. The SEC therefore has no jurisdiction over the
complaint.
2. NO. The doctrine of corporation by estoppel advanced by private
respondent cannot override jurisdictional requirements. Jurisdiction is
fixed by law and is not subject to the agreement of the parties. It cannot
be acquired through or waived, enlarged or diminished by, any act or
omission of the parties; neither can it be conferred by the acquiescence
of the court. Corporation by estoppel is founded on principles of equity
and is designed to prevent injustice and unfairness. It applies when
persons assume to form a corporation and exercise corporate functions
and enter into business relations with third persons. Where there is no
third person involved and the conflict arises only among those assuming
the form of a corporation, who therefore know that it has not been
registered, there is no corporation by estoppel.


Lim Tong Lim v. Philippine Fishing Gear Industries
Facts:
On behalf of "Ocean Quest Fishing Corporation," Antonio Chua and Peter
Yao entered into a Contract, for the purchase of fishing nets from the
Philippine Fishing Gear Industries, Inc. (PFGI). They claimed that they were
engaged in a business venture with Lim Tong Lim, who however was not a
signatory to the agreement.
The buyers, however, failed to pay for the fishing nets and the floats; hence,
PFGI filed a collection suit against Chua, Yao and Lim Tong Lim with a prayer
for a writ of preliminary attachment. The suit was brought against the three
in their capacities as general partners, on the allegation that "Ocean Quest
Fishing Corporation" was a nonexistent corporation as shown by a
Certification from the SEC.
The lower court issued a Writ of Preliminary Attachment, which the sheriff
enforced by attaching the fishing nets on board F/B Lourdes which was then
docked at the Fisheries Port, Navotas, Metro Manila. Lim Tong Lim, on the
other hand, filed an Answer with Counterclaim and Crossclaim and moved
for the lifting of the Writ of Attachment.
Lim argues, among others, that under the doctrine of corporation by
estoppel, liability can be imputed only to Chua and Yao, and not to him.

Issue:
Whether Lim should be held jointly liable with Chua and Yao.

Held:
In the first instance, an unincorporated association, which represented itself
to be a corporation, will be estopped from denying its corporate capacity in
a suit against it by a third person who relied in good faith on such
representation. It cannot allege lack of personality to be sued to evade its
responsibility for a contract it entered into and by virtue of which it received
advantages and benefits. On the other hand, a third party who, knowing an
association to be unincorporated, nonetheless treated it as a corporation
and received benefits from it, may be barred from denying its corporate
existence in a suit brought against the alleged corporation. In such case, all
those who benefited from the transaction made by the ostensible
corporation, despite knowledge of its legal defects, may be held liable for
26 | P a g e

contracts they impliedly assented to or took advantage of. There is no
dispute that PFGI is entitled to be paid for the nets it sold. The only question
here is whether Lim should be held jointly liable with Chua and Yao. Lim
contests such liability, insisting that only those who dealt in the name of the
ostensible corporation should be held liable. Although technically it is true
that Lim did not directly act on behalf of the corporation; however, having
reaped the benefits of the contract entered into by persons with whom he
previously had an existing relationship, he is deemed to be part of said
association and is covered by the scope of the doctrine of corporation by
estoppel.










DE FACTO CORPORATION

Sawadjaan v. Court of Appeals
Facts:
Sappari K. Sawadjaan was among the first employees of the Philippine
Amanah Bank (PAB) when it was created. He rose through the ranks,
working his way up from his initial designation as security guard.
In February 1988, while still designated as appraiser/investigator,
Sawadjaan was assigned to inspect the properties offered as collaterals by
Compressed Air Machineries and Equipment Corporation (CAMEC) for a
credit line of Five Million Pesos secure by REM over the latters poperties.
On the basis of his Inspection and Appraisal Report, the PAB granted the
loan application.
In the meantime, Sawadjaan was promoted to Loans Analyst I.
In January 1990, Congress passed Republic Act 6848 creating the AIIBP and
repealing P.D. No. 264 (which created the PAB). By virtue of which all assets,
liabilities and capital accounts of the PAB were transferred to the AIIBP, and
the existing personnel of the PAB were to continue to discharge their
functions unless discharged. In the ensuing reorganization, Sawadjaan was
among the personnel retained by the AIIBP.
When CAMEC failed to pay despite the given extension, the bank, now
referred to as the AIIBP, discovered that TCT No. N-130671 was spurious,
the property described therein non-existent, and that the property covered
by TCT No. C-52576 had a prior existing mortgage in favor of one Divina
Pablico.
The Board of Directors of the AIIBP created an Investigating Committee to
look into the CAMEC transaction. They found petitioner guilty of conduct
prejudicial to the best interest of the service. The board suspended the
petitioner, prompting the latter to appeal the decision citing AIIBPs lack of
legal standing to sue since it was not able to file its by-laws within the
prescribed period.

Issue:
Whether a corporation which failed to file its by-laws within the prescribed
period ipso facto lose its power as such

Held:
NO. At the very least, by its failure to submit its by-laws on time, the AIIBP
may be considered a de facto corporation whose right to exercise corporate
powers may not be inquired into collaterally in any private suit to which
such corporations may be a party. Moreover, a corporation which has failed
to file its by-laws within the prescribed period does not ipso facto lose its
powers as such. The SEC Rules on Suspension/Revocation of the Certificate
of Registration of Corporations, details the procedures and remedies that
may be availed of before an order of revocation can be issued. There is no
showing that such a procedure has been initiated in this case.


BOARD OF DIRECTORS

Gokongwie v. SEC
Facts:
John Gokongwei Jr., as stockholder of San Miguel Corporation, filed with the
SEC a petition for "declaration of nullity of amended by-laws, cancellation of
certificate of filing of amended by-laws, injunction and damages with prayer
27 | P a g e

for a preliminary injunction" against the majority of the members of the
Board of Directors and San Miguel Corporation as an unwilling petitioner.
Gokongwei alleged that the Board amended the bylaws of the corporation,
prescribing additional qualifications for its directors, that no person shall
qualify or be eligible for nomination if he is engaged in any business which
competes with that of the Corporation.
The board based their authority to do so on a resolution of the stockholders.
It was contended that according to section 22 of the Corporation Law and
Article VIII of the by-laws of the corporation, the power to amend, modify,
repeal or adopt new by-laws may be delegated to the Board of Directors
only by the affirmative vote of stockholders representing not less than 2/3
of the subscribed and paid up capital stock of the corporation, which 2/3
should have been computed on the basis of the capitalization at the time of
the amendment. Since the amendment was based on the 1961
authorization, Gokongwei contended that the Board acted without
authority and in usurpation of the power of the stockholders.
Gokongwei claimed that prior to the questioned amendment, he had all the
qualifications to be a director of the corporation, being a substantial
stockholder thereof; that as a stockholder, Gokongwei had acquired rights
inherent in stock ownership, such as the rights to vote and to be voted upon
in the election of directors; and that in amending the by-laws, Soriano, et.
al. purposely provided for Gokongwei's disqualification and deprived him of
his vested right as afore-mentioned, hence the amended by-laws are null
and void.
As additional causes of action, it was alleged that corporations have no
inherent power to disqualify a stockholder from being elected as a director
and, therefore, the questioned act is ultra vires and void.

Issue:
Whether the corporation has the power to provide for the (additional)
qualifications of its directors

Held:
YES. It is recognized by all authorities that "every corporation has the
inherent power to adopt by-laws 'for its internal government, and to
regulate the conduct and prescribe the rights and duties of its members
towards itself and among themselves in reference to the management of its
affairs.'" In this jurisdiction under section 21 of the Corporation Law, a
corporation may prescribe in its by-laws "the qualifications, duties and
compensation of directors, officers and employees." This must necessarily
refer to a qualification in addition to that specified by section 30 of the
Corporation Law, which provides that "every director must own in his right
at least one share of the capital stock of the stock corporation of which he is
a director." Any person "who buys stock in a corporation does so with the
knowledge that its affairs are dominated by a majority of the stockholders
and that he impliedly contracts that the will of the majority shall govern in
all matters within the limits of the act of incorporation and lawfully enacted
by-laws and not forbidden by law." To this extent, therefore, the
stockholder may be considered to have "parted with his personal right or
privilege to regulate the disposition of his property which he has invested in
the capital stock of the corporation, and surrendered it to the will of the
majority of his fellow incorporators. It cannot therefore be justly said that
the contract, express or implied, between the corporation and the
stockholders is infringed by any act of the former which is authorized by a
majority." Pursuant to section 18 of the Corporation Law, any corporation
may amend its articles of incorporation by a vote or written assent of the
stockholders representing at least two-thirds of the subscribed capital stock
of the corporation. If the amendment changes, diminishes or restricts the
rights of the existing shareholders, then the dissenting minority has only
one right, viz.: "to object thereto in writing and demand payment for his
share." Under section 22 of the same law, the owners of the majority of the
subscribed capital stock may amend or repeal any by-law or adopt new by-
laws. It cannot be said, therefore, that Gokongwei has a vested right to be
elected director, in the face of the fact that the law at the time such right as
stockholder was acquired contained the prescription that the corporate
charter and the by-law shall be subject to amendment, alteration and
modification.

Note:
A corporation is authorized to prescribe qualifications of its directors; such
is not invalid, provided, however that before such nominee is disqualified,
he should be given due process to show that he is not covered by such
disqualification. A director stands in fiduciary relation to the corporation
and its stockholders. The disqualification of a competition from being
elected to the board of directors is a reasonable exercise of corporate
authority. Sound principles of corporate management counsel against
28 | P a g e

sharing sensitive information with a director whose fiduciary duty to loyalty
may require that he discloses this information to a competitive rival.


Sales v. SEC
Facts:
State Investment House, Inc. (formerly State Financing Center, Inc.) entered
into a sales agreement with Sipalay Mining whereby the latter sold to the
former 200,000,000 common shares of its capital stock in the amount of
P2,600,000.00. The sales agreement between Sipalay Mining and State
Investment contained the following terms and conditions:
2. That the stockbroker shall not sell more than 1,000,000 shares
per buyer, to the extent practicable;
3. In the event you decide to make a public offering [of] additional
shares in the future, whether with Sipalay Mining and Exploration
Corporation or any other corporation organized by Sipalay Mining
Exploration Corporation, you hereby grant us a right of first refusal
to undertake the same;
4. The Corporation shall as soon as practicable after the offering
period of our shares, apply for listing in the Stock Exchange in
accordance with the rules and regulations of the Securities and
Exchange Commission. The timing of the date of listing shall be
mutually decided by us.
5. That State Financing Center, Inc. shall issue a voting trust in favor
of the Board of Directors of Sipalay Mining Exploration Corporation
which shall only be good up to the time the sale to the public of said
shares has been effected.
The 200,000,000 shares of stock of Sipalay Mining, covered by ten
certificates of stock, were delivered to State Investment. Subsequently, the
restriction on the sale of the shares was modified by allowing sale in blocks
of 5,000,000 shares per buyer.
On December 1975, State Investment requested Sipalay Mining to transfer
the 200,000,000 shares to Anselmo Trinidad & Co., Inc. (hereinafter referred
to as ATCO), to which it had sold the shares. Sipalay Mining complied with
this request. During the time that ATCO held the shares, it voted them in the
stockholders' meetings of Sipalay Mining.
Some two and a half years later, ATCO in turn sold 198,500,000 of the
shares to respondent VULCAN. Sipalay Mining was requested by ATCO to
transfer the 198,500,000 shares to the name of VULCAN.
Eight days prior to the scheduled annual stockholders' meeting of Sipalay
Mining, petitioners filed before the SEC a petition to nullify the sale of the
shares to VULCAN, with a prayer for the issuance of a writ of preliminary
injunction to enjoin VULCAN from voting the shares.

Issue:
Whether the transferee of the shares can be deprived of his right to vote
due to the fact that the transferor has violated a condition in the sales
agreement it entered when it acquired the subject shares?

Held:
NO, THE BUYERS RIGHT TO VOTE THE SHARE IS A RIGHT INHERENT TO
OWNERSHIP. Where the corporation sold its share to an investment house
on the condition that the same shall be sold to the public through
stockbrokers in block of 1 million shares per buyer and the condition was
not fulfilled, the sale is nevertheless valid unless set aside by a competent
court. Thus, the buyer, as a stockholder, cannot be deprived of his right to
vote his shares as it is a right inherent to ownership. The stockholder may
be deprived of the right to vote only upon clear showing of its lawful denial
under the articles of incorporation or by- laws of the corporation.
Further issues relating to the directive of the board of directors of the
issuing corporation to issue a stock certificate in favor of the buyer are
questions of policy or management and are left solely to the honest decision
of officers and directors of a corporation, and so long as they act in good
faith, their orders are not reviewable by the courts.



Pena v. Court of Appeals
Facts:
PAMBUSCO mortgaged the subject properties to the DBP. For failure to pay
its obligation, the mortgage was foreclosed and was sold to Pena.
The Board of Directors of PAMBUSCO, through three out of its five directors,
resolved to assign its right of redemption over the said properties and
authorized one of its members, Atty. Briones to execute and sign a Deed of
29 | P a g e

Assignment for and in behalf of PAMBUSCO. The remaining asset of the
corporation was the right to redeem the foreclosed parcels of land.
Under the by laws of PAMBUSCO, to constitute a quorum in a special
meeting of the Board, at least four members must be present.
Consequently, Briones executed executed a Deed of Assignment in favor of
Enriquez. The latter redeemed the properties and sold the same in favor of
Spouses Yap.
The Yaps wrote Pena asking for the payment of the back rentals.
Despite the foregoing, Pena remained in possession of the lots prompting
the Yaps to file an action in court.

Issue:
Whether or not the board resolution authorizing the assignment of the right
to redeem the properties is valid

Held:
NO. three out of five directors of the board of directors present in a special
meeting do not constitute a quorum to validly transact business when its
by- kaws require at least four members to constitute a quorum. Under
Section 25 of the Corporation Code, the articles of incorporation or by-laws
may fix a greater number than the majority of the number of directors to
constitute a quorum. Any number less than the number provided in the
articles or by- laws cannot constitute a quorum; any act therein would not
bind the corporation; all the attending directors could do is to adjourn.
Further, where the remaining asset of a corporation was its right to redeem
the parcels of land that were foreclosed, the assignment of the right to
redeem requires in addition to a proper board resolution, the affirmative
votes of the stockholders representing at least 2/3 of the outstanding
capital stock. There having been no stockholders approval the redemption
made by the assignee is invalid.



Visayan v. NLRC
Facts:
Upon organization of private respondent Fujiyama Hotel & Restaurant, Inc.,
in which Rivera holds majority interest; it immediately opened a Japanese
establishment, known as Fujiyama Hotel & Restaurant. In order to fully offer
an authentic Japanese cuisine and traditional Japanese style of service,
private respondent hired the services of Isamu Akasako as its chef and
restaurant supervisor.
In June, 1980, Lourdes Jureidini and Milagros Tsuchiya, allegedly pretending
to be stockholders of the corporation, filed a case against Rivera and
Akasako to wrest control over the establishment.
The court issued a writ of preliminary mandatory injunction transferring
possession of all the assets of the company and the management thereof to
Jureidini and Tsuchiya.
Upon assuming management, Jureidini and Tsuchiya replaced almost all of
the existing employees with new ones, majority of whom are the present
petitioners in the instant case.
In the meantime, Rivera and the rest of the stockholders elevated the civil
case to the Supreme Court through a petition for certiorari assailing the
ground for the issuance of the writ of preliminary mandatory injunction by
the lower court.
The SC issued a writ of preliminary injunction to enjoin enforcement of the
writ of preliminary mandatory injunction issued by the lower court.
Pursuant to the above-quoted resolution, Rivera and Akasako regained
control and management of Fujiyama Hotel & Restaurant, Inc. Immediately
upon assumption of the management of the corporation, Rivera et al.,
refused to recognize as employees of the corporation all persons that were
hired by Jureidini and Tsuchiya during the one-year period that the latter
had operated the company and reinstated the employees previously hired
by them. This gave rise to the filing of the present case by the dismissed
employees hired by Jureidini and Tsuchiya (some of whom had allegedly
been hired by Rivera and Akasako even before Jureidini and Tsuchiya
assumed management of the corporation) against Fujiyama Hotel &
Restaurant, Inc. for illegal dismissal.

Issue:
whether or not there is privity of contract between petitioners and private
respondent as to establish an employer-employee relationship between the
parties.
Held:
NO PRIVITY OF CONTRACT, THE HIRING OF EMPLOYEES BY THOSE WHO
TOOK OVER THE COMPANY DOES NOT BIND THE LATTER.
30 | P a g e

Where two parties forcibly took over the management of the corporation by
virtue of a writ of preliminary injunction issued on the basis of their claim
that they were stockholders, their hiring of new employees to replace the
original employees cannot bint the corporation, as the act was done
without authority from the board. The claim of illegal dismissal by the new
employees is without merit as they were not hired by the corporation or its
duly authorized officers of agents.


Lee v. Court of Appeals
Facts
On November 1985, filed a complaint for a sum of money against Sacoba
Manufacturing Corp who, in turn, filed a third party complaint against ALFA
and the petitioners.
IN the course of the proceedings, Lee and Lacdao informed the court that by
virtue of the voting trust agreement they ceased to be officers and directors
of ALFA, hence, they could no longer receive summons or any court
processes for or on behalf of ALFA.; that management of ALFA had been
transferred to DBP.
The DBP, on the other hand, claimed that it was not authorized to receive
summons on behalf of ALFA since the DBP had not taken over the company
which has a separate and distinct corporate personality and existence.
The trial court issued an order advising Sacoba Manufacturing, et. al. to take
the appropriate steps to serve the summons to ALFA. Sacoba
Manufacturing, et. al. filed a Manifestation and Motion for the Declaration
of Proper Service of Summons which the trial court granted.

Issue:
whether or not there was proper service of summons ALFA through the
petitioners as president and vice-president, allegedly, of the subject
corporation after the execution of a voting trust agreement between ALFA
and DBP.

Held:
NO. Any director who ceases to be the owner of at least 1 share of the
capital stock of the corporation of which he is a director shall thereby cease
to be a director. Since a director who executes a VTA over all his shares
ceases to be a stockholder of record in the books of the corporation and
cease to be a director, he cannot be served with summons intended for the
corporation.


Citibank N.A. v. Chua
Facts:
Citibank is a foreign commercial banking corporation duly licensed to do
business in the Philippines. Spouses Cresencio and Zenaida Velez, were
good clients of the bank.
They filed a complaint for specific performance and damages against the
bank.
On the date of the pre-trial conference, counsel for petitioner bank
appeared, presented a special power of attorney executed by Citibank
officer Florencia Tarriela in favor of petitioner bank's counsel, the J.P. Garcia
& Associates, authorizing it to represent and bind petitioner bank at the
pre-trial conference of the case.
The by-laws of the bank grant to its Executing Officer and Secretary Pro-Tem
the power to delegate to a Citibank officer, in this case Ferguson, the
authority to represent and defend the bank and its interests.
In spite of this SPA, counsel for private respondents orally moved to declare
petitioner bank in default on the ground that the SPA was not executed by
the Board of Directors of Citibank. Petitioner bank was then required to file
a written opposition to this oral motion to declare it in default. In said
opposition petitioner bank attached another SPA made by William W.
Ferguson, Vice President and highest ranking officer of Citibank, Philippines,
constituting and appointing the J.P. Garcia & Associates to represent and
bind the BANK at the pre-trial conference and/or trial of the case.

Issue:
Whether a board resolution is necessary for its legal counsel or Citibank
employees to act as its attorney-in-fact in the case because petitioner
bank's by-laws grant to its Executing Officer and Secretary Pro-Tem the
power to delegate to a Citibank officer, in this case William W. Ferguson,
the authority to represent and defend the bank and its interests.

Held:
NO. In the corporate hierarchy, there are three levels of control:
31 | P a g e

1 the board of directors, which is responsible for corporate policies and the
general management of the business affairs of the corporation;
2 the officers, who in theory execute the policies laid down by the board,
but in practice often have wide latitude in determining the course of
business operations; and
3 the stockholders who have the residual power over fundamental
corporate changes, like amendments of the articles of incorporation.

However, just as a natural person may authorize another to do certain acts
in his behalf, so may the board of directors of a corporation validly delegate
some of its functions to individual officers or agents appointed by it. It is
clear that corporate powers may be directly conferred upon corporate
officers or agents by statute, the articles of incorporation, the by-laws or by
resolution or other act of the board of directors. Since the by-laws are a
source of authority for corporate officers and agents of the corporation, a
resolution of the Board of Directors of Citibank appointing an attorney in
fact to represent and bind it during the pre-trial conference of the case at
bar is not necessary because its by-laws allow its officers, the Executing
Officer and the Secretary Pro-Tem, to execute a power of attorney to a
designated bank officer, William W. Ferguson in this case, clothing him with
authority to direct and manage corporate affairs.


Grace Christian Highschool v. Court of Appeals
Facts:
Grace Christian High School is an educational. Grace Village Association,
Inc., on the other hand, is an organization of lot and/or building owners,
lessees and residents at Grace Village.
On December 1975, a committee of the board of directors prepared a draft
of an amendment to the by-laws, providing that GRACE CHRISTIAN HIGH
SCHOOL representative is a permanent Director of the ASSOCIATION." This
draft was never presented to the general membership for approval. For 15
years, Grace Christian High School was given a permanent seat in the board
of directors of the association.
After some time, the association's committee on election informed James
Tan, principal of the school, that "it was the sentiment that all directors
should be elected by members of the association." For this reason, Tan was
told that "the proposal to make the Grace Christian High School
representative as a permanent director of the association, although
previously tolerated in the past elections should be re-examined."



Issue:
Whether or not provision in the by- laws allowing a director to hold the
position perpetually is valid.

Held:
NO. The BOD of Corporations must be elected from among the stockholders
or members. Since the provision is contrary to law, the fact that for 15 years
it has not been questioned cannot forestall a later challenge to its validity.


Naguiat v. NLRC
Facts:
CFTI held a concessionaires contract with the Army Air Force Exchange
Services (AAFES) for the operation of taxi services within Clark Air Base.
Sergio Naguiat was CFTIs president while Antolin Naguiat was its VP. The
Naguiats were actively engaged in the management of CFTI.
Clark Air Base was not spared from the phase out of the US Military bases in
the country, as a result, the AAFES was dissolved and employees were
terminated.
The AAFES Taxi Drivers Association (UNION for brevity) and CFTI negotiated
as regards separation benefits for the displaced employees. An agreement
was reached, however some of the employees refused to accept the same,
they disaffiliated from the Union and joined the National Organization of
Workingmen (NOWM).
NOWM filed a complaint against AAFES, Sergio, Antolin and Sergio F.
Naguiat Enterprises.
The enterprise was impleaded on the ground that both CFTI and Naguiat
Enterprises were "close family corporations" owned by the Naguiat family

Issue:
Whether the corporate officers of CFTI can be held solidarily liable for
corporate debts

32 | P a g e

Held:
YES. Stockholders who are actively engaged in the management or
operation of the business and affairs of a close corporation shall be
personally liable for the corporate torts unless the corporation has obtained
reasonably adequate liability insurance coverage.

Benguet Electric Cooperative, Inc. v. NLRC
Facts:
Cosalan was the General manager of Benguet Electric Cooperative, Inc.
(BENECO).
BENECO, through Casalan received COA Audit Report stating the financial
status and irregularities in the utilization of funds released by the National
Electrification Administration (NEA) to BENECO.
Heving been made aware of the serious financial condition of BENECO,
Casalan implemented remedial measures.
The members of the Board reacted by adopting a series of resolutions. The
resolutions adopted abolished the housing allowance of Casalan and
reduced his salary, among others.
Another resolution was adopted, removing Casalan as General Manager.
Despite such, he continued to work as a GM in the belief that he could be
suspended or removed only by duly authorized officials of NEA.
Casalan requested BENECO to release his salary, the latter denied the
request. This prompted Casalan to challenge the validity of the board
resolutions with the NLlRC.
LA held both BENECO and its board members to be jointly and severally
liable.

Issue:
Whether the members of BOD are liable

Held:
YES.
GR: the board members of the corporation who in good faith purport to act
for and in behalf of the corporation within the lawful scope of their
authority do not become liable for the consequences of their acts.
EXP: when the directors abolished the benefits and eventually dismissed in
bad faith and without procedural due process the GM who tried to
implement remedial measures to solve the serious financial condition of the
corporation.


Metropolitan bank and Trust Company v. Quilts & All Inc.
Facts:
de los Santos the then Corporate Secretary of Quilts issued a Secretary's
Certificate which certified that in a special meeting of the Board of Directors
its President, Dizon was authorized and empowered to mortgage in favor of
Metrobank, a property belonging to Quilts. On the basis of the Secretary's
Certificate, Metrobank restructured Dizon's existing personal loan, which
was secured by a mortgaged over the personal property of Dizon and of the
corporation.
More than a year later, Metrobank received a letter from Atty. Villanueva,
Quilt's counsel offering an amount for the cancellation of the mortgage on
the property owned by Quilts because, allegedly, "Mr. & Mrs. Senen Dizon
had left the Philippines, leaving several creditors." Metrobank refused the
offer since the amount offered did not approximate the appraised value of
the mortgaged property.
Atty. Trinidad, Quilt's new counsel wrote Metrobank reiterating the
mortgage cancellation. In addition, counsel claimed that the alleged April 7,
1987 special meeting could not have taken place for lack of the requisite
number of directors present to constitute a quorum since the Chairman and
2 other members of the Board of Directors were aboard on that date.

Issue:
Whether or not the loan may be annulled

Held:
NO. A mortgage on corporate property accepted by a bank as basis for
restructuring a personal loan cannot be annulled even though it could not
have been authorized by the board of directors for lack of quorum where
the bank relied on the secretarys certificate attesting to the existence of a
board resolution approving the mortgage.


Lopez Realty Inc. v. Fontecha
Facts:
33 | P a g e

Lopez Realty, Inc., is a corporation engaged in real estate business,
petitioner Gonzales is one of its majority shareholders.
Sometime in 1978, Lopez submitted a proposal relative to the the reduction
of employees with provision for their gratuity pay. The proposal was
deliberated upon and approved in a special meeting of the board of
directors. It appears that petitioner corporation approved two (2)
resolutions providing for the gratuity pay of its employees.
Private respondents were the retained employees of the Corporation. In a
letter, the private respondents requested for the full payment of their
gratuity pay. Their request was granted in a special meeting held. At that,
time, however, Gonzales was still abroad. Allegedly, while she was still out
of the country, she sent a cablegram to the corporation, objecting to certain
matters taken up by the board in her absence, such as the sale of some of
the assets of the corporation. Upon her return, she filed a derivative suit
with the SEC against majority shareholder Lopez.
Notwithstanding the "corporate squabble" between Gonzales and Lopez,
the first two (2) installments of the gratuity pay of the private respondents
were paid by the corporation. Also, the corporation had prepared the cash
vouchers and checks for the third installments of gratuity pay of said private
respondents.
For some reason, said vouchers were cancelled by Gonzales. Likewise, the
first, second and third installments of gratuity pay of the rest of private
respondents were prepared but cancelled by Gonzales. Despite private
respondents' repeated demands for their gratuity pay, corporation refused
to pay the same.

Issue:
Whether the corporation is bound to grant its employees gratuity pay
despite the lack of notice to a board director during the meeting wherein
the said resolution was passed

Held:
YES. As a general rule, a corporation through its board of directors should
act in the manner and within the formalities prescribed by its charter or by
the general law. Thus, directors must act as a body in a meeting called
pursuant to the law or corporations by- laws, otherwise any action may be
questioned by any objecting stockholder. However, an action of the board
of directors during a meeting, which was illegal for lack of notice may be
ratified either expressly, by the action of the directors in subsequent legal
meeting or impliedly by the corporations subsequent course of conduct.
Thus, a director who was not notified of a board meeting is precluded from
questioning the validity of the resolution granting gratuity pay to employee
approved at that meeting if she later on acquiesced to it by signing the
vouchers for the payment of the gratuity pay.


Premium Marble Resources Inc. v. Court of Appeals
Facts:
Premium Marble Resources, Inc. (PREMIUM for brevity), assisted by Atty.
Dumadag as counsel, filed an action for damages against International
Corporate Bank (ICB).
Meantime, the same corporation, i.e., PREMIUM, but this time represented
by Siguion Reyna, Montecillio and Ongsiako Law Office as counsel, filed a
motion to dismiss (MTD) on the ground that the filing of the case was
without authority from its duly constituted board of directors as shown by
the excerpt of the minutes of PREMIUMs board of directors meeting. In its
opposition to the MTD, PREMIUM thru Atty. Dumadag contended that the
persons who signed the board resolution are not directors of the
corporation and were allegedly former officers and stockholders of
PREMIUM who were dismissed for various irregularities and fraudulent acts;
On the other hand, Siguion Reyna Law firm as counsel of PREMIUM in a
rejoinder, asserted that it is the general information sheet filed with the
SEC, among others, that is the best evidence that would show who are the
stockholders of a corporation and not the AOI since the latter does not keep
track of the many changes that take place after new stockholders subscribe
to corporate shares of stocks.

Issue:
Whether the filing of the action was with authorization from the BOD

Held:
NO. By express mandate of the Corporation Code, all corporations duly
organized pursuant thereof are required to file with SEC the names,
nationalities and residence of the directors and officers elected. In
determining whether the filing of an action was authorized by the board, it
34 | P a g e

is the list of directors in the latest general information sheet as siled with
the SEC which is controlling.


Esguerra v. Court of Appeals
Facts:
Julieta Esguerra filed a complaint for administration of conjugal partnership
or separation of property against her husband Vicente Esguerra, Jr. The said
complaint was later amended impleading V. Esguerra Construction Co., Inc.
(VECCI for brevity) and other family corporations as defendants.
The parties entered into a compromise agreement which was submitted to
the court. On the basis of the said agreement, the court rendered two
partial judgments: one between Vicente and Esguerra; and the other as
between the latter and VECCI. By virtue of said agreement, Esguerra Bldg. I
was sold and the net proceeds distributed according to the agreement.
The controversy arose with respect to Esguerra Building II. Esguerra started
claiming one-half of the rentals of the said building which VECCI refused.
Esguerra filed a motion with respondent court praying that VECCI be
ordered to remit one-half of the rentals to her until the same be sold.
VECCI opposed said motion. Meanwhile, Esguerra Bldg. II was sold to
Sureste Properties, Inc. Sureste relied on the certification given by the
corporate secretary that such sale was authorized by resolution of the biard
Esguerra sought to nullify the sale on the ground that VECCI is not the lawful
and absolute owner thereof and that she has not been notified nor
consulted as to the terms and conditions of the sale.

Issue:
Whether or not the sale may be invalidated

Held:
NO. A stockholder cannot invalidate the sale of corporate properties for
failure to comply with Section 40 of the Corporation Code, where the buyer
relied on the secretarys certificate that the sale had been authorized by
resolution of the board of directors and of the stockholders. Being regular
on its face, a Secretarys Certificate is sufficient for a third party to rely on.
It does not have to investigate the truth of the facts contained in such
certification, otherwise business transaction of corporations would become
tortuously slow and unnecessarily hampered.


Traders Royal Bank v. Court of Appeals
Facts:
Filriters Guaranty Assurance Corporation (FILRITERS) is the registered owner
of CBCI No. D891. Under a deed of assignment, FILRITERS transferred CBCI
No. D891 to Philippine Underwriters Finance Corporation (PHILFINANCE).
Subsequently, PHILFINANCE transferred CBCI No. D891, which was still
registered in the name of FILRITERS, to Traders Royal Bank (TRB).
The transfer was made under a repurchase agreement granting
PHILFINANCE the right to repurchase the instrument on or before April 27,
1981. When PHILFINANCE failed to buy back the note on maturity date, it
executed a deed of assignment, dated April 27, 1981, conveying to TRB all
its rights and title to CBCI No. D891.
Armed with the deed of assignment, TRB then sought the transfer and
registration of CBCI No. D891 in its name before the Security and Servicing
Department of the Central Bank (CB). Central Bank, however, refused to
effect the transfer and registration in view of an adverse claim filed by
FILRITERS.

Issue:
Whether or not the deed of assignment is valid

Held:
NO. The assignment made without authorization of the board of directors
does not bind the corporation


Western Institute Technology, Inc. v. Salas
Facts:
The members of the Salas family are the majority and controlling members
of the Board of Trustees of Western Institute of Technology, Inc. (WIT, for
short).
The minority stockholders of WIT held a Special Board meeting in the
principal office of WIT. In attendance were other members of the Board.
Prior to the Special Board Meeting, copies of notice thereof were
distributed to all Board Members. In said meeting, the Board of Trustees
35 | P a g e

passed Resolution No. 48, granting monthly compensation to the private
respondents as corporate officers.
A few years later, two (2) separate criminal informations were filed, one for
falsification of a public document and the other for estafa were filed.
The charge for falsification was anchored on the ground that Resolution No.
4 was passed on a date not covered by the WIT income statement passed
with the SEC.
The private respondents were acquitted of the complaints.
Petitioners however contend that private respondents should be held civilly
liable despite their acquittal. They base their claim on the alleged illegal
issuance by private respondents of Resolution No. 48, ordering the
disbursement of corporate funds in favor of private respondents, board
members of WIT. Petitioners maintain that this grant of compensation to
private respondents is proscribed under Section 30 of the Corporation Code.
Thus, private respondents are obliged to return these amounts to the
corporation with interest.

Issue:
Whether or not the grant of additional compensation to WITs Board of
Directors as corporate officers was valid
Held:
YES. In the case at bench, Resolution No. 48, s. 1986 granted monthly
compensation to private respondents not in their capacity as members of
the board, but rather as officers of the corporation, more particularly as
Chairman, Vice-Chairman, Treasurer and Secretary of Western Institute of
Technology.

Note:
Members of the board of directors may receive compensation, in addition
to reasonable per diems, in the following cases:
When there is a provision in the by- laws fixing their compensation;
When the stockholders representing at least majority of the outstanding
capital stock at a regular or special stockholders meeting agree to give it to
them; and
When they render services to the corporation in any capacity other than as
a director


Bitong v. Court of Appeals
Facts:
Ex Libris Publishing Co., Inc was incorporated by Senator Enrile, Cristina
Ponce Enrile through Jaka Investments Corporation (JAKA) and the Apostols.
When Ex Libris suffered financial difficulties, JAKA and the Apostols together
with the new investors restructured Ex Libris by organizing a new
corporation known as Mr. & Mrs.
Bitong as treasurer and member of the Board of Mr & Ms company
complained of the irregularities committed by Apostol, the President and
Chairperson of the Board.
She further claimed that except for the sale of the name Philippine Inquirer
to Philippine Daily Inquirer (PDI hereafter) all other transactions and
agreements entered into by Mr. & Ms. with PDI were not supported by any
bond and/or stockholders resolution. That upon instructions of Eugenia D.
Apostol, Mr. & Ms. made several cash advances to PDI on various occasions
which was not supported by any board or stockholders resolution, contract
nor any other document.
Bitong alleged stocks were sold and endorsed in his favour by Senator
Enrile, by virtue of which he has the right to bring such action in behalf of
the corporation; records however showed that the subject stocks were
issued in favour of Apostol by virtue of Declaration of trust and Deed of Sale
executed by JAKA.

Issue:
1. Whether Bitong has the capacity to initiate a derivative suit in
behalf of the corporation to assail the supposed fraudulent acts of
the private respondents
2. Whether or not Bitong is entitled to dividends declared by the
corporation

Held:
1. NO. The power to sue and be sued in any court by a corporation is
lodged in the board of directors that exercises its corporate powers and
not in the president or any officer thereof. A derivative suit should not
prosper if it is filed by a person who is not authorized by the corporate
share for whose benefit the shares are held.
2. NO. The alleged endorsement of the Certificate of Stock in the name of
JAKA nor the deed of sale executed by Sen. Enrile directly in favor of
36 | P a g e

petitioner could not have legally transferred or assigned the shares of
stock in favor of petitioner because said Certificate of Stock in the name
of JAKA was already cancelled and a new one was issued in favor of
respondent Apostol by virtue of a Declaration of Trust and Deed of Sale.
Petitioner was well aware of this trust, being the person in charge of
this documentation and being one of the witnesses to the execution of
this document. Hence, petitioner is not entitled to dividends.
When a dividend is declared, it belongs to the person who is the
substantial and beneficial owner of the stock at the time regardless of
when the distribution profit was earned.


Tan Wing Talk v. Makasiar
Facts:
Petitioner as director of Concord-World Properties, Inc., (CONCORD for
brevity), filed an action for violation of BP 22 with the City Prosecutor
against Vic Ang Siong .
The complaint alleged that the check issued by Vic Ang Siong in favor of
Concord was dishonored when presented for encashment.
Vic Ang Siong sought the dismissal of the case on two grounds: First, that
petitioner had no authority to file the case on behalf of CONCORD, the
payee of the dishonored check, since the firms board of directors had not
empowered him to act on its behalf. Second, he and CONCORD had already
agreed to amicably settle the issue after he made a partial payment of on
the dishonored check.
On March 23, 1994, the City Prosecutor dismissed the complaint on the
following grounds: (1) that petitioner lacked the requisite authority to
initiate the criminal complaint for and on Concords behalf; and (2) that
CONCORD and Vic Ang Siong had already agreed upon the payment of the
latters balance on the dishonored check.

Issue:
Whether Tam Wing Tak has the authority to sue in behalf of the corporation

Held:
NO. Under Section 36 of the Corporation Code, read in relation of Section
23, it is clear that where the corporation is an injured party, its power to sue
is lodges with its board of directors or trustees. In the absence of proof that
he was authorized or granted specific powers by the board of directors, a
minority stockholder and member of the BOD had no such power or
authority to sue on behalf of the corporation.


DBP v. Court of Appeals
Facts:
Marinduque Mining Corporation (MMC) purchased construction materials
from Remington Industrial Sales Corporation (RISC).
MMC failed to pay, this prompted RISC to file an action for collection with
the court.
In the meantime, MMC obtained various loan accommodations from the
PNB, as a security, the former (MMC) executed REMs and CMs over its
properties in favour of the bank.
The Mortgage Agreement was amended it was agreed that Marinduque
mortgaged in favour of PNB all other real and personal properties and other
real rights the latter may subsequently acquire.
For failure to settle the account, the bank extrajudicially foreclosed the
mortgaged property. PNB and DBP emerged as the highest bidders.
PNB and DBP assigned and conveyed all their rights, interests and
participation over the foreclosed properties in favour of Nonoc Mining,
Mariculum Mining, Island Cement and APT.
Sometime in 1987, PNB and DBP pursuant to Resolution No. 50 assigned,
transferred and conveyed to the National Government thru the Asset
Privatization Trust (APT) all its rights and interest over MMC which were
earlier assigned to other corporations.
Remington impleaded as co defendants PNB, DBP and other assignee
corporations on the ground that all of them must be treated in law as one
and the same entity by disregarding the veil of corporate fiction since all of
these corporations share the same or almost the same set of directors.

Issue:
Whether Remington, a third person may raise the issue on interlocking
directors

Held:
NO. The rule pertaining to transactions between corporation with
interlocking directors resulting in prejudice to one of the corporation does
37 | P a g e

not apply where the corporation allegedly prejudiced is a third party, not
one of the corporations with interlocking directors. Thus, when a mortgage
bank foreclosed the mortgage on the real and personal property of the
debtor and thereafter assigned and the properties to a corporation it
formed to manage and thereafter assigned the properties to a corporation
it formed to manage the foreclosed assets, the unpaid seller of the debtor
can not complain that the assignment is invalid simply because the
morgagee and the assignee have interlocking directors.


A.F. Realty & Devt Inc. v. Dieselman Freight Services
Facts:
Manuel Cruz member of BOD of Dieselman Corporation authorized Polintan,
a real estate broker of the CNP Real Estate Brokerage to look for a buyer of
a parcel of land registered under the name of the corporation. Cruz,
however had no authority from the corporation to sell the said property.
In turn, Polintan authorized Noble to sell the same lot.
Noble then offered the property to A.F. Realty.
A.F. through Ranullo, a board member and the vice president accepted the
offer and issued a check payable to the order of Dieselman. Ranullo then
asked Polintan for the board resolution of Dieselman authorizing the sale of
said lot, which the latter failed to produce; what was given was the TCT, tax
declaration and other documents.
A higher price was demanded from A.F. Realty; however it reiterated that it
is willing to pay only the balance. Because of this, Cruz terminated the offer
and demanded the return of the title and other documents.
A.F Realty then filed a complaint for specific performance against Dieselman
and Cruz.
Meanwhile, Dieselman and Midas Development Corporation executed a
Deed of Absolute Sale over the same property.
The lower court held that the acts of Cruz bound the corporation.
CA reversed.

Issue:
Whether the alleged sale of land to A.F. Realty bound the corporation

Held:
NO. Contracts or acts of a corporation must be made either by the BOD or
by a corporate agent duly authorized by the board. Absent such valid
delegation/ authorization, the rule is that the declaration of an individual
director relating to the affairs of the corporation, but not in the course of, or
connected with the performance of authorized duties of such director, is
held not binding on the corporation.
Thus, where the director was not authorized by the board to sell corporate
property, its sale is not binding on the corporation. The sale cannot be
ratified despite acceptance by the corporation of the partial payment if
what is involved is sale of land. Considering that the officers who
represented and acted as agents in behalf of the corporation were not
authorized, the contract of sale is null and void under Article 1874 of the
Civil Code. Being a void contract, it is not susceptible of ratification.


Filipinas Port Services, Inc. v. Go
Facts:
Cruz, FILPORTs president wrote a letter to the BOD questioning the boards
creation of certain positions with a monthly remuneration and the election
thereto of certain members of the board,
Cruz requested the board to take necessary actions to recover from those
elected the salaries they have received.
However, whatever action the board took did not sit well with Cruz.
Cruz as a response filed a derivative suit with the SEC for alleged
mismanagement of the corporation.
The derivative suit however hibernated with the SEC and was turned over to
the RTC which ruled that the creation of the questioned positions were
unnecessary.
CA reversed.

Issue:
Whether or not the creation of additional offices/ positions by the board
may be questioned in court through the filing of a derivative suit

Held:
NO AS LONG AS THE BOD ACTS IN GOOD FAITH AND IN THE EXERCISE OF
HONEST JUDGMENT IN THE INTEREST OF THE CORPORATION.
38 | P a g e

The determination of the necessity for additional offices and/ or positions in
a corporation, if authorized under the by- laws, is a management
prerogative which the courts will not review in the absence of any proof
that such prerogative was exercised in bad faith or with malice.
Questions of policy or of management are left solely to the honest decision
of the board as the business manager of the corporation, and the court is
without authority to substitute its judgment for that of the board, and as
long as it acts in good faith and in the exercise of honest judgment in the
interest of the corporation, its orders are not reviewable by the courts.


Associated Bank v. Spouses Pronstroller
Facts:
Spouses Vaca failed to pay their obligation with Association Bank which led
to the foreclosure and sale of their property with the bank as the highest
bidder. The Vacas filed an action to annul the foreclosure as well as the sale.
During the pendency of the case, the bank offered the property for sale,
Spouses Pronstroller offered to purchase the same; the offer was made
through Atty. Saluta, the banks VP, Corporate Secretary, Board member
and in- house counsel. The offer was accepted and negotiations between
the parties ensued which resulted to a letter- agreement between them.
The parties agreed that deposit must be made within ninety days from the
signing of the agreement.
On July 1993 or prior to the expiration of the 90-day, in view of the
pendency of the case between the spouses Vaca and the bank involving the
subject property, Spouses Pronstroller requested that the balance of the
purchase price be made payable only upon service on them of a final
decision or resolution of this Court affirming petitioner's right to possess the
subject property. Atty. Saluta agreed through another letter- agreement
without any board resolution, thus in effect extending the 90 day period.
After some time the bank underwent reorganization and Atty. Saluta was
relieved of his responsibilities, the new management reviewed the records
of the bank and found out that Spouses Pronstroller failed to pay the
balance. The bank through Asset Recovery and Remedial Management
Committee (ARRMC) disapproved the request of the Pronstrollers. The
latter submitted a proposal as to how the balance will be paid but the bank
disapproved.
For failure of the parties to reach an agreement, the Pronstrollers informed
the bank that they will enforce the July 1993 agreement; the bank
countered that Atty. Saluta was not authorized to sign in behalf of the bank.

Issue:
Whether the Letter Agreement signed by Atty. Saluta is binding on the bank
under the doctrine of apparent authority

Held:
YES. The doctrine of apparent authority had long been recognized in this
jurisdiction. Apparent authority is derived not merely from practice. Its
existence may be ascertained through:
the general manner in which the corporation holds out an officer or agent
as having the power to act in general, with which it clothes him; or
the acquiescence in his acts of a particular nature, with actual or
constructive knowledge thereof, within or beyond the scope of his ordinary
powers.
Accordingly, the authority to act for and to bind a corporation may be
presumed from acts of recognition in other instances, wherein the power
was exercised without any objection from its board or shareholders,
Undoubtedly, the bank had previously allowed its in- house counsel to enter
into the first agreement without a board resolution expressly authorizing
him to sell corporate property; thus it had clothe him with apparent
authority to modify the same via the second letter- agreement. Thus, the
corporation is bound by the acts entered into by its in house counsel even
though he was subsequently relieved of the position.
Naturally, the third person has little or no information as to what occurs in
corporate meetings; and he must necessarily rely upon the external
manifestations of corporate consent. The integrity of commercial
transactions can only be maintained by holding the corporation strictly to
the liability fixed upon it by its agents in accordance with law.




Cebu Country Club, Inc. v. Elizagaque
Facts:
39 | P a g e

Sometime in 1987, San Miguel Corporation, a special company proprietary
member of CCCI, designated respondent Ricardo F. Elizagaque, its Senior
Vice President and Operations Manager for the Visayas and Mindanao, as a
special non-proprietary member. The designation was thereafter approved
by the CCCIs Board of Directors.
Elizagaque filed with CCCI an application for proprietary membership.
After several inquiries as to the status of his application he was informed
that the Board rejected his application.
A significant amendment to CCCIs Amended By-Laws requiring the
unanimous vote of the directors present at a special or regular meeting was
not printed on the application form respondent filled and submitted to
CCCI; considering that such amendment was introduced 20 years before
respondent filed his application. What was printed thereon was the original
provision which was silent on the required number of votes needed for
admission of an applicant as a proprietary member. Thus, respondent was
not informed as to why his application was rejected; he does not even have
knowledge that unanimous vote of the Board members was required.
This prompted Elizagaque to file an action for damages which ruled and its
favor. CA affirmed.

Issue:
whether in disapproving respondents application for proprietary
membership with CCCI, petitioners are liable to respondent for damages,
and if so, whether their liability is joint and several

Held:
YES. In rejecting respondents application for proprietary membership, we
find that petitioners violated the rules governing human relations, the basic
principles to be observed for the rightful relationship between human
beings and for the stability of social order. The trial court and the Court of
Appeals aptly held that petitioners committed fraud and evident bad faith in
disapproving respondents applications.
The CCCI Board of Directors, under its Articles of Incorporation, has the right
to approve or disapprove an application for proprietary membership. But
such right should not be exercised arbitrarily.



People v. Hermenegildo Dumlao and Emilio Lao
Facts:
An information was filed before the Sandiganbayan charging Dumlao and
others with violation of the anti graft and corrupt practices act. The
information alleged that the respondent being then members of the board
of trustees of GSIS entered into a contract of lease purchase with Lao
whereby GSIS agreed to sell to Lao a GSIS property.
Dumlao for his defense presented the Agreement and as well as the
minutes of the meeting. He argued that the allegedly approved Board
Resolution was not in fact approved by the GSIS Board of Trustees, contrary
to the allegations in the information. Since the signatures of Fabian Ver,
Roman Cruz, Aber Canlas and Jacobo Clave did not appear in the minutes of
the meeting held on 23 April 1982, he said it was safe to conclude that these
people did not participate in the alleged approval of the Lease-Purchase
Agreement. This being the case, he maintained that there was no quorum of
the board to approve the supposed resolution authorizing the sale of the
GSIS property. There being no approval by the majority of the Board of
Trustees, there can be no resolution approving the Lease-Purchase
Agreement. The unapproved resolution, he added, proved his innocence.
The Sandiganbayan ruled that the minutes of the meeting showed that the
Board failed to approve the Lease- Purchase agreement because only three
out of the seven members signed the minutes; that it is required that in
order to validly pass a resolution at least a majority of four of the Board of
Trustees must sign and approve the same.

Issue:
whether or not the signatures of the majority of the GSIS board of trustees
are necessary on the minutes of meeting to give force and effect to the
Board Resolution pproving the proposed agreement by and among the GSIS
and Lao.

Held:
NO. In a criminal case involving a lease-purchase agreement allegedly
disadvantageous to the government, the Sandiganbyan erred in concluding
that there was no such agreement into and thus negating criminal liability
since only three members out of seven signed the minutes of the meeting.
The non-signing by the majority of the members of the Board of Trustees of
the said minutes does not necessarily mean that the supposed resolution
40 | P a g e

was not approved by the Board. The signing of the minutes by all the
members of the board is not required. There is no provision in the
Corporation Code of the Philippines that requires that the minutes of the
meeting should be signed by all the members of the board. The proper
custodian of the books, minutes and official records of a corporation is
usually the corporate secretary. Being the custodian of corporate records,
the corporate secretary has the duty to record and prepare the minutes of
the meeting. The Signature of the corporate secretary gives the minutes of
the meeting. The signature of the corporate secretary gives the minutes of
the meeting probative value and credibility. Moreover, the entries
contained in the minutes are prima facie evidence of what actually took
place during the meeting.


Westmont Bank formerly Associated Citizens Bank then United Overseas
Bank, Phils. v. Inland Construction and Development Corp.
Facts:
Inland obtained various loans from Associated Bank now Westmont. REMs
were executed to secure the full payment of the obligation.
Under a Deed of Assignment, Abrantes assumed the obligations of Inland
and Aranda.
The records show that Calo was the one assigned to transact on petitioners
behalf respecting the loan transactions and arrangements of Inland as well
as those of Hanil-Gonzales and Abrantes. Since it conducted business
through Calo, who is an Account Officer, it is presumed that he had
authority to sign for the bank in the Deed of Assignment; that petitioner
sent a reply-letter which indicates that it had full and complete knowledge
of the assumption by Abrantes of Inlands obligation. Thus, the assertion
that the petitioner cannot be faulted for its delay in repudiating the
apparent authority of Calo is similarly flawed, there being no evidence on
record that it had actually repudiated such apparent authority. It should be
noted that it was the bank which pleaded that defense in the first place.
What is extant in the records is a reasonable certainty that the bank had
ratified the Deed of Assignment.

Issue:
Whether the bank ratified the questioned Deed of Assignment

Held:
The general rule remains that, in the absence of authority from the board of
directors, no person, not even its officers, can validly bind a corporation. If a
corporation, however, consciously lets one of its officers, or any other
agent, to act within the scope of an apparent authority, it will be estopped
from denying such officers authority. Where the bank conducted business
through its Account Officer, it is presumed that the latter had authority to
sign for the bank in the Deed of Assignment. In this case, it is incumbent
upon the bank to show that its account officer is not authorized to transact
for the corporation.


CORPORATE OFFICERS
Board of Liquidators v. Kalaw
Facts:
National Coconut Corporations charter was amended granting express
power to buy, sell, barter or export copra, coconut and their by- product.
Under its by- laws the general manager has the duty to perform or execute
on behalf of the corporation upon prior approval of the board all contracts
necessary and essential to the accomplishment of the purposes for which it
was organized.
The general manager and Board chairman, Kalaw, entered into various
contracts for the delivery of copra without prior approval of the
corporations directors but said contracts were known to them.
The contracts, however became unprofitable due to typhoons. Despite
which, the board unanimously approved the same. The corporation
however, was only able to partially perform the contracts.
One of the buyers sued the corporation which culminated in amicable
settlements. The corporation sued Kalaw for the recovery of the amount it
paid in the settlement.

Issue:
Whether or not Kalaw is liable

Held:
NO. A corporate officer, entrusted with the general management and
control of its business, has implied authority to make any contract or do any
41 | P a g e

other act which is necessary or appropriate to the conduct of the ordinary
business of the corporation. The general manager of the corporation who
entered into contracts for sale of corpra products could not be held
personally liable for losses incurred because of said contracts despite the
fact that he did not obtain prior approval of the Board of based on customs
and practices within the company and given the speculative nature of the
contracts which required on the spot decision, the Board had previously
allowed him to enter into similar contracts without prior board approval.


De Tavera v. Philippine Tubercolosis Society Inc.
Facts:
The by- laws of the PTSI provides that the Executive Secretary, Auditor and
all other officers and employees shall hold office at the pleasure of the BOD
unless their term of employment shall have been fixed in their employment
contract.
Dr. Buktaw, the Executive Secretary, applied for leave for 300 working days.
In his place, Nita De Tavera was appointed as Acting Exec. Sec.
When Dr. Buktaw tendered his application for optional retirement, De
Tavera was appointed as the Executive Secretary through a letter which
indicates no fixed term.
Later on, another person was appointed in the same position. De Tavera
filed an action against PTSI alleging that she was summarily removed from
her position without due process.

Issue:
Whether De Tavera was illegally removed

Held:
NO. The employment of a corporate officer, who under the by-laws holds
office at the pleasure of the Board of Directors, may be terminated at any
time once said trust and confidence cease. The ouster of such corporate
officer is not one of removal but expiration of term.


Sia v. People
Facts:
Petitioner Sia was charged with estafa for violating a trust receipt
agreement with Continental Bank. Petitioner contends that having only
acted for and in behalf of the metal company, in dealing with Continental
Bank, as president thereof, he cannot be held liable for the crime charged
and that the violation of a trust receipt does not constitute estafa.
Issue:
Whether or not Sia is liable for estafa

Held:
NO. An officer of a corporation can be held criminally liable for acts or
omissions done in behalf of the corporation only where he is made by
specific provision of law personally answer for his corporate action.
Note:
This case was decided prior to the effectivity of the Trust Receipt Law


AC Ransom Labor Union v. NLRC
Facts:
AC Ransom Labor Union filed a case for unfair labor practice against AC
Ransom. Both the CIR and LA ruled in favor of the Union, holding the
company liable for the payment of backwages and ordering the same to
immediately reinstate the members of the Union.
The NLRC reversed.
Pending resolution, AC Ransom claimed to be suffering from business
reversal. However a new corporation, the Rosario Industrial Corporation
(ROSARIO), was organized using the same equipment and having the same
officers and principal stockholders.
AC Ransom used as a defense the clearance given by SEC to cease to
operate due to financial difficulties in order to lessen the award given by the
court. It also declared that ROSARIO is a distinct and separate corporation,
which was organized long before these instant cases were decided
adversely against RANSOM.
The court held the president of AC Ransom liable to pay the employees.

Issue:
42 | P a g e

1. Whether or not ROSARIO Company should be held liable for the
claims of employees
2. Whether or not the officers and directors of AC Ransom should be
held liable for backwages?

Held:
1. YES. Both corporations were closed corporations owned and managed
by members of the same family. Its organization proved to be a
convenient instrument to avoid payment of backwages and the
reinstatement of the 22 workers. This is another instance where the
fiction of separate and distinct corporate entities should be disregarded.
It is very obvious that the second corporation seeks the protective
shield of a corporate fiction whose veil in the present case could, and
should, be pierced as it was deliberately and maliciously designed to
evade its financial obligation to its employees.... When a notion of legal
entity is used to.defeat public convenience, justify wrong, protect fraud,
or defend crime, the law will regard the corporation as an association or
persons, or, in the case of two corporations, will merge them into one
2. YES. If the records do not clearly identify the officer/s directly
responsible for the payment of monetary benefits to the employees, the
president of the corporation, as the responsible officer of the
corporation may be ordered to respond personally in case of closure of
the corporation.


Prime White Cement Corporation v. Intermediate Appellate Court
Facts:
Alejandro Te, member of the Board and auditor, and Prime White through
its President, Falcon and Trazo, as Chairman of the Board, entered into a
dealership agreement which was to sell and deliver to Te white cement for a
fixed price of P9.70 per bag. The Corporation at the time of the contract was
not even manufacturing white cement.
After some time, was a considerable change in the price of white cement;
however no change was made in the dealership agreement.
Sometime in 1969, Te entered into a written agreement with several
hardware stores for the allocation of cement. The corporation through the
corporate secretary replied favorably and in fact imposed several
conditions. However, the corporation refused to comply with the conditions
imposed by Te one of which was to change the price set for each bag of
cement; as a result, he was forced to cancel the agreement for the supply of
white cement with third parties, which were concluded by virtue of the
dealership agreement.
The trual court ruled in favor of Te. On appeal, the same was affirmed.

Issue:
whether or not the "dealership agreement" referred by the President and
Chairman of the Board of petitioner corporation is a valid and enforceable
contract



Held:
NO. In the instant case respondent Te was not an ordinary stockholder; he
was a member of the Board of Directors and Auditor of the corporation as
well. He was what is often referred to as a "self-dealing" director.
A director of a corporation holds a position of trust and as such, he owes a
duty of loyalty to his corporation.
9
In case his interests conflict with those of
the corporation, he cannot sacrifice the latter to his own advantage and
benefit. As corporate managers, directors are committed to seek the
maximum amount of profits for the corporation. This trust relationship "is
not a matter of statutory or technical law. It springs from the fact that
directors have the control and guidance of corporate affairs and property
and hence of the property interests of the stockholders."
Te acted for his sole benefit to the prejudice of the corporation.

Note:
Under the Corporation Law, which was then in force at the time this case
arose,

as well as under the present Corporation Code, all corporate powers
shall be exercised by the Board of Directors, except as otherwise provided
by law.

Although it cannot completely abdicate its power and responsibility
to act for the juridical entity, the Board may expressly delegate specific
powers to its President or any of its officers. In the absence of such express
delegation, a contract entered into by its President, on behalf of the
corporation, may still bind the corporation if the board should ratify the
same expressly or impliedly. Implied ratification may take various forms
like silence or acquiescence; by acts showing approval or adoption of the
43 | P a g e

contract; or by acceptance and retention of benefits flowing
therefrom.

Furthermore, even in the absence of express or implied
authority by ratification, the President as such may, as a general rule, bind
the corporation by a contract in the ordinary course of business, provided
the same is reasonable under the circumstances.

These rules are basic, but
are all general and thus quite flexible. They apply where the President or
other officer, purportedly acting for the corporation, is dealing with a third
person, i. e., a person outside the corporation.


De Guzman v. NLRC
Facts:
Petitioner was the general manager of the Manila Office of Affiliated
Machineries Agency, Ltd. (AMAL). Respondents were the former employees
of AMAL.
The respondents obtained a favorable judgment against AMAL when the
latter ceased its operation in 1986. De Guzman was impleaded for allegedly
selling part of AMALs assets and applying its proceeds to satisfy his own
claim against the company. He likewise formed a new company which was
engaged in the same line of business with the former clients of AMAL.

Issue:
Whether or not De Guzman should be held liable together with the
corporation for the payment of the monetary claims to the respondents

Held:
YES. By taking undue advantage of his position as general manager of AMAL,
petitioner was able to facilitate the consummation of his acts as he had
access over the company's assets. He was able to appropriate for himself
the assets of AMAL which rendered the satisfaction of respondent
employees claims impossible.


New Durawood v. Court of Appeals
Facts:
New Durawood represented by its Branch Manager, Wilson Gaw, filed a
petition for Judicial Reconstitution of title. Attached to said petition was an
affidavit of loss signed by Bongat one of the stockholders of the
corporation.
New Durawood discovered that original TCT was cancelled and in lieu
thereof a new one was issued in the name of Durawood Construction and
Lumber Supply, Inc.
Petitioner claimed that Gaw had no authority to institute the petition for
reconstitution because the Board Resolution authorizing Gaw was passed
without the required quorum.

Issue:
Whether or not Gaw has the authority to institute the petition for
reconstitution

Held;
NO. There having been no quorum present during the meeting where his
authority was supposed to have been given, the filing of a petition for the
reconstitution of the owners duplicate of a transfer certificate of title by a
branch manager is unauthorized.
The doctrine of apparent authority cannot apply because being a mere
branch manager, he could not be looked as a corporate officer clothed with
the implied or apparent power to file the suit and the unauthorized action
was hidden from it.







Tabang v. NLRC
Facts:
Under the by laws of Pamana Golden Care Medical Center Foundation Inc.
the Board of Trustees is given the power to appoint a Medical Director and
such other officers as it may deem necessary and prescribe their powers
and duties.
Accordingly, Purification Tabang, who was a founding member and member
of the Board of Trustee was appointed as Medical and Hospital Director.
44 | P a g e

Later, Tabang was informed personally by Dr. Naval that in a special meeting
held, the Board of Trustee passed a resolution relieving her of her position
as Medical Director and Hospital Director and appointing Dr. Naval and Dr.
Donasco as her replacements.
Tabang filed a case for illegal dismissa with the LA. The LA as affirmed by the
NLRC dismissed the complaint on the ground that the position of Medical
Director is akin to that of an Executive Position in a corporate ladder
structure, hence an intracorporate controversy not within the jurisdiction of
the LA.

Issue:
Whether the positions of Medical Director and Hospital Director are
considered as corporate offices

Held:
An OFFICE is created by the charter of the corporation and the officer is
elected by the directors or stockholders, on the other hand, an EMPLOYEE
usually occupies no office and is generally employed not by the action of the
directors or stockholders but by the managing officer of the corporation
who also determines its compensation.
Tabang, unlike an ordinary employee was appointed by the Board of
Trustee; he is deemed an officer of the corporation.

Note:
A corporate officers dismissal is always a corporate act or an intracorporate
controversy which is cognizable by the RTC as special commercial court






Reahs Corporation v. NLRC.
Facts:
The complainants in the case were the former employees of Reahs. They
were made to work beyond 8 hours a day without overtime pay and some
were even underpaid.
Sometime in 1986, Ms. Domingo the sole proprietress and operator of
Rainbow Sauna sold her business to Reahs. After the sale, all the assets
were turned over to Reahs.
Reahs did not bother to correct the violations of the Labor Code committed
by the previous owner.
Due to poor business, increase in rental cost among others, the Sauna
suffered losses which led to its closure.
The employees filed a complaint for illegal dismissal and unfair labor
practice which was dismissed by the LA but the claims for separation pay
was upheld and the chairman and officers were held jointly and severally
liable with Reahs.

Issue:
Whether or not the officers of Reahs can be held jointly and severally liable
with the corporation in the payment of monetary claims in the absence of
finding of ULP and illegal dismissal

Held:
YES. The corporate officers who were aware that the corporation was
violationg the provisions of labor standards but did not correct the
violations and closed the corporation without paying the employees their
separation benefits are solidarily liable with the corporation for the claims
of the employees.

Uichico v. NLRC
Facts:
The high ranking officers and directors of CRISPA, INC. passed a resolution
terminating several workers employed fors many years in the latters
garment factory on the ground of serious business losses.
The dismissed employees filed an action for illegal dismissal.
CRISPA INC. failed to prove its financial losses and merely presented a
statement of profit and losses which did not bear the signature of a certified
public accountant nor audited by an independent auditor.
LA dismissed the complaints for illegal dismissal but ordered CRISPA INC
together with its major stockholder Floro as well as other officers to pay
separation pay.

Issue:
45 | P a g e

Whether or not officers are liable together with the corporation

Held:
YES. The signing of the Board Resolution retrenching the employees by the
high ranking officers and directors of CRISPA INC on the feigned ground of
serious business losses that had no basis apart form the unsigned and
unaudited Profit and Loss statement which had no evidentiary value
whatever is indicative of bad faith.

Torres, Jr. v. Court of Appeals
Facts:
Judge Manuel Torres Jr. was the majority stockholder of Tormil Realty &
Development Corporation. He adopted an estate planning scheme under
which he assigned to Tormil various real estate properties as well as shares
of stocks in exchange for Tormil shares.
However inly 225,000 shares were issued to Judge Torres, as a result he
revoked the Deed of Assignment covering the Makati and Pasay properties.
Prior to the annual stockholders meeting Judge Torres assigned one shares
each to his nominees to qualify them for election to the Board. Judge Toress
himself entered said assignments in the corporations stock and transfer
book.
Dispute between two competing camps ensued during the stockholders
meeting, the group of Judge Torres were forced to leave, they left and
resumed the meeting; as a result Judge Torres nominees were all elected.
Private stockholders as minority stockholder filed a complaint with the SEC
for the annulment of the election of Judge Torres nominees.

Issue:
Whether or not there was a valid assignment and therefore valid election to
the Board of Directors

Held:
NO. It is the corporate secretarys duty and obligation to register valid
transfers of stocks and if said corporate officer refuse to comply, the
transferor- stockholder may rightfully bring suit to compel performance.

San Juan Structure and Steel Fabricators, Inc. v. Court of Appeals
Facts:
San Juan Structure and MOTORICH SALES through its treasurer, Nenita Lee
Gruenberg entered into an agreement for the transfer of a parcel of land by
the latter to the former through sale.
MOTORICH refused to execute the Transfer of Rights/ Deed of Assignment.
This prompted San Juan to file a complaint with the court to compel
MOTORICH to comply with its obligation under the contract.
In its answer, MOTORICH and Gruenberg interposed as an affirmative
defense that the President and Chairment of MOTORICH did not sign the
said agreement and therefore the same does not bind the corporation.

Issue:
1. Whether or not the plaintiff had the right to compel defendands to
execute a deed of absolute sale in accordance with the agreement;

2. Whether or not the contract is binding upon the corporation

Held:
NO. As a general rule, the acts of the corporate officers within the scope of
their authority are binding on the corporation. But when these officers
exceed their authority their actions cannot bind the corporation unless it
has ratified such acts or is estopped from disclaiming them.

Peoples Aircargo and Warehousing Inc. v. Court of Appeals
Facts:
Peoples Aircargo was organized to operate a customs bonded warehouse.
To obtain a license from the Bureau of Customs (BOC), Punsalan, its
President solicited a proposal from Sano for the preparation of a feasibility
study and another contract to retain him as consultant for the preparation
of the operations manual and the conduct of seminar/ workshop for the
employees.
Thereafter the works of Sano was submitted to the BOC and became the
basis by which Peoples Aircargo was issued license to operate.
However, after Punsalan sold his shares and resigned, the Board refused to
pay Sano on the ground that the President entered into a contract without
Board approval.
Sano filed a collection case against the corporation.

46 | P a g e

Issue:
Whether a single instance where the corporation had previously allowed its
president to enter into a contract with another without a board resolution
expressly authorizing him, has clothed its president with apparent authority
to execute the subject contract

Held:
YES. The authority of certain individuals to bind the corporation is generally
derived from law, corporate by- laws or authorization from the board, either
expressly or impliedly by habit, customs or acquiescence in the general
course of business. In the absence of a charter or by- law provision to the
contrary, the president is presumed to have the authority to act within the
domain of the general objectives of its business and within the scope of his
usual duties. And even if a certain contract is outside the usual powers of a
president, the corporations ratification of the same and acceptance of the
benefits make it binding.

Restaurante Las Conchas v. Llego
Facts:
Restaurante Services Corporation (RSC) operates Restaurante Las Conchas.
David and Elizabeth Gonzales were board members and officers of RSC. It
got involved in a legal battle with AYALA LAND INC. over the land being
occupied by the restaurant.
AYALA obtained a favourable judgment and RSC was ordered to vacate the
premises.
As a result, the employees of Restaurante Las Conchas were terminated.
The dismissed employees filed a complaint with the LA.
LA dismissed the complain; NLRC however ruled that Restaurante was liable
as well as David and Elizabeth.

Issue:
Can the David and Elizabeth be held personally liable as officers and
members of the BOD of the corporation, notwithstanding the fact that it is
the latter which is the owner of Restaurante Las Conchas?

Held:
YES. Assuming that indeed, the Restaurant Services Corporation was the
owner of the Restaurante Las Conchas and the employer of private
respondents, this will not absolve petitioners David Gonzales and Elizabeth
Gonzales from their liability as corporate officers. Although, as a rule, the
officers and members of a corporation are not personally liable for acts
done in the performance of their duties, this rule admits of exceptions, one
of which is when the employer corporation is no longer existing and is
unable to satisfy the judgment in favor of the employee, the officers should
be held to satisfy the judgment in favor of the employee, the officers should
be held liable for acting on behalf of the corporation.

Note:
An exception to the rule that officers and members of a corporation are not
personally liable for acts done in the performance of their duties is when
the employer corporation is no longer existing and is unable to satisfy the
judgment in favor of the employee. The officers in this case should be liable
for acting on behalf of the corporation.


Consolidated Food Corporation v. NLRC
Facts:
CFC is a domestic corporation engaged in the sale of food products, one of
which is Presto Ice Cream. John Gokongwei was its president, Victoria
Fadrilan was the general manager and Jaime S. Abalos was its unit manager
for Northern Luzon.
Wifredo Baron was a bonded merchandiser at CFC who received
commendations for consistently filling sales quotas in their assigned areas.
Thereafter, he was tasked, as acting section manager for northern Luzon, to
deliver for sale CFC Presto Ice Cream products to stores and outlets in
Baguio City, make inventories thereof, replace and retrieve bad orders and
to handle funds in relation to his function.
In 1990, a killer earthquake hit Baguio causing severe damage in the area.
Because the power lines were cut off, the presto ice cream products in the
possession of customers and sales outlets were damaged and became bad
orders. Consequently, Baron was tasked to conduct liquidation on account
of non-sales operations due to fortuitous event. When CFC concluded an
audit on the accountabilities of Baron, it reflected a shortage.
For the alleged irregularities committed, Baron was re-assigned to the head
office without pay pending investigation by management. There is nothing
in the records that will show that Baron was placed under preventive
47 | P a g e

suspension. Further, the withholding of Barons salaries were known to
Gokongwei, Fadrilan and Abalos. Baron, thru a memorandum, was informed
of the discrepancies appearing in the audit of accountabilities and was given
an opportunity to explain his side in writing. However, Baron, left his job
without submitting the required final explanation on the alleged
irregularities and filed a complaint for constructive dismissal. LA, affirmed
by NLRC, held that Baron was constructively dismissed.

Issue:
Whether or not the President and the other high ranking official of CFC are
liable

Held:
YES. The employers act of conducting audits and investigation on the
alleged irregularities committed by an employee and in re-assigning him to
another place of work pending the results of investigation are valid and do
not amount to constructive dismissal. However, the officers who knew and
allowed the withholding of salaries and bonuses of an employee pending his
investigation but without being placed under preventive suspension are
solidarily liable with the corporation despite the valid dismissal of the
employee.


Rural bank of Milaor v. Ocfemia
Facts:
The Ocfemias, Felicisimo and Juanita, mortgaged seven parcels of land in
favor of Rural bank of Milaor. The mortagae was later on foreclosed and
ownership was transferred to the bank.
Five of the seven parcels of land were re- sold by the bank thru its manager,
Tena, who had previously transacted business in behalf of the bank, to
Renato Ocfemia and his wife. Renato is the son of Felicisimo and Juanita.
After the execution of the Deed of Sale, the Ocfemias occupied the land and
paid the corresponding real estate taxes due thereon.
When Juanita became very sickly, the Ocfemias (Renato and wife) became
interested in having the properties registered in their names to enable
them to mortgage the same to shoulder the medical expenses of Juanita.
For the property to be transferred in their names, the Register of Deeds
required the submission of a Board Resolution from the bank confirming
both the Deed of Sale and the authority of the bank manager to enter into
such transaction.
The bank did not made such resolution on the ground that it had no records
from the old manager. This prompted the Ocfemias to file an action for
mandamus.

Issue:
May the Board of Directors of a rural bank be compelled to confirm a deed
of sale when such deed of sale was executed or entered into by the bank
manager without prior authority from the board?

Held:
YES. When a bank, by its acts and failure to act, has clearly clothed its
managers with apparent authority to sell an acquired asset in the normal
course of business, it is legally obliged to confirm the transaction by issuing
a board resolution to enable the buyers to register the property in their
names. It has a duty to perform necessary and lawful acts to enable the
other parties to enjoy all the benefits of the contract which it had
authorized.


Security Bank and Trust Co. v. Cuenca
Facts:
Security Bank and Trust Co. (SBTC) granted Sta. Ines Melale Corporaion
(SIMC), a credit wire to assist the latter in meeting the additional
capitalization requirements of its logging operations. As one of the
conditions of the credit loan facility, SBTC required the joint and solidary
signatures of Rodolfo Cuenca, the president and chairman of the BOD of
SIMC. Consequently, Cuenca executed an indemnity agreement whereby he
jointly and solidarily bound himself with SIMC. During the lifetime of the
credit loan facility, SIMC repeatedly availed of its credit line and obtained
loans from SBTC.
Meanwhile, Cuenca resigned and his shares were bought by Adolfo. When
SIMC encountered difficulty in making amortization payment on its loan, it
requested SBTC for a complete restructuring of its indebtedness. During this
time, Cuenca was no longer an officer of the corporation. SBTC
accommodated the request and agreed to restructure. Still, SIMC defaulted
48 | P a g e

in the payment of its structured loan obligations despite demands. SBTC
filed a complaint for collection of sum of money.
SBTC filed an action for collection. RTC ordered SIMC and Cuenca to jointly
and severally pay SBTC.
CA released Cuenca from any liability.

Issue:
Whether or not Cuenca may be held liable with SIMC

Held:
NO. It is a common practice to require the JSS (joint and solidary signature)
of a major stockholder or corporate officer, as an additional security for
loans granted corporations. However, there was no reason for the
corporation to assume that the officer who signed as surety would still
agree to act as surety in the restructuring agreement because at that time,
he is no longer an officer of the corporation. However, there was no reason
for the corporation to assume that the officer who signed as surety would
still agree to act as surety in the restructuring agreement, which is a new
contract, because at that time, he is no longer an officer of the corporation.
The restructuring agreement novated, not merely extended the period of
the old contract in view of the express provision to liquidate the principal
and earlier of the earlier indebtedness. Hence the indemnity surety
agreement, which was an accessory to the first loan agreement, was
likewise extinguished. Assuming the restructuring agreement merely
extended the first loan, the officer is still released from liability because an
extension granted by the creditor to the debtor without the consent of the
guarantor extinguishes the guaranty.



DOCTRINE OF CORPORATE NEGLIGENCE
Professional Services Inc. v. Court of Appeals
Facts:
Natividad underwent a sugery at the Medical City General Hospital. Later, it
was found that a gauze was left inside her vaginal area which caused her
death.

Issue:

Held:
Under the doctrine of corporate negligence, the hospitals failure to
supervise its resident physicians and nurses and to take an active step in
order to remedy their negligence renders it directly liable. The duty of
providing quality medical service is no longer the sole prerogative and
responsibility of the physician.


QUORUM

Jesus v. Court of Appeals

AUTHORITY TO SIGN CERTIFICATION AGAINST NON- FORUM SHOPPING

Philippine Rabbit Bus Line, Inc. v. Aladdin Transit Corporation

Cagayan Valley Drug Corp. v. CIR
Facts:
Petitioner operates two drug stores in Cagayan and Isabela under the name
Mercury Drug. It granted 20% sales discount to qualified senior citizens
which granted them tax credit pursuant to RA 7432.
When petitioner claimed for tax credit from the BIR, it was not acted upon.
In the decision of the CA, it dismissed the petition because the signatory
was the president without adducing proof that he was duly authorized by
the BOD to do so.

Issue:

Held:
While jurisprudence do not provide a complete listing of authorized
signatories to the verification and certification required by the rules, the
determination of the sufficiency of the authority was done on a case to case
basis. The rationale applied in other cases is to justify the authority of
corporate officers or representatives of the corporation to sign the
verification or certificate against forum shopping, being in a position to
verify the truthfulness and correctness of the allegations in the petition.
49 | P a g e



Maranaws Hotel & Resort Corp. v. Court of Appeals









TELECONFERENCING

Expert Travel & Tours Inc. v. Court of Appeals
Facts:
Korean Airlines (KAL) is a corporation established and registered in the
Republic of South Korea and is licensed to do business in the Philippines. Its
general manager in the Philippines is Suk Kyoo Kim, while its appointed
counsel was Atty. Mario Aguinaldo and his law firm.
On a certain date, KAL, through Atty. Aguinaldo, filed a Complaint against
Expert Travel & Tours, Inc. (ETI) for the collection of money. The verification
and certification against forum shopping was signed by Atty. Aguinaldo, who
indicated therein that he was the resident agent and legal counsel of KAL
and had caused the preparation of the complaint.
ETI filed a motion to dismiss the complaint on the ground that Atty.
Aguinaldo was not authorized to execute the verification and certificate of
non-forum shopping.
During the hearing, Atty. Aguinaldo claimed that he had been authorized to
file the complaint through a resolution of the KAL Board of Directors
approved during a special meeting. Upon his motion, KAL was given a
period of 10 days within which to submit a copy of the said resolution. KAL
then submitted an Affidavit of even date executed by its general manager
Suk Kyoo Kim, alleging that the board of directors conducted a special
teleconference, which he and Atty. Aguinaldo attended. It was also averred
that in that same teleconference, the board of directors approved a
resolution authorizing Atty. Aguinaldo to execute the certificate of non-
forum shopping and to file the complaint. Suk Kyoo Kim also alleged,
however, that the corporation had no written copy of the aforesaid
resolution.
The trial court denied the motion to dismiss, giving credence to the claims
of Atty. Aguinaldo and Suk Kyoo Kim that the KAL Board of Directors indeed
conducted a teleconference during which it approved a resolution as quoted
in the submitted affidavit.
Hence, the instant petition by ETI contending that it was inappropriate for
the court to take judicial notice of the said teleconference without any prior
hearing.

Issue:
Whether or not the order of the trial was proper with respect to ruling of
the validity of the teleconference during which the alleged board resolution
was granted

Held:
NO. In this age of modern technology, the courts may take judicial notice
that persons in the Philippines may have a teleconference with a group of
persons in South Korea relating to business transactions or corporate
governance. However a court cannot take judicial notice to any fact, which
in part is dependent on the existence or non-existence of a fact which the
court has no constructive knowledge. The allegation of a foreign
corporation that its board of directors conducted a teleconference and
approved the resolution authorizing its resident agent to file the complaint
and execute the certificate against forum shopping, incredible, given the
additional fact that no such allegation was made in the complaint. If the
resolution had indeed been approved, long before the complaint was filed,
the foreign corporation should have incorporated it in its complaint, or at
least appended a copy thereof.


PROXY

GSIS v. Court of Appeals
GR NO

Facts:
Issue:
50 | P a g e

Held:


POWERS OF A CORPORATION

Republic v. Acoje Mining Company, Inc.
Facts:
Acoje Mining wrote the Director of Posts (Director) requesting the opening
of a post, telegraph and money order offices at its mining camp in Sta. Cruz,
Zambales. The Director replied that Acoje Mining must provide for free
quarters, all essential equipment and must assign a responsible employee to
perform the duties of a postmaster. Acoje Mining agreed to comply with the
conditions and requested further that it be furnished with the necessary
forms for the early establishment of a post office branch.
The director again wrote Acoje Mining suggesting that a Board Resolution
be adopted assuming direct responsibility for whatever pecuniary loss that
maybe suffered by the Bureau of Posts by reason of any acts of dishonesty,
carelessness or negligence on the part of the companys employess who is
assigned to take charge of the post office. Acoje Mining complied with the
orders. A post office branch was opened at the camp. An employee of the
company was assigned as postmaster.
On a certain dates, the postmaster went on a 3-day leave but never
returned. The company informed the Manila Post Office and Provincial
Auditor of the disappearance and a shortage was found in the accounts.
Several demands were made by the government against the mining
company ut to no avail.
Hence, this present action.
Acoje Mining alleged that the Board Resolution assuming liability for the
acts of the postmaster is ultra vires and in any event, its only liability is as a
guarantor.

Issue:
1. Whether or not the act of Acoje Mining Company in opening a post
office is an ultra vires act
2. Whether or not Acoje Mining is liable for the shortage

Held:
1. NO. While as a rule, ultra vires act is committed outside the object for
which the corporation is created, there are however certain acts that
maybe performed outside of the scope of the powers expressly
conferred if they are necessary to promote the interest or welfare of
the corporation such as the establishment of a local post office in a
mining camp which is far removed from the postal facilities. The
distinction between ultra vires act and illegal act is: the first is merely
voidable and can e enforced by performance, ratification or estoppels or
on equitable grounds while the latter is void and cannot be validated.
2. YES. Acoje Mining agreed to the conditions imposed by the Director of
Post. Hence, the validity of a resolution of the board of directors
accepting full responsibility in connection with funds to be received by
its postmaster should be upheld on the ground of estoppel.



NAPOCOR v. Vera
Facts:
Sea Lion filed a complaint for prohibition and Mandamus against NPC and
Philippine Ports Authority (PPA) wherein it alleged that NPC acted in bad
faith and with grave abuse of discretion in not renewing its contract for
stevedoring services for coal handling operation at NPCs plant and in taking
over its stevedoring services.
Respondent Judge issued injunction after finding that NPC was not
empowered by its charter to engage in stevedoring and arrestre services.

Issue:
Whether or not the operation of stevedoring services is within the power of
NAPOCOR

Held:
YES. A corporation is not restricted to the exercise of powers expressly
conferred upon it by its charter but has the power to do what is reasonably
necessary or proper to promote the interest or welfare of the corporation.
NPC is formed for the purpose of generating electrical power can undertake
51 | P a g e

stevedoring services to unload coal into its pier to be brought to and fuel its
power plant, since this is reasonably necessary for the operation and
maintenance of its power plant.

Aurbach v. Sanitary Wares Manufacturing Corporation
Facts:
One of the incorporators of Saniwares, Mr. Young went abroad to look for
foreign partners. ASI, a foreign corporation domiciled in the US entered into
an agreement with Saniwares and some Filipino investors whereby ASI and
the Filipino investors agreed to participate in the ownership of an enterprise
which would engage primarily in the business of manufacturing in the
Philippines and selling here and abroad China and sanitary wares. The
parties agreed that the business operations in the Philippines shall be
carried on by an incorporated enterprise which name shall be Sanitary
Wares Manufacturing Corporation.
The agreement allowed for the cumulative voting for directors, further, it
has the provision that the management of the corporation shall be vested in
the Board of Directors (BOD) which shall consist of 9 individuals. And as long
as ASI will own 30% of the outstanding capital stock, 3 of the 9 directors
shall be designated by ASI and the other directors by the other stockholders.
Later the 30% capital stock of ASI was increased to 40%.
Conflict aroused during the annual stockholder meeting because ASI wanted
to vote its additional 10%; the Group of Filipino investors did not approve as
it would dilute their right to vote, they then voted their six nominees and
only the three nominees of ASI were accepted. The meeting was adjourned
without the resolution of the conflict. The ASI Group continued the meeting
at the elevator lobby of the building, it nominated its four nominees; Salazar
voted for himself, thus the said five directors were certified as elected
directors.
This triggered the filing of separate petitions by the parties with the SEC.
Both set of parties claimed to be the legitimate directors of the corporation.

Issue:
Who were the duly elected directors of Saniwares?
To answer this question:
a. What is the nature of the business established by the parties, was it
a joint venture or a corporation
b. Whether or not ASI Group may vote their additional 10% equity

Held:
Only those duly nominated in the annual stockholders meeting
a. The rule is that whether the parties to a particular contract have
thereby established among themselves a joint venture or some other
relation depends upon their actual intention which is determined in
accordance with the rules governing the interpretation and construction
of contracts.
Examination of the provisions of the agreement shows that the parties
establish a joint venture and not a corporation. In the selection of the
nine directors each group is assured of a fixed number of directors in
the board. Moreover ASI in its communications referred to the
enterprise as a joint venture.
b. NO. The ASI group and Salazar reiterate their theory that the ASI Group
has the right to vote their additional equity pursuant to Section 24 of
the Corporation Code which gives the stockholders the right to
cumulate their votes in electing directors.
The contention does not hold water that provision does not apply to a
joint venture as what the parties have established in the case at bar.
To allow ASI Group to vote their additional equity to help elect even a
Filipino director would be to obliterate their minority status as agreed
upon by the parties.
Cumulative voting may not be used as a device to enable ASI to achieve
stealthily or indirectly what they cannot accomplish openly. There are
substantial safeguards in the agreements which are intended to
preserve the majority status of the Filipino investors as well as the
minority status of the foreign investors group.

Notes:
A corporation cannot enter into a partnership contract but may
engage in a joint venture with other.
Cumulative method of voting- multiplying the number of shares
registered in his name by the number of directors to be elected;
give one candidate as many votes as the number of directors to be
elected multiplied by the number of his shares.

Laureano Investment & Development Corporation v. Court of Appeals
Facts:
52 | P a g e

Records show that Spouses Laureano was the majority stockholders of
Laureano Investment. Said corporation entered into a loan and credit
transactions with PNCB. For failure of the corporation to pay its obligations,
the security to the loans was foreclosed and the bank was the purchaser
during the auction sale. Titles thereof were consolidated in PNCBs name.
Bormaheco, Inc. became successor of the obligations and liabilities of PNCB
under a deed of sale/assignment wherein PNCB was bought by Bormaheco
under a bulk sale. Included therein were the properties by the petitioner
corporation which were acquired by PNCB in the foreclosure sale.
Bormaheco then filed an ex parte petition for issuance of writ of possession
against the spouses Laureano.
The corporation filed a Motion for intervention however, the corporation
instead of using its complete name used LIDECO Corporation.
Bormaheco filed a Motion to Strike out the complaint in intervention and all
other related pleadings filed by LIDECO
Petitioner contended that it was Bormaheco which first made use of LIDECO
as a shorter term prior to the filing of complaint in intervention by the
corporation under the name of LIDECO, hence BOrmaheco should be
considered estopped from denying that petitioner and LIDECO are one and
the same.
The court granted the Motion to Strike on the ground that LIDECO
Corporation and Laureano Investment and Development Corporation are
two separate and distinct entities.
Examining the records of the case showed that Bormaheco indeed made
use of the term LIDECO but it was distinctly specified that it was the shorter
term for Laureano Investment and Development Corporation.

Issue:
May a plaintiff/ petitioner which purports to be a corporation validly bring
suit under a name other than that registered with the SEC?

Held:
NO. If a corporation wishes to intervene in a case, it should use its complete
name and not an acronym. LIDECO, not being registered with the SEC, has
no juridical personality to intervene in a court suit.

Heirs of Tang Eng Kee v. CA
Facts:
After the death of Tan Eng Kee, his common-law spouse and his children
filed a suit against Tan Eng Kees brother, Tan Eng Lay. The complaint was
for accounting, liquidation and winding up of the alleged partnership
between tan Eng Kee and Tan Eng Lay. On its amended complaint,
petitioners impleaded Benguet Lumber Company.
The amended complaint alleged that Tan Eng Kee and Tan Eng Lay, pooling
their industry and resources, entered into a partnership engage in business
of selling lumber, hardware and construction supplies. They named their
business Benguet Lumber which they mutually managed until Tan Eng
Kees death. Petitioners further alleged that Tan Eng Lay and his children
caused the conversion of the partnership into a corporation called Benguet
Lumber Company to deprive the petitioners of their rightful participation in
the profits of the business.
Private respondents alleged that Tan Eng Kee was a mere employee of the
Benguet Lumber.

Issue:
Whether or not a partnership was formed between Tan Eng Kee and Tan
Eng Lay

Held:
NO. Tan Eng Kee was a mere employee , not a partner
The best evidence would have been the contract of partnership itself, or the
articles of partnership itself, but there is none.
Particular partnership distinguished from a joint venture, to wit:
a. A joint venture (an American concept similar to our joint
accounting) is a sort of informal partnership, with no firm name and
no legal personality. In a joint account, the participating merchants
can transact business under their own name and can be individually
liable thereof.
b. Usually, but not necessarily, a joint venture is limited to a single
transaction, although the business of pursuing to a successful
termination may continue for a number of years; a partnership
generally relates to continuing business of various transactions
certain kind.

Pilipinas Loan Company , Inc. v. SEC
53 | P a g e

Facts:
Filipinas Pawnshop is a duly organized corporation registered with SEC. Its
articles of incorporation states that its primary purpose is to extend loans at
legal interest on the security of either personal properties or on the security
of real properties and to finance installment sales of motor vehicles, home
appliances and other chattels.
Pilipinas Loan Company is a lending corporation duly registered with SEC.
Based on the articles of incorporation, its primary purpose is to act as a
lending investor without however, engaging in pawn broking as defined
under PD 114.
Filipinas Pawnshop filed a complaint against Pilipinas Loan Company with
SEC alleging that Pilipinas Loan Company, contrary to the restriction set by
the Commission has been operating and doing business as a pawn broker
and pawnshop or sanglaan in the same neighborhood where Filipinas
Pawnshop has had its own pawnshop for 30 years in violation of its primary
purpose and without the imprimatur of Central Bank to engage in the
pawnshop business thereby causing unjust and unfair competition with
Filipinas Pawnshop and the business name PILIPINAS ears similarity in
spelling and phonetics with the corporate name of private respondent,
FILIPINAS, creating a confusion in the minds of the public and customers of
Filipnas Pawnshop.

Issue:
Whether or not Pilipinas Loan Company has the power to engage in pawn
broking

Held:
NONE. A corporation under the Corporation Code has only such powers as
are expressly granted to it by law and by its articles of incorporation, those
which maybe incidental to such conferred powers, those reasonably
necessary to accomplish its purposes and those which maybe incident to its
existence.
In the case at bar, the limit of the powers of Pilipinas Loan Company as a
corporation is very clear, it is categorically prohibited from engaging in
pawn broking as defined under PD 114.

Heirs of Antonio Pael v. Court of Appeals
Facts:
The title of the subject property was reinstated in the name of Chin and
Mallari.
Prina Mining and Exploration Inc. a company engaged in mining, filed action
for annulment contending that it acquired said property from the Heirs of
Pael by virtue of a deed of Assignment.
During the pendency of the action, Prina changed its name to Pfina and
thereafter sought to intervene

Issue:
Whether or not it is within the power of Prina to acquire said property

Held:
NO. a mining corporation has no valid grounds in the highly speculative
business of urban real estate development.


DIVIDENDS

Nielson & Co, Inc. v. Lepanto Consolidated Mining Co.
Facts:
Nielson had agreed, for a period of five years, with the right to renew for
like period, to explore, develop and operate the mining claims of Lepanto
and to mine or mine and mill, such pay ore as may be found therein and to
market the metallic products recovered there from which may proved to be
marketable, as well as to render for Lepanto other services specified in the
contract in exchange for a management fee of P2, 500.00 a month and a
10% participation in the net profits resulting from the operation in the
operation of the mining properties.

Issue:
Whether or not Nielson is entitled to the cash dividends

Held:
NO. A corporation may legally issue shares of stock in consideration of
services rendered to it by a person not a stockholder or in payment of its
indebtedness. A share of stock issued to pay for services rendered is
equivalent to a stock issued in exchange of property because, services is
equivalent to property. Likewise, a share of stock issued in payment of
54 | P a g e

indebtedness is equivalent to issuing a stock in exchange for cash. But a
share of stock thus issued should be part of the original capital stock of the
corporation upon its organization, or part of the stocks issued when the
increase of the capitalization of corporation is properly authorized. In other
words, it is the shares of stock that are originally issued by the corporation
and forming part of the capital that can be exchanged for cash or services
rendered, or property; that is, if the corporation has original shares of stock
unsold or unsubscribed, either coming from the original capitalization or
from the increased capitalization. Those shares of stock may be issued to a
person who is not a stockholder or to a person already a stockholder in
exchange for services rendered or for cash or property. But a share of stock
coming from stock dividends declared cannot be issued to one who is not a
stockholder of a corporation.

Madrigal & Company Inc. v. Zamora
Facts:
Madrigal & Company, Inc. was engaged, among several other corporate
objectives, in the management of Rizal Cement Co., Inc..
On a certain date, Madrigal Central Office Employees Union (UNION) sought
for the renewal of its collective bargaining agreement with the petitioner.
However, petitioner requested for a deferment of the negotiations.
Subsequently, by an alleged resolution of its stockholders, the petitioner
reduced its capital stock. On another date, petitioner further reduced its
capital stock.
After petitioners failure to sit down with the UNION, the latter commenced
a case with the NLRC on a complaint for unfair labor practice with claims for
backwages and damages. Petitioner filed its position paper alleging
operational losses because Rizal Cement Co., Inc. from which it derives
income as the General Manager or Agent had ceased operating
temporarily. It further alleged that whatever profits it earned were in the
nature of dividends declared on its shareholdings in other companies in the
earning of which the employees had no participation whatsoever. Petitioner
posited that cash dividends are absolutely property of the stockholders and
cannot be made available for disposition if only to meet the employees
economic demand.

Issue:
Whether or not profits received by the corporation may be made available
for wage increments of its employees.

Held:
YES. Cash dividends received by the corporation arising from the corporate
investments in other companies are corporate earnings. As such
shareholder, the dividends paid to it were its own money which may then
be available for wage increments. It is not case of the corporation
distributing dividends in favor of its stockholders in which case, such
dividends would be the absolute property of the shareholders and hence,
out of reach by the creditors of the corporation.

Republic Planters Bank v. Agana
Facts:
Robes-Francisco Realty & Development Corporation secured a loan from
petitioner Bank; part of the proceeds of the loan, preferred shares of stocks
were issued to private respondent Corporation. In other words, instead of
giving the legal tender totalling to the full amount of the loan the Bank lent
such amount partially in the form of money and partially in the form of
stock certificate. Said stock certificates were in the name of private
respondent Adalia Robes and Carlos Robes, who subsequently endorsed his
shares in favor of Adalia Robes.
Said stocks are preferred stocks, as stated in the certificate of stocks, and
have the right to receive a quarterly dividend of 1%, cumulative and
participating. Adalia Robes then filed an action for specific performance to
compel petitioner to pay 1% quarterly interest thereon owing her under the
terms and conditions. Respondent Judge ruled that this indicates that the
same are interest bearing stocks which are stocks issued by a corporation
under an agreement to pay a certain rate of interest thereon and as such,
plaintiffs become entitled to the payment thereof as a matter of right
without necessity of a prior declaration of dividend

Issue:
Whether or not Robes is entitled to 1% as a matter of right

Held:
NO. Dividends cannot be declared for preferred shares which were
guaranteed a quarterly dividend if there are no unrestricted retained
55 | P a g e

earnings. Interest bearing stocks on which the Corporation agrees
absolutely to pay interest before dividends are paid to common
stockholders, is legal only when construed as requiring payment of interest
as dividends from net earnings or surplus only.


BY- LAWS

China Banking Corporation v. Court of Appeals
Facts:
Calapatia, a stockholder of private respondent Valley Golf and Country Club,
Inc. (VGCCI) pledged his stock certificate to China Bank as a security for a
loan he obtained.
For failure to pay his obligation, China Bank filed a petition for extrajudicial
foreclosure and requested a conduct of public auction sale of the pledged
stock. China Bank informed VGCCI of the above mentioned foreclosure
proceedings and requested that the pledged stock be transferred to China
Banks name and the same e recorded in the corporate books. However,
VGCCI wrote petitioner expressing its inability to accede to its request in
view of Calapatias unsettled accounts with the club.
Despite the foregoing, public auction was pursued and China Bank emerged
as the highest bidder. Consequently, China Bank was issued the
corresponding certificate of sale.
Subsequently, VGCCI sent Calapatia a notice demanding full payment of his
overdue account and on a later date, VGCCI caused to be published in the
newspaper a notice of auction sale of Calapatias shares of stock in said
corporation. Later on, Calapatias stock was sold at public auction for
reason of delinquency.
China Bank then filed a complaint with the SEC for the nullification of the
sale of Calapatias stock by VGCCI.
VGCCI claims a prior right over the subject share anchored mainly on
Section 3 of its by-laws which provides that after a member shall have
been posted as delinquent, the board may order his/her/its share sold to
satisfy the claims of the Club


Issue:
Whether or not China Bank or a third person is bound by a corporations by-
laws

Held:
NO. In order to be bound, a third party must have acquired knowledge of
the pertinent by-laws at the time the transaction or agreement was entered
into. Thus, a provision in the by-laws of a country club granting it preferred
lien over the share of stock of a member for unpaid dues is not binding on
the pledge of the same share of stock if the latter had no actual knowledge
of it.

Loyola Grand Villas Homeowners Association (south) v. Court of Appeals
Facts:
Loyola Grand Villas Homeowners Association (LGVHA) was organized as the
association of homeowners and residents of the Loyola Grand Villas. It was
registered with the Home Financing Corporation, the predecessor of herein
respondent, Home Insurance and Guaranty Corporation (HIGC), as the sole
homeowner organization in said subdivision. For unknown reasons
however, LGVHA did not file its corporate by-laws. On a later date, the
officers of LGVHA tried to register its by-laws. They failed to do so. They
discovered that there were two other organizations within the subdivision
the North Association and the South Association. When the President of
LGVHA inquired about the status of LGVHA, HIGC told that LGVHA had been
automatically dissolved for two reasons on the grounds that it did not
submit its by-laws in the period provided by the Corporation Code and that
there was non-user of corporate charter because HIGC had not received any
report on the associations activities. Apparently, this resulted in the
registration of the South Association with the HIGC.
These developments prompted the officers of LGVHA to lodge a
complaint with the HIGC. They questioned the revocation of LGVHAs
certificate of registration without due notice and hearing and concomitantly
prayed for the cancellation of registration of North and South Association.
LGVHA obtained a favorable ruling from HIGC; hence this instant
petition.

Issue:
Whether or not the failure to file by-laws within one month from date of
incorporation result in its automatic dissolution
56 | P a g e


Held:
NO. The legislative deliberations demonstrate that corporate dissolution for
failure to file the by-laws on time was never the intention of the legislature.
There can be no automatic corporate dissolution simply because the
incorporators failed to abide by the required filing of by-laws. The
incorporators must be given the chance to explain their neglect or omission
and to remedy the same.


PMI College v. NLRC
Facts:
PMI, an educational institution, hired Galvan as a contractual instructor.
Pursuant to this engagement, he organized classes in marine engineering.
Initially, Galvan and other instructors were compensated for services
rendered. However, for reasons unknown to Galvan, he stopped receiving
payment for the succeeding rendition of services. This prompted him then
filed a complaint with the NLRC.
Later in the proceedings, PMI manifested that under PMIs by-laws, only the
chairman is authorized to sign any contract and that Galvan , in any event,
failed to submit documents on the alleged contract of employment
between him and PMI as signed by the chairman.

Issue:
Whether or not the contract between PMI and Galvan is valid

Held:
YES. Since by-laws operate merely as internal rules among the stockholders,
they cannot affect or prejudice third persons who deal with the corporation,
unless they have knowledge of the same. Thus, a provision in the by-laws
authorizing the chairman of the board of directors only to sign contracts will
not affect the validity of the contract of a teacher who had no knowledge of
it.

Nakpil v. Intercontinental Broadcasting Corporation
Facts:
Nacpil was the Assistant General Manager for Finance and Comptroller of
Intercontinental Broadcasting Corporation (IBC). When a new president
assumed office, Nacpil was forced to resign due to harassment and
humiliation he experienced from the new president. His retirement
benefits were not paid, hence, Nacpil filed a complaint for illegal dismissal
and non-payment of benefits with the Labor Arbiter. IBC filed a Motion
to Dismiss alleging Labor Arbiters lack of jurisdiction because the case
involved intra-corporate dispute cognizable by SEC.

Issue:
Whether or not the dispute was an intracorporate dispute cognizable by the
SEC
Held:
That the position of Comptroller is not expressly mentioned among the
officers of the IBC in the by-laws is of no moment, because the IBCs Board
of Directors is empowered under Section 25 of the Corporation Code and
under the Corporation By-Laws to appoint such other officers as it may
deem necessary.
Where the corporate office is not specifically indicated in the roster of
corporate offices in the by-laws of the corporation, the board of directors
may also e empowered under the laws to create additional officers as may
be necessary. And where the by-laws authorized the directors to create
office, the directors may create the office comptroller. Since the
appointment as comptroller required formal action of the board, petitioner
is a corporate officer whose dismissal maybe subject of intra-corporate
controversy cognizable by the SEC under Section 5 (c) of PD 902-A
Note:
Intracorporate disputes are now cognizable by the RTC as a special
commercial court


SUBSCRIPTION CONTRACT

Ong v. Tiu
Facts:
The Tius invited the Ongs to invest in First Landlink Asia Development
Corporation (FLADC) for the construction of Masagana Citimall in Pasay City.
Under the pre-subscription agreement, they agreed to maintain equal
shareholdings and that the Tius were entitled to nominate the Vice-
57 | P a g e

president and the treasurer plus five directors while the Ongs were entitled
to nominate the President and secretary and six directors.
The Ongs paid P100 million in cash while the Tius committed to contribute a
four storey building and two parcels of land, P30 million and P48.9 million to
cover their stock subscription. The business harmony was short lived and
the Tius rescinded the pre-subscription contract accusing the Ongs of
refusing to credit to FLADC shares covering the real property contribution,
preventing them from assuming the position of the vice-president and
treasurer and refusing to give them office spaces as agreed upon.
The Ongs for its part alleged that although the Tius executed a Deed of
Assignment in favour of FLADC, the Tius refused to pay the capital gains tax
and documentary stamp tax, without which, the SEC will not approve the
valuation of the Tius property contribution.

Issue:
Whether or not the pre-subscription contract may be rescinded

Held:
NO. when a subscriber assigned properties and infused capital to the
corporation upon the invitation of a majority stockholder and in exchange
for shares of stock under a pre- subscription agreement, the agreement
cannot be rescinded since the subject matter of the contract was the
unissued shares of the Corporation allocated to the subscriber. Further the
pre- subscription contract was in fact a subscription contract as defined
under Section 60 of the Corporation Code.

TRUST FUND DOCTRINE

Ong v. Tiu
Issue:
Whether or not the rescission will violate the trust fund doctrine.

Held:
YES. The Trust Fund Doctrine provides that subscriptions to the capital stock
of a corporation constitute a fund to which the creditors have a right to look
for the satisfaction of their claims. This doctrine is the underlying principle
in the procedure for the distribution of capital assets, embodied in the
Corporation Code, which allows the distribution of corporate capital only in
3 instances amendment of the Articles of Incorporation to reduce the
authorized capital stock, purchase of redeemable shares by the corporation,
regardless of the existence of unrestricted retained earnings, and
dissolution and eventual liquidation of the corporation. Furthermore, the
doctrine is articulated in Section 41 on the power of a corporation to
acquire its own shares and in Section 122 on the prohibition against the
distribution of corporate assets and property unless the stringent
requirements therefor are complied with.
Rescission of a PRE-SUBSCRIPTION AGREEMENT between the subscribers
and the corporation will effectively result in the unauthorized distribution of
the capital assets and property of the corporation, thereby violating the
Trust Fund Doctrine and the Corporation Code, since rescission of a
subscription agreement is not one of the instances when distribution of
capital assets and property of the corporation is allowed.


STOCKHOLDERS

Baltazar v. Lingayen Gulf Electric Power Company, Inc.
Facts:
Baltazar and Rose were among the incorporators of Lingayen Gulf having
subscribed to 600 and 400 shares of capital stock or a total par value of
P60,000 and P40,000 respectively. It is alleged that it has always been the
practice and procedure of the Corporation to issue certificates of stock to its
individual subscribers for unpaid shares of stock.
Of the 600 shares, Baltazar paid 535 shares and the corporation issued in his
name fully paid up and non assessable certificate. After making transfers
and acquiring new shares, he had to his credit 375 shares of fully paid
stocks, duly covered by certificate of stocks issued to him.
On the annual stockholders meeting the Ungson Group which controlled
the corporation since 1955 passed three resolutions:

Resolution No. 2, declaring all watered stocks issued to Baltazar and
Rose among others of no value and consequently cancelled from
the books of the corporation;
Resolution No. 3, declaring that all unpaid subscription should bear
interest annually, and any or all payments made on said unpaid
subscription be applied first to the payment of the interest; and
58 | P a g e

Resolution No. 4, disqualifying the delinquent subscribers from
exercising their right to vote until full payment of the unpaid
subscription.

An amicable settlement was entered and was approved by the court, the
court ruled that the condonation and reduction of the interests as
stipulated by the parties is valid in the absence of fraud.

Issue:
Whether or not a stockholder who has not fully paid his subscription be
divested of his voting rights.

Held:
NO. If a stockholder in a stock corporation subscribes to a certain number of
shares of stock, and makes partial payment for which he is issued
certificates of stock, he is entitled to vote the latter, notwithstanding the
fact that he has not paid the balance of his subscription which has been
called for payment or declared delinquent.

Datu Tagoranao v. SEC
Facts:
Datu Tagoranao subscribed 469 shares in Jamiatul Philippine-Al Islamia, Inc.
Sometime on October 1975, the corporation filed a certificate of increase of
its capital stock from P200,000 to P1,000,000 which was approved. Of the
increased capital stock,P160,000 worth of shares were subscribed by Fatima
Ramos, Tarhata Lucman and moki-in Alonto.
Datu Tagoranao filed with SEC alleging that the additional issue of
previously subscribed shares violated his pre-emptive right and it was illegal
considering that he wa not notified of the meeting wherein the proposed
increase was in the agenda. The SEC hearing officer, which was affirmed by
SEC en banc ruled against Datu Tagoranao.

Issue:
Whether or not stockholders meeting is necessary for issuance of
unsubscribed portion of capital stock

Held:
NO. The power to issue shares of stocks is lodged in the board of directors
and no stockholders meeting is necessary to consider it because additional
issuance of shares of stocks does not need the approval of the stockholders.
The by-laws of the Corporation itself states that the Board of Trustees
shall, in accordance with law, provide for the issue and transfer of shares of
stock of the institute and shall prescribe the form of certificate of stock of
the Institute.

Edward Keller & Co., Ltd. v. COB Group Marketing Inc.
Facts:
Keller appointed COB as the exclusive distributor of its household products.
Under the agreement, Keller sold on credit its products to COB. As security
Manahan mortgaged her land in favor of Keller and assumed solidarily with
COB the faithful compliance of the sales agreement.
A second sales agreement was made, as security Lorenzo Jr. and Sr.
mortgaged their land.
The credit purchases were appraised by Bax, the president and General
Manager of Keller, it was found out that COB owed Keller about P179,000.
After several days, COB executed a CM over its 12 trucks which however
was already mortgaged to Northern Motors, the latter did not gave its
consent to such.
COB failed to pay and the mortgages were foreclosed.

Issue:
Whether or not the stockholders who mortgaged their property are
solidarily liable with the corporation

Held:
YES. A stockholder is personally liable for the financial obligations of a
corporation to the extent of his unpaid subscription.

Apocada v. NLRC
Facts:
Apocada was an employee of Intrans Philippines Inc. Sometime in August
1985, Mirasol persuaded Apocada to subscribe to 1,500 shares. He agreed
and made an initial payment of P37,500. He became the president and
general manager but on 1986 he resigned.
59 | P a g e

The same year he instituted a complaint for unpaid wages with the NLRC.
Intrams admitted that there is due to Apocada an amount of P17,000 but
the corporation applied the same to the unpaid balance of his subscription.

Issue:
Can an obligation arising from non payment of stock subscription be off set
against a money claim of an employee against his employer.

Held:
NO. An obligation arising from non- payment of stock subscriptions to a
corporation cannot be offset against a money claim of an employee against
the employer. Unpaid subscriptions are not due and payable until a calls
made by the corporation for payment through a board resolution and the
contract of subscription does not specify the due date of payment.


Africa v. PCGG
Facts:
In 1986 the PCGG sequestered Eastern Telecommunications Philippines Inc.
(ETPI). The sequestration order was partially lifted and 40% of the shares of
stocks owned by Cable & Wireless Ltd. were freed from sequestration.
Pursuant to a PCGG resolution, the annual stockholders meeting convened.
Eduardo Villanueva as PCGG nominee was elected as president, the other
nominees were likewise elected to different positions. This triggered a chain
of contentious proceedings before the Sandiganbayan and the Supreme
Court between the members of the ETPI board and PCGG nominees elected
ETPI Board.
A series of action for injunction was filed with the SC and the
Sandiganbayan, seeking to enjoin PCGG and its nominees from exercising
the powers of said positions. They further contend that the assumption by
Villanueva to said position was effected without due process of law through
the PCGG using and voting sequestered shares without legal justification.
PCGG and its nominees to the ETPI board filed a motion to dismiss on the
ground of lack of jurisdiction alleging among other things that the complaint
partakes of the nature of a suit against the State without its consent and
that the plaintiffs are not the real parties in interest because it constitutes
an action for quo warranto.
Pending the resolution of the motion to dismiss filed with the
Sandiganbayan, the court upon request issued a subpoena duces tecum and
ad testificandum ordering the PCGG or its representatives to produce the
stock and transfer book and all stubs of the outstanding stock certificates of
ETPI and the minutes of all meetings of the Board of Directors and
Stockholders of ETPI held from January 29, 1988.
PCGG moved to quash the subpoenas but the motion was denied. PCGG
then filed an urgent petition for certiorari contending that the incidental
orders were issued grave abuse of discretion

Issue:
Whether or not sequestration automatically deprives a stockholder of his
right of inspection

Held:
NO. The right of inspection of a stockholder is subject to the following
limitations:
The right of inspection should be exercised at reasonable hours of business
days;
The person demanding the right to examine and copy excerpts from the
corporate records and minutes has not improperly used any information
secured through any previous examination of the records of such
corporation; and
The demand is made in good faith or for legitimate purpose.

Note:
PCGG, being a conservator, cannot exercise acts of dominion over the
property sequestered. the act of sequestration does not divest title over
said property; it does not make PCGG the owner of the same.
As to the defense of immunity from suit laid down by the nominees of
PCGG- the action does not impose liability against PCGG as a government
entity but imposes liability in their personal and individual capacity.


Republic v. Sandiganbayan
Facts:
60 | P a g e

UNIMOLCO and Benedicto were among the Stockholders of ETPI.
Benedicto ceded to the government 204,000 shares of stocks in ETPI
equivalent to 51% equity.
The remaining 49% were released from sequestration. On April 24, 1996,
UNIMOLCO through a letter offered to sell to ETPI the remaining 49%. ETPI
took no action. The AOI of ETPI provided that the corporation enjoys the
right of first refusal; that it must be exercised by the corporation within 30
days from receipt of the written offer to sell.
Since no action as regards the offer was taken, UNIMOLCO sold to SMART
the 49% equity.
On June 1996 the PCGG issued a resolution enjoining all stockholders from
selling their shares without written conformity from the PCGG.
The PCGG through the Sol. Gen filed an action to rescind the sale on the
ground of violation of the right of first refusal as provided in the AOI of the
corporation, the petitioner contended that it received the notice of offer to
sell only on August 30, 1996 therefore it had until September 26 to exercise
the right of first of refusal; that on September 24 it sent a letter to
UNIMOLCO stating that the Government will exercise the right in the form
of a set off against the assets of Benedicto which was still due to the
Philippine Government.
The court however ruled that notice was given not on August but on April,
therefore the right had already lapsed.

Issue:
Whether or not the PCGG validly exercised its right of first refusal

Held:
NO. Petitioners right of first refusal was not seasonably exercised.
Stockholders may be given the right of first refusal if so stipulated in the
AOI. If the right is required to be exercised within 30 days of written notice
of the intended sale, the stockholder cannot complain of the violation of his
right if it did not exercises such right within the same period from actual
knowledge of the intended sale.

DERIVATIVE SUIT

Republic v. Cuaderno
Facts:
Damaso Perez, a stockholder of Republic Bank, instituted a derivative suit
against Miguel Cuaderno, Beinvenido Dizon, Boardof Directors of the
Republic Bank and the Monetary Board of Central Bank. In his complaint, he
alleged that frauds were committed when Pablo Roman, chairman of Board
of Directors of the Republic Bank granted loans to fictitious and non-existing
persons amounting to almost 4 million. Acting upon the complaint,
Cuaderno and Monetary Board conducted an investigation. Later on, it
ordered that a new board of the Republic Bank be elected. Damaso
therefore prayed for writ of preliminary injunction against Monetary Board
to prevent its confirmation of the appointment of Cuaderno and Dizon. In
its answer, the Monetary Board alleged that the plaintiff had no legal
capacity to sue.

Issue:
Whether or not Perez has the capacity to sue in behalf of the corporation

Held:
YES. An individual stockholder may institute a derivative suit on behalf of
the corporation, wherein he holds stock, in order to protect corporate
rights, whenever the officials of the corporation refuse to sue, or are the
ones to be sued or hold the control of the corporation. In such actions, the
suing stockholder is regarded as the nominal party, with the corporation as
the real party in interest.


San Miguel Corporation v. Kahn
Facts:
Fourteen Corporations acquired shares of stock of San Miguel. The shares
were placed under a voting trust agreement in favour of Andres Soriano Jr.
and when he died, Eduardo Cojuangco was elected Substitute trustee with
the power to delegate the trusteeship in writing to Andre Soriano III.
On March 1986, an agreement was executed where Andres Soriano III as
the buyer and the fourteen corporations as the seller. According to Soriano,
the real buyer is Neptunia Corporation Limited, a wholly owned subsidiary
of San Miguel International and that it was Neptunia which made the down
payment from the proceeds of certain loan.
Later on, PCGG sequestered the shares on the ground that the stock
belonged to Eduardo Cojuangco. However, in December, SMC Board, by
61 | P a g e

resolution, decided to assume the loans incurred by Neptunia. Eduardo
delos Angeles, one of PCGG representatives in SMC Board, impugned the
resolution and filed a derivative suit with the SEC against the 10 Board of
Directors of SMC. Ernest Kahn moved to dismiss the derivative suit alleging
that delos Angeles has no legal capacity to sue but his motion was denied by
the SEC hearing officer which was reversed by the court.

Issue:
Whether or not delos Angeles has the legal capacity to sue

Held:
YES. A stockholder filing a derivative suit is not suing in his own behalf of the
corporation, the fact that his shareholding is significant does not preclude
him from filing the suit. It is also not necessary that a stockholder be a
director to be entitled to file a derivate suit.

Commart Philippines, Inc. v. SEC
Facts:
Two brothers, Jesus and Mariano Maglutac organized Commart (Phils.), Inc.
Sometime in June 1984, the two brothers agreed to go their separate ways,
with Mariano being persuaded to sell to Jesus his shareholdings in Commart
amounting to 25% of the outstanding capital stock.
Marianos wife, Alice M. Maglutac (Private respondent herein) who has
been for years a stockholder and director of Commart, did not dispose of
her shareholdings, and thus continued as such even after the sale of
Marianos equity.
Shortly after the sale of his equity in Commart to Jesus, Mariano allegedly
discovered that for several years, Jesus and his wife Corazon (who was
herself a director) had been siphoning and diverting to their private bank
accounts in the United States and in Hongkong gargantuan amounts sliced
off from commissions due Commart from some foreign suppliers.
Consequently, spouses Mariano and Alice Maglutac filed a complaint with
the SEC against Jesus Maglutac and rest of the members of the Board of
Directors of Commart.
SEC held that the suit is a derivative suit.

Issue:
Whether or not SEC correctly held that the case was a derivative suit

Held:
YES. SEC correctly hold that the case was a minority stockholdings
derivative suit and correctly sustained the hearing panels denial of the
motions to dismiss. Readily shows that it avers the diversion of corporate
income into the private bank accounts of petitioner Jesus T. Maglutac and
his wife. Likewise, the principal relief prayed for in the complaint is the
recovery of a sum of money in favor of the corporation. This being the case,
the complaint is definitely a derivative suit. Consequently, the SEC correctly
held that the case was a minority stockholders derivative suit and correctly
sustained the hearing panels denial- insofar as Alice Maglutac was
concerned- of the motions to dismiss it.

Goachan v. Young
Facts:
Felix Gochan Sr.,s daughter, Alice inherited 50 shares of stock in Gochan
Realty. Alice died leaving the 50 shares to her husband, John Young Sr.
John Sr., requested Gochan Realty to partition the shares of his late wife by
cancelling the stock certificates in his name and issuing in lieu thereof, new
stock certificates in the names of (herein respondents). Gochan Realty
refused, citing as reason, the right of first refusal granted to the remaining
stockholders as provided under the Articles of Incorporation.
Cecilia Gochan Uy and Miguel Uy as stockholders of Gochan filed a
complaint with the SEC for issuance of shares of stock to the rightful
owners, nullification of shares of stock, reconveyance of property impressed
with trust, accounting, removal of officers and directors and damages
against respondents. SEC ruled against complainant heirs. Aggrieved, they
filed a petition for review with the Court of Appeals.

Issue:
Whether or not Spouses Uy could properly bring a derivative suit in the
name of Gochan Realty to redress the wrongs allegedly committed against it
for which the directors refused to sue

Held:
62 | P a g e

YES. The spouses have the capacity to file a derivative suit in behalf of the
corporation and for the benefit of the corporation. The reason for this is
that, the allegations in the complaint make them out as stockholders at the
time the questioned transaction occurred, as well as at the time the action
was filed and during the pendency of the action.

Reyes v. Hon. RTC Makati Branch 142
Facts:
Oscar and Rodrigo Reyes are two of the four children of Pedro and
Anastacia. They owned shares of stock in Zenith Insurance Corporation.
When Pedro died, his estate was judicially partitioned but when Anastacia
died later on, no partition was made.
Zenith and Rodrigo filed a complaint with SEC against Oscar to obtain an
accounting of the funds and assets of Zenith Insurance Corporation which
are in the possession of Oscar and to determine the shares of stocks of
deceased Pedro and Anastacia that were arbitrarily and fraudulently
appropriated by Oscar to himself.
Oscar denied the allegation and asserted that the suit is not a bona fide
derivative suit because the requisites therefore have not been complied
with.
RTC ruled against Oscar. The same was affirmed by the Court of Appeals.

Issue:
Whether or not the complaint is a derivative suit

Held:
Rodrigos bare claim that the complaint is a derivative suit will not suffice to
confer jurisdiction of RTC (as special commercial court) if he cannot comply
with the requisites for the existence of derivative suit. These requisites are:
the party bringing the suit is a shareholder during the time the transaction
complained of, the party has tried to exhaust intra-corporate remedies, the
cause of action actually devolves upon the corporation. And based on these
standards, the complaint is not a derivative suit. Rodrigo, not being a
shareholder, being a mere transferee-heir and he was not able to exhaust
the available remedies within the corporation.


Hi- Yield Inc. v. Court of Appeals
Facts:
Roberto Torres filed a petition for the annulment of real estate mortgage
and foreclosure sale over two parcels of land located in Marikina and
Quezon City. HI-yield moved for the dismissal of the petition on the ground
of improper venue. RTC denied the motion and held that the case was in
the nature of ac derivative suit cognizable by the special commercial court.

Issue:
Whether or not the action to annul the real estate mortgage and
foreclosure sale is a mere incident of the derivative suit

Held:
Where a minority stockholder alleged in his petition that earnest efforts
were made to reach a compromise among family members/stockholders
before he filed the case and that the Board of Directors did nothing to
rectify the unauthorized loan and mortgage by the corporation, the
derivative suit is proper. The action to annul the real estate mortgage
should only be seen as incidental to the derivative suit. The RTC of the city
where the principal office of the corporation is located has jurisdiction even
though the mortgaged properties are located in different jurisdiction.


SHARES OF STOCK/ STOCK CERTIFICATES

Rivera v. Florendo
Facts:
Fujiyama Hotel and Restaurant Inc. was organized by Rivera and four other
incorporators.
Isamu Akasa, a Japanese national alleged that he was the real owner of the
shares of stock in Riveras name. He sold to Tsuchiya the share with the
assurance that the latter will be made president of the corporation.
After the sale was consummated and the consideration was paid, Rivera
refused to make the indorsement unless he is also paid.
The buyers made several attempts to register their stock certificates with
the corporation, but the latter refused. This prompted the buyers to file a
special civil action for mandamus. The corporation moved to dismiss the
63 | P a g e

petition on the ground that the case being intracorporate falls under the
jurisdiction of the SEC.

Issue:
Whether or not mandamus will lie

Held:
NO. Mandamus will not lie when the shares of stock are not indorsed by the
registered owner who is specifically objecting to the registration thereof in
the corporate books.

Embassy Farms Inc. v. Court of Appeals
Facts:
Alexander G. Asuncion (AGA) and Eduardo B. Evangelista (EBE) entered into
a MOA. Under their agreement EBE was to transfer to AGA 19 parcels of
land together with all of his shares of stocks in Embassy Farms Inc.
AGA for his part was to pay to EBE a total sum close to 8.6M.
Pursuant to the MOA, EBE turned over to AGA the effective control and
management of the piggery at Embassy Farms.
Despite indorsement, EBE retained possession of the shares and opted to
deliver the same only upon full compliance of AGA of his obligations under
the MOA.
Notwithstanding the non- delivery of shares, AGA transferred shares of
stocks to several persons.

Issue:
Whether or not the transfers made by AGA were valid

Held:
NO. Where the seller indorsed the stock certificates but did not deliver
them, ownership of the shares cannot be transferred to the buyer. For an
effective transfer of shares of stocks there must be indorsement coupled
with delivery.


Tan v. SEC
Facts:
Alfonso Tan had 400 shares of capital stock standing in his name in the
books of Visayan Educational Supply Corporation evidenced by Stock
Certificate No. 2.
Alfonso sold 50 of his 400 shares to Angel and Teodoro Tan to enable the
latter to qualify for a seat in the board.
As a result of the sale, Stock Cert. No. 2 was cancelled and the
corresponding stock certificates No. 6 and 8 were issued to the transferees
and Alfonso.
Stock Certificate No. 2 was given back to Alfonso for indorsement, however
he deliberately withheld them for unknown reasons.
After some time Alfonso withdrew from the corporation, further he
exchanged his stock certificate no. 8 for stock- in trade worth 8M
considering that the par value of such was only P35,000.
Years later, Alfonso filed an action with the SEC questioning the cancellation
of Stock Certificate No. 2 contending that the same was made without valid
indorsement from him.
SEC declared the cancellation null and void. SEC en banc reversed.

Issue:
Whether the cancellation was valid

Held:
YES. Where the stockholder sold his share and was paid therefor and a new
stock certificate has been issued to the buyer but the stockholder did not
return the stock certificate when it was sent to him for endorsement, the
cancellation of the stock certificate is valid despite the lack of endorsement.

Razon v. Intermediate Appellate Court
Facts:
Razon organized E. Razon Inc.
In 1966, 1,500 shares were issued in the name of Chuidan, on such basis he
was elected as a director. From the time of issuance Razon did not question
the ownership of Chuidan.
Vicente Chuidan, the administrator of the intestate estate of his father
Chuidan demanded for the delivery of the stock certificates covering his
fathers shares which were in the possession of Razon.
64 | P a g e

Razon refused to deliver the same to Chuidian alleging that it was
voluntarily given to him and it was he who paid for all the subscription of
the shares.

Issue:
Who is the owner of the share

Held:
CHIUDAN. Since the certificate of stock covering the questioned sharres was
never indorsed to Razon, the inevitable conclusion is that the shares belong
to Chuidan.
The person who claims ownership over the questioned stock must show
that the same were transferred to him by proving that all the requirement
for effective transfers were made.

Rural Bank of Salinas, Inc. v. Court of Appeals
Facts:
Clemente Guerrero executed a SPA in favor of his wife, authorizing the
latter to sell his share of stocks in the bank.
Pursuant to the SPA, Melania the wife sold the shares. Clemente after some
time died.
Melania presented the deed of assignments to the bank for registration
with request for the transfer of the shares in favor of the transferee and
issuance of new certificate of stocks in the name of the buyers.
The bank refused on the ground that upon the death of Clement the subject
shares became the property of his estate which should be settled and
liquidated before the any distribution could be effected.
This refusal prompted Melania to file a petition for Mandamus to compel
the bank to effect the transfer.

Issue:
Whether the corporation may refuse to register the shares on account of
the stockholders death

Held:
NO. Where a SH executed a SPA in favor of his wife who pursuant to the SPA
sold the shares, but after the sale, the SH died, the corporation cannot
refuse to register the shares in favor of the assignee on the pretext that
upon the death of the SH, his share became the property of his estate which
should settled and liquidated first before any distribution could be effected.
It is the ministerial duty of the corporation to register the shares of stock
which were assigned in the name of the assignee even if there is a pending
action in court questioning the validity of the assignment.

Garcia v. Jomouad
Facts:
Dico was employed by Garcia as manager of his Young Auto Supply.
In order to assist him in entertaining clients Garcia lent his proprietary
ownership certificate (POC) in the Cebu Country Club.
Thereafter, Dico resigned as manager of Young Auto Supply. Upon demand
of Garcia, Dico returned POC to him. Dico then executed a Deed of Transfer
covering the subject certificate in favor of petitioner. The Club was
furnished with a copy of said deed but the transfer was not recorded in the
books of the Club because Garcia failed to present proof of payment of the
requisite capital gains tax. Meanwhile, a collection case was filed by Spouses
Antinon against Dico, whereby judgement was rendered ordering him to
pay. When judgement became executory, the sheriff proceeded to levy the
POC in the name of Dico and scheduled it to be sold at a public auction.
Claiming ownership over the POC, Garcis sought to enjoin the auction

Issue:
Whether or not the transfer made by Dico to Garcia was valid against the
Atinon Spouses

Held:
NO. the transfer of the subject certificate made by Dico to Garcia was not
valid as to the spouses Atinon, the judgment creditors, as the same still
stood in the name of Dico.
Section 63 of the Corporation Code provides that no transfer shall be valid
except as between the parties, until the transfer is recorded in the books of
the corporation showing the names of the parties to transaction; the date of
transfer, the number of certificate/s and the number of shares transferred.

Rural Bank of Lipa, Inc. v. Court of Appeals
Facts:
65 | P a g e

Villanueva and his wife executed a Deed of Assignment in favor of the Rural
Bank of Lipa wherein they acknowledged their indebtedness to the bank
and that by virtue of which they agreed to assign their shares in the said
bank or liquidate their shareholding in the bank in the event that the
proceeds of their real property is not sufficient to cover their full obligation.
The Spouses failed to settle their obligations with the bank, consequently
the shares were converted into treasury shares.
However, the certificate of stocks were neither indorsed nor delivered, they
remained in the possession of the Spouses.
Later the Villanuevas questioned the legality of the conversion of their
shares.
A stockholders meeting was conducted but the Villanuevas were not
notified, the latter questioned such, the new set of officers replied that the
Villanuevas have already relinquished their rights as stockholder by virtue of
the agreement and hence they are no longer entitled to any notice.

Issue:
Whether or not the Villanuevas may be deprived of their rights as
stockholder by virtue of the conversion or transfer of the stocks.

Held:
NO. For a valid transfer of stocks, there must be strict compliance with the
mode of transfer prescribed by law. The requirements are:
a. There must be delivery of the stock certificate:
b. The certificate must be endorsed by the owner or his attorney-
in-fact or other persons legally authorized to make the transfer;
and
c. To be valid against third parties, the transfer must be recorded
in the books of the corporation. As it is, compliance with any of
these requisites has not been clearly and sufficiently shown.

A deed of assignment of shares without requisite endorsement and delivery
is valid only as between the parties. It does not necessarily make the
transfer effective against the corporation. Consequently the assignees
cannot enjoy the status of a stockholder, cannot vote or be voted for, and
will not be entitled to dividend in so far as the assigned shares are
concerned. The assignors cannot, as yet be deprived of their rights as
stockholders, until and unless the issue of ownership and transfer of the
shares in question are resolved with finality.

Ponce v. Alsons Cement Corporation
Facts:
Gaid was an incorporator of Victory Cement. From the time of
incorporation, no certificate of stocks was issued in the name of Gaid. In a
Deed of Undertaking and Indorsement, Gaid assigned and endorsed his
shares of stock in favor of Ponce. No record of any assignment of transfer
were made in the books of the corporation, neither were there instructions
or authority from Gaid for such transfer or assignment.
Later the corporation changed its name to Alsons. Ponce demanded from
Alsons and its corporate secretary Giron, Jr. the issuance of certificate of
stocks in his favor by virtue of the assignment made by Gaid. ACC and Giron
refused. Ponce filed a petition for mandamus to compel ACC to effect the
transfer.

Issue:
Whether or not the certificate of stock should be issued in favor of Ponce.

Held:
Pursuant to Section 63 of the Corporation Code, a transfer of shares of stock
not recorded in the stock and transfer book of the corporation is non-
existent as far as the corporation is concerned. Without such recording the
transferee may not be regarded by the corporation as one among its
stockholders and the corporation may legally refuse the issuance of stock
certificates in the name of the transferee even when there has been
compliance with the requirements of Section 64 of the Corporation Code.
The situation would be different if the petitioner was himself the registered
owner of the stock which he sought to transfer to a third party, for then he
would be entitled to the remedy of mandamus. It has been made claear that
before a transferee may ask for the issuance of stock certificates, he must
first cause the registration of the transfer and thereby enjoy the status of a
stockholder insofar as the corporation is concerned. A corporate secretary
may not be compelled to register transfer of shares on the basis merely of
an indorsement of stock certificates. With more reason a corporate
secretary may not be compelled to issue stock certificates without such
registration
66 | P a g e


Republic v. Estate of Hans Menzi
Facts:
Several writs of sequestration were issued by the PCGG in pursuit of the
reputedly vast Marcos fortune.
Following a lead that Marcos had substantial holdings in Bulletin Publishing
Corporation (Bulletin), the PCGG issued a Writ of Sequestration
sequestering the shares of Marcos, Zalamea, Yap, Eduardo M. Cojuangco,
Jr. and their nominees and agents in Bulletin. Another Writ of Sequestration
followed, this time sequestering the shares of stock, assets, properties,
records and documents of Hans Menzi Holdings and Management, Inc.
(HMHMI). The Republic then instituted before the SB, a complaint for
reconveyance, reversion, accounting, restitution and damages. The
complaint substantially averred that Yap, Cojuangco and Zalamea knowingly
and willingly acted as the dummy, nominee or agent of the Marcos spouses.
The complaint was amended dropping Zalamea as defendant in view of the
Deed of Assignment which he executed, assigning, transferring and ceding
to the Government Bulletin shares registered in his name.
On the other hand , the Estate of Menzi for its defense insists that Campos,
Cojuangco and Zalamea were nominees of Menzi, not dummies of Marcos,
because the stock certificates covering the contested blocks of shares were
indorsed in blank and remained in Menzis possession

Issue:
1. Whether or not possession of the stock certificates is tantamount to
ownership over the share covered by the same
2. Whether or not Campos and the others validly ceded the subject
shares in favor of the government
Held:
1. NO. The fact that the stock certificate were found in Menzis possession
does not prove that he owns the same. A stock certificate is only a
tangible evidence of ownership; its presence or absence does not affect
the rights of the owners.
2. YES.

NAUTICA CANNING CORPORATION v. ROBERTO YUMUL
Facts:
Yumul had 1 share in Nautica given by Dee.
First Dominion Prime Holdings Inc., Nauticas parent company, through
Chairman Dee granted Yumul an option to purchase up to 15% of the total
stocks it subscribed from Nautica.
First Prime Holdings through a Deed of Trust and Assignment assigned its
14,999 shares in Nautica to Yumul. The agreement provided that the
shares were acquired and paid for in the name of the assignor only for
convenience, but actually executed in behalf of and in trust for the assignee.
Nautica declared a P35,000,000 cash dividend, P8,250,000 of which was
paid to Yumul representing his 15% share.
In 1996, he resigned from Nautica. He wrote to Dee requesting the latter to
formalize his offer to buy Yumuls 15% shares on or before August 20, 1996
and demanding the issuance of certificate of stocks in his name should Dee
refused to buy the same. The request was denied on the ground that Yumul
was not a stockholder of the corporation.
Sometime in 1996 Yumul requested that the Deed of Trust be recorded in
the Stock and Transfer Book of Nautica, and that he as a stockholder, be
allowed to inspect the books of the corporation.
Yumuls request denied because he neither exercised the option to
purchase the shares nor paid for the acquisition price of the shares. It was
further contended that the cash dividend he received was held only in trust
for First Dominion Prime Holdings. This prompted Yumul to file an action for
Mandamus before the SEC with the prayer that Nautica be ordered to
record the transfer and that stock certificate be issued in his name.
Both the SEC en banc and the CA ruled for Yumul.

Issue:
Whether Yumul is a stockholder of the corporation, thereby sustaining his
right of inspection

Held:
YES, as such he can exercise his right of inspection under the Corporation
Code.
In the case at bar, the SEC and the Court of Appeals correctly found Yumul
to be a stockholder of Nautica, of one share of stock recorded in Yumuls
name, although allegedly held in trust for Dee. Nauticas Articles of
Incorporation and By-laws, as well as the General Information Sheet filed
with the SEC indicated that Yumul was an incorporator and subscriber of
one share. E ven granting that there was an agreement between Yumul and
67 | P a g e

Dee whereby the former is holding the share in trust for Dee, the same is
binding only as between them. From the corporations vantage point,
Yumul is its stockholder with one share, considering that there is no
showing that Yumul transferred his subscription to Dee, the alleged real
owner of the share, after Nauticas incorporation.

Note:
The validity of a business wholly owned by one individual is not affected
when such individual gives nominal ownership of only one share of stock to
each of the other four incorporators. This is not necessarily illegal, but valid
only between and among the incorporators privy to the agreement. It does
bind the corporation which, at the time the agreement is made, was non-
existent. Thus, incorporators continue to be stockholders of a corporation
unless, subsequent to the incorporation, they have validly transferred their
subscriptions to the real parties in interest. As between the corporation on
the one hand, and its shareholders and third persons on the other, the
corporation looks only to its books for the purpose of determining who its
shareholders are.

A person is considered a stockholder if the AOI and By-laws as well as
General Information Sheet filed with the SEC indicated that such person was
an incorporator and subscriber of one share.


PACIFIC BASIN SECURITIES CO., INC. v. ORIENTAL PETROLEUM AND
MINERALS CORP.
Facts:
Pacific Basin, through a stock brokerage firm First Resources Management
and Securities Corporation (FRMSC) purchased Class A shares from Oriental
which were fully paid. The shares were listed and traded in the Makati Stock
Exchange.
It turned out that the Oriental shares were owned by Piedras Petroleum
Mining Corporation, a sequestered company controlled by the nominees of
the PCGG.
PCGG sent a letter to Equitable, Orientals stock and transfer agent,
confirming Piedras sale of the Oriental shares in favor of Pacific Basin. In the
same letter, Equitable was requested to record the acquisition and issue
corresponding stock certificates.
Equitable however informed FRMSC that it cannot effect the transfer
because the endorser, a certain Mr. Madarag was not among the authorized
signatories of Piedras and because there was no board resolution
authorizing the sale.
Despite compliance with the requirement imposed by Equitable, the latter
still refused to record the transfer.
This prompted Pacific Basin to file an action for Mandamus.
In their answer, Oriental and Equitable claimed that the governments title
over the subject OPMC shares was based on the cession made by Benedicto
in exchange for immunity from prosecution. The Compromise Agreement
between the government (through PCGG) and Benedicto shows that part of
the assets to be turned over by Mr. Benedicto to the government were all of
the OPMC shares owned by Piedras Petroleum. The Court, however issued a
TRO enjoining the enforcement of the Compromise Agreement. Thus, OPMC
and EBC maintained that the basis for PCGGs claim of title over the OPMC
shares disappeared as the effectivity of the supposed cession made by
Benedicto is suspended.
It was further contended that even assuming that the government had valid
title over the shares, the sale of the same was void for non compliance with
the requirements for the disposition of government owned assets; among
which: non holding of a public bidding.
The SEC Hearing Officer, SEC en banc and CA ruled for Pacific Basin on the
ground that the TRO was rendered moot by the dismissal of the petition and
denial of the MR; and that the sale of shares through public stock exchange
is tantamount to a public bidding.
Pacific Basin appealed the deletion of the award of damages. Oriental and
Equitable likewise questioned the ruling.


Issue:
1. Whether the Oriental shares may be disposed only through public
bidding
2. Whether Oriental may be compelled to record the transfer

Held:
1. NO. Shares originally owned by a sequestered corporation controlled by
the nominees of the PCGG remain to be privately owned until such time
when the court declares that the subject shares were acquired through
68 | P a g e

government funds. The PCGG, as mere conservator, does not
automatically become the owner of a sequestered property in behalf of
the government. There must be a final determination by the courts if
the property is in fact ill-gotten and was acquired by using government
funds. Even on the assumption that the subject shares are government
assets, the sale of the subject shares through the stock exchange is
valid and binding, as there is no law which mandates that listed shares
which are owned by the government be sold only through public
bidding. Thus, the transferee/ assignee has the right to have the stocks
transferred to his name, this being an inherent right flowing from his
ownership of the stocks. The duty of the corporation is ministerial one
and if it refuses to make such transaction without good cause, it may be
compelled to do so by mandamus. The only limitation imposed by
Section 63 of the Corporation Code is when the corporation holds any
unpaid claim against the shares intended to be transferred.
2. YES. PacificBasin satisfied the condition of full payment of the Oriental
shares, this fact was never denied by both Oriental and
Equitable.Therefore, upon PacificBasins full payment of the shares, it
became a ministerial duty on the part of Oriental to record the transfer
in its stock and transfer book and issue new stock certificates in favor of
PacificBasin.
Thus, Orientals and Equitables refusal to record the transfer is violative
of Section 63 of the Corporation Code and Orientals own amended by-
laws.


VALLEY GOLF AND COUNTRY CLUB, INC. v. VDA. DE CARAM
Facts:
The shareholders of Valley Golf were required to pay monthly dues.
Congressman Fermin Caram, who in 1961 subscribed and paid in full one
Golf share in the capital stock of Valley Golf stopped paying his monthly
dues beginning 1980. These dues were continually assessed until 1987.
Valley Golf claimed to have sent notices to the address furnished by Caram.
The third and fourth demand letter was addressed to Est. of Fermin
Caram. The fifth and final notice was addressed to Fermin Caram.
Despite the notices, Caram did not settle his account, this prompted Valley
Golf to sell at a public auction the shares to satisfy its claims.
Pursuant to its by-laws, the Board through a resolution authorized the sale
which was scheduled on June 1987.
However it turned out that Congressman Caram died on October 1986. In
the intestate proceeding, the Golf share was included as part of the estate
which later on was adjudicated to the respondent.
It was only in 1990 that the heirs learned of the sale following their inquiry
about the share. The heirs were subsequently informed that they were
entitled to a refund out of the proceeds of the sale.
Respondent filed an action for reconveyance of the shares with the SEC.
The SEC hearing officer ruled that the shares could not have been
delinquent since the same was fully paid; therefore the sale had no basis in
law and was thus a nullity.
The hearing officer further ruled that provision creating a lien upon shares
of stocks for unpaid debts to the corporation should be embodied in the
AOI and not merely in the by-laws. And lastly, that the non- payment of
monthly dues was merely an ordinary debt enforceable by judicial action.
The SEC en banc and CA affirmed.

Issue:
1. May a non-stock corporation seize and dispose of the membership
share of a fully-paid member on account of its unpaid debts to the
corporation when it is authorized to do so under the corporate by-
laws but not by the Articles of Incorporation?
2. Does the Corporation Code permit the termination of membership
without due notice to the member?
3. In a nutshell, were the actions of Valley Golf concerning the Golf
Share and membership of Caram warranted?

Held:
1. Generally in theory, a non-stock corporation has the power to effect the
termination of a member without having to constitute a lien on the
membership share or to undertake the elaborate process of selling the
same at public auction. The articles of incorporation or the by-laws can
very well simply provide that the failure of a member to pay the dues on
time is cause for the board of directors to terminate membership. Yet
Valley Golf was organized in such a way that membership is adjunct to
ownership of a share in the club; hence the necessity to dispose of the
share to terminate membership.
69 | P a g e

Share ownership introduces another dimension to the casethe reality
that termination of membership may also lead to the infringement of
property rights. Even though Valley Golf is a non-stock corporation, as
evinced by the fact that it is not authorized to distribute to the holder of
its shares dividends or allotments of the surplus profits on the basis of
shares held, the Golf Share has an assigned value reflected on the
certificate of membership itself. Termination of membership in Valley
Golf does not merely lead to the withdrawal of the rights and privileges
of the member to club properties and facilities but also to the loss of
the Golf Share itself for which the member had fully paid.
The claim of Valley Golf is limited to the amount of unpaid dues plus
incremental costs. On the other hand, Carams loss may encompass not
only the amount he had paid for the share but also the price it would
have fetched in the market at the time his membership was terminated.
A simple remedy would be to provide in the by-laws refund mechanism,
but this was not provided in Clubs by-laws.
Although the Code itself is silent on that matter, and the argument can
be made that if no notice is provided for in the articles of incorporation
or in the by-laws, then termination may be effected without any notice
at all, the setup is to the extreme detriment to the member. It is
important to note that purchase of an ownership share is a condition
sine qua non to acquire membership in Valley Golf, a non stock
corporation. Therefore, membership in Valley Golf entails the
acquisition of a property right because the loss of membership in a non-
stock corporation also entails the loss of property rights.
2. NO. The arrangement provided for in the afore-quoted by-laws of Valley
Golf whereby a lien is constituted on the membership share to answer
for subsequent obligations to the corporation finds applicable parallels
under the Civil Code. Membership shares are considered as movable or
personal property, and they can be constituted as security to secure a
principal obligation, such as the dues and fees. There are at least two
contractual modes under the Civil Code by which personal property can
be used to secure a principal obligation. The first is through a contract
of pledge, while the second is through a chattel mortgage. A pledge
would require the pledgor to surrender possession of the thing pledged,
i.e., the membership share, to the pledge in order that the contract of
pledge may be constituted.


Is delivery of the share cannot be effected, the suitable security
transaction is the chattel mortgage. Under Article 2124 of the Civil
Code, movables may be the object of a chattel mortgage. The Chattel
mortgage is governed by Act No. 1508, otherwise known The Chattel
Mortgage Law, and the Civil Code.
3. In this case, Caram had not signed any document that manifests his
agreement to constitute his Golf Share as security in favor of Valley Golf
to answer for his obligations to the club. There is no document we can
assess that it is substantially compliant with the form of chattel
mortgages under Section 5 of Act No. 1508. The by-laws could not
suffice for that purpose since it is not designed as a bilateral contract
between Caram and Valley Golf, or a vehicle by which Caram expressed
his consent to constitute his Golf Share as security for his account with
Valley Golf.
Note:
SEC. 91. Termination of membership.Membership shall be terminated in
the manner and for the causes provided in the articles of incorporation or
the by-laws. Termination of membership shall have the effect of
extinguishing all rights of a member in the corporation or in its property,
unless otherwise provided in the articles of incorporation or the by-laws.
(Emphasis supplied)

CALATAGAN GOLF CLUB INC. v. CLEMENTE JR.
Facts:
Clemente applied to purchase one share of stock of Calatagan,
paying P120,000.00 for the share.
Calatagan charges monthly dues on its members to meet expenses for
general operations, as well as costs for upkeep and improvement of the
grounds and facilities. The provision on monthly dues is incorporated in
Calatagans Articles of Incorporation and By-Laws. It is also reproduced at
the back of each certificate of stock.
When Clemente became a member he was charged with monthly dues. He
paid twice then he ceased paying the dues.
70 | P a g e

Calatagan declared Clemente delinquent for having failed to pay his
monthly dues. Calatagan BOD adopted a resolution authorizing the
foreclosure of shares of delinquent members, including Clementes; and the
public auction of these shares.
Calatagan sent a letter to Clemente warning that unless he settles his
outstanding dues, his share would be included among the delinquent shares
to be sold at public auction. the letter was sent to Clementes mailing
address that had already been closed.
A notice of auction sale was posted on the Clubs bulletin board, as well as
on the clubs premises.
The auction sale took place as scheduled and Clementes share was sold.
Clemente learned of the sale of his share. He filed a claim with the SEC
seeking the restoration of his shareholding in Calatagan with damages.
SEC rendered a decision dismissing Clementes complaint.

Issue:
Whether or not OPMC can refuse registration of the sale in the books on the
corporation on the ground that Piedras Petroleum is under sequestration by
the PCGG?

Held:
The 6 months limitation to file an action to nullify the sale of delinquent
shares under section 69 applies only to sale of delinquent stocks due to
nonpayment of subscription price for the share of the stock itself. In case of
termination of membership in a non-stock corporation, due to the
nonpayment of dues of the member, the grounds and procedures for
membership termination under the articles or by-laws should apply. It must
also conform to the requirements of substantial justice. The Corporation is
clearly in bad faith when it sent the notices of sale to the postal office box of
the stockholder knowing fully well that the box had already been closed

CORPORATE RECORDS

Republic v. Sandiganbayan
Facts:
Eduardo Cojuangco, Jr requested the San Miguel Corporation (SMC) and its
corporate secretary the production, inspection, examination/verification
and/or photocopying of the SMC corporate records to inform him of the
decisions, policies, acts and performance of the management of the SMC
under the PCGG-Board.
Subsequently, private respondent was informed by the SMC that all
requests for the examination, inspection and photocopying of its corporate
records should be coursed through the PCGG. The request of private
respondent for the inspection/examination of SMC's corporate records was
denied by the PCGG. As regards the corporate records of URPB, private
respondent was likewise advised to course his request through the PCGG
Thereafter, private respondent filed two separate petitions for prohibition
and mandamus before the Sandiganbayan seeking to enforce his
stockholder's right to inspect the corporate records of SMC and the UCPB.
Sandiganbayan issued an order allowing respondent Eduardo Cojuangco, Jr.,
to inspect the corporate records of United Coconut Planters Bank. Hence
this present petition.

Issue:
Whether private respondent-stockholder may examine the corporate
records

Held:
YES. In the case at bar, petitioner failed to discharge the burden of proof to
show that private respondent's action in seeking examination of the
corporate records was moved by unlawful or ill-motivated designs which
could appropriately call for a judicial protection against the exercise of such
right.

Bitong v. Court of Appeals
Held:
Books and records of a corporation which include even the stock and
transfer book are generally admissible in evidence in favor of or against the
corporation and its members to prove the corporate acts, its financial status
and other matters including one's status as a stockholder. They are
ordinarily the best evidence of corporate acts and proceedings.
However, the books and records of a corporation are not conclusive even
against the corporation but are prima facie evidence only. Parol evidence
may be admitted to supply omissions in the records, explain ambiguities, or
71 | P a g e

show what transpired where no records were kept, or in some cases where
such records were contradicted. The effect of entries in the books of the
corporation which purport to be regular records of the proceedings of its
board of directors or stockholders can be destroyed by testimony of a more
conclusive character than mere suspicion that there was an irregularity in
the manner in which the books were kept.
The foregoing considerations are founded on the basic principle that stock
issued without authority and in violation of law is void and confers no rights
on the person to whom it is issued and subjects him to no liabilities. Where
there is an inherent lack of power in the corporation to issue the stock,
neither the corporation nor the person to whom the stock is issued is
estopped to question its validity since an estopped cannot operate to create
stock which under the law cannot have existence.


APPRAISAL RIGHT

Boman Environmental Development Corp. v. Court of Appeals
Facts:
Fajilan offered in writing to resign as President and Member of the Board of
Directors of Boman Environmental Development Corporation (BEDECO),
and to sell to the company all his shares, rights, and interests therein.
Fajilan's resignation as president was accepted and new officers were
elected. Fajilan's offer to sell his shares back to the corporation was
approved, the Board promising to pay for them on a staggered basis.
However, BEDECO defaulted in paying the balance of P200,000. Fajilan filed
a complaint for collection of that balance from BEDECO.
The trial court dismissed the complaint for lack of jurisdiction. It ruled that
the controversy arose out of intracorporate relations, hence, the SEC has
original and exclusive jurisdiction to hear and decide it. CA reversed.

Issue:
Whether a suit brought by a withdrawing stockholder against the
corporation to enforce payment of the balance due on the consideration
the surrender of his shares of stock and interests in the corporation,
involves an intra-corporate dispute.

Held:
YES. This case involves an intra-corporate controversy because the parties
are a stockholder and the corporation. As correctly observed by the trial
court, the perfection of the agreement to sell Fajilan's participation and
interests in BEDECO and the execution of the promissory note for payment
of the price of the sale did not remove the dispute from the coverage of
Section 5(b) of P.D. No. 902, as amended, for both the said agreement and
the promissory note arose from intra-corporate relations. Fajilan's offer to
resign as president and director "effective as soon as my shares and
interests thereto (sic) are sold and fully paid implied that he would remain a
stockholder until his shares and interests were fully paid for, for one cannot
be a director or president of a corporation unless he is also a stockholder
thereof. The fact that he was replaced as president of the corporation did
not necessarily mean that he ceased to be a stockholder considering how
the corporation failed to complete payment of the consideration for the
purchase of his shares of stock and interests in the goodwill of the business.
There has been no actual transfer of his shares to the corporation. In the
books of the corporation he is still a stockholder.

Note:
Intracorporate controversies are now cognizable by the Regional Trial Court
acting as a special commercial court


MERGERS

Edward J. Nell Co. v. Pacific Farms
Facts:
Edward J. Nell Company obtained a favourable judgment against Insular
Farms, Inc. representing the unpaid balance of the price of a pump sold by
appellant to Insular Farms. Howevr, the writ of execution was returned
unsatisfied, stating that Insular Farms had no leviable property.
Thereafter, appellant filed before the municipal court the present action
against Pacific Farms, Inc. for the collection of the judgment
aforementioned, upon the theory that appellee is the alter ego of Insular
Farms, because the former had purchased all or substantially all of the
shares of stock, as well as the real and personal properties of the latter,
including the pumping equipment sold by appellant to Insular Farms.
72 | P a g e

The municipal court rendered judgment dismissing appellant's complaint.
Appellant appealed, with the same result, to the court of first instance and,
subsequently, to the Court of Appeals. Hence this appeal by certiorari.

Issue:
Whether or not Pacific Farms is liable for the debt of Insular Farms to the
appellee in this case?

Held:
Generally where one corporation sells or otherwise transfers all of its assets
to another corporation, the latter is not liable for the debts and liabilities of
the transferor, except:
1. where the purchaser expressly or impliedly agrees to
assume such debts;
2. where the transaction amounts to a consolidation or
merger of the corporations;
3. where the purchasing corporation is merely a continuation
of the selling corporation; and
4. where the transaction is entered into fraudulently in order
to escape liability for such debts.

In the case at bar, there is neither proof nor allegation that appellee had
expressly or impliedly agreed to assume the debt of Insular Farms in favor of
appellant herein, or that the appellee is a continuation of Insular Farms, or
that the sale of either the shares of stock or the assets of Insular Farms to
the appellee has been entered into fraudulently, in order to escape liability
for the debt of the Insular Farms. Appellee purchased the shares of stock of
Insular Farms as the highest bidder at an auction sale held at the instance of
a bank to which said shares had been pledged as security for an obligation
of Insular Farms in favor of said bank.
Neither is it claimed that these transactions have resulted in the
consolidation or merger of the Insular Farms and appellee herein. On the
contrary, appellant's theory to the effect that appellee is an alter ego of the
Insular Farms negates such consolidation or merger, for a corporation
cannot be its own alter ego.


Associated Bank v. Court of Appeals
Facts:
Associated Banking Corporation and Citizens Bank and Trust Company
merged to form just one banking corporation known as Associated Citizens
Bank, the surviving bank. The Associated Citizens Bank changed its
corporate name to Associated Bank by amending its AOI
Sarmiento, Jr., executed in favor of Citizens Bank a promissory note. The
defendant, to date, still owes plaintiff bank the amount exclusive of interest
and other charges. Despite repeated demands the defendant failed to pay
the amount due.
In his answer Sarmiento, Jr averred that the complaint states no valid cause
of action and that the plaintiff is not the proper party in interest because
the promissory note was executed in favor of Citizens Bank and Trust
Company.
The trial court ruled in favor of the Associated Bank.
The appellate court however, held that the Associated Bank had no cause of
action against Lorenzo Sarmiento Jr., since said bank was not privy to the
promissory note executed by Sarmiento in favor of Citizens Bank and Trust
Company. The court ruled that the earlier merger between the two banks
could not have vested Associated Bank with any interest arising from the
promissory note executed in favor of CBTC after such merger.
Hence this present petition.

Issue:
Whether Associated Bank, the surviving corporation, may enforce the
promissory note made by private respondent in favor of CBTC, the absorbed
company, after the merger agreement had been signed.

Held:
YES. In the merger of two or more existing corporations, one of the
combining corporations survives and continues the combined business,
while the rest are dissolved and all their rights, properties and liabilities are
acquired by the surviving corporation. Although there is a dissolution of the
absorbed corporations, there is no winding up of their affairs or liquidation
of their assets, because the surviving corporation automatically acquires all
their rights, privileges and powers, as well as their liabilities.


PNB v. Andrada Electric and Engineering Co.
73 | P a g e

Facts:
Pampanga Sugar Mills (PASUMIL) failed to pay Andrada Electric
&Engineering Company for electrical rewinding and repair, and construction
of certain improvements made by the latter.
Subsequently, PNB acquired the assets of the defendant PASUMIL that were
earlier foreclosed by the Development Bank of the Philippines.
Andrada Electric besought PNB to pay the outstanding obligation of the
defendant PASUMIL, inasmuch as PNB now owned and possessed the assets
of the PASUMIL. However, PNB refused to pay.
Thus, Andrada Electric filed before the trial court a collection case against
PNB, claiming that it should be held liable for the unpaid obligations of
PASUMIL by virtue of LOI Nos. 189-A and 311, which expressly authorized
PASUMIL and PNB to merge or consolidate, among others.
On the other hand, PNB contends that their takeover of the operations of
PASUMIL did not involve any corporate merger or consolidation, because
the latter had never lost its separate identity as a corporation.
The trial court rendered a decision holding PNB liable for the debt of
PASUMIL to Andrada Electric. On appeal, the CA affirmed said decision.
Hence this petition

Issue:
Whether or not there was a merger between PNB and PASUMIL

Held:
NO. In the case at bar, we hold that there is no merger or consolidation with
respect to PASUMIL and PNB. The procedure prescribed under Title IX of the
Corporation Code was not followed.
In fact, PASUMILs corporate existence, had not been legally extinguished or
terminated. Neither did petitioner expressly or impliedly agree to assume
the debt of PASUMIL to respondent. LOI No. 11 explicitly provides that PNB
shall study and submit recommendations on the claims of PASUMILs
creditors. Clearly, the corporate separateness between PASUMIL and PNB
remains, despite respondents insistence to the contrary.

Note:
A consolidation is the union of two or more existing entities to form a new
entity called the consolidated corporation. A merger, on the other hand, is a
union whereby one or more existing corporations are absorbed by another
corporation that survives and continues the combined business.
The merger, however, does not become effective upon the mere agreement
of the constituent corporations. Since a merger or consolidation involves
fundamental changes in the corporation, as well as in the rights of
stockholders and creditors, there must be an express provision of law
authorizing them. For a valid merger or consolidation, the approval by the
Securities and Exchange Commission (SEC) of the articles of merger or
consolidation is required. These articles must likewise be duly approved by a
majority of the respective stockholders of the constituent corporations.

Babst v. Court of Appeals
Facts:
Elizalde Steel Consolidated Inc. (ELISCON) failed to pay the full amount of
loan it owed to Commercial Bank and Trust Company (CBTC).
Chester G. Babst executed a Continuing Suretyship, whereby he bound
himself jointly and severally liable to pay any existing indebtedness of
MULTI to CBTC to the extent of.
BPI and CBTC entered into a merger, wherein BPI, as the surviving
corporation, acquired all the assets and assumed all the liabilities of CBTC.
Meanwhile, ELISCON encountered financial difficulties and became heavily
indebted to DBP. In order to settle its obligations, DBP formally took over
the assets of ELISCON, including its indebtedness to BPI. Thereafter, DBP
proposed formulas for the settlement of all of ELISCON's obligations to its
creditors, but BPI expressly rejected the formula submitted to it for not
being acceptable.
Consequently, BPI instituted with the Regional Trial Court of Makati
complaint for sum of money against ELISCON, MULTI and Babst.
ELISCON and MULTI assail BPI's legal capacity to recover their obligation to
CBTC based on novation on the part of the creditor.

Issue:
Whether or not BPI can collect payment from the defendants

Held:
YES. A bank which merged with another bank can sue a debtor of the
absorbed bank because it acquired the rights of the latter. Novation
(because of the change of creditor) is not a valid defense because it is
74 | P a g e

settled that in a merger of two existing corporations, one of the
corporations survives and continues the business, while the other is
dissolved and all its rights, properties and liabilities are required by the
surviving corporation.


Omictin v. Court of Appeals
Facts:
A criminal case for estafa was filed against Lagos, on the ground that he
refused to return two company vehicles entrusted to him while he was still
president of Saag Philippines Inc.
Lagos filed a motion to suspend proceedings on the basis of a prejudicial
question because of a pending petition with the SEC involving the same
parties. It appears that Lagos filed an SEC Case for the declaration of nullity
of the appointment of Omictin as Operations Manager Ad Interim of Saag
Phils., Inc., among others.
The trial court denied the motion to suspend proceedings.
His motion for reconsideration having been denied by the trial court,
respondent filed with the CA the petition for certiorari assailing the
aforesaid orders.
The CA rendered held that a prejudicial question exists which calls for the
suspension of the criminal proceedings before the lower court.
Hence this petition.

Issue:
Whether or not a prejudicial question exists to warrant the suspension of
the criminal proceedings pending the resolution of the intra-corporate
controversy that was originally filed with the SEC?

Held:
YES. , the resolution of the issues raised in the intra-corporate dispute will
determine the guilt or innocence of private respondent in the crime of
estafa filed against him by petitioner before the RTC of Makati. As correctly
stated by the CA, one of the elements of the crime of estafa with abuse of
confidence under Article 315, par. 1(b) of the Revised Penal Code is a
demand made by the offended party to the offender.
Logically, under the circumstances, since the alleged offended party is Saag
Phils., Inc., the validity of the demand for the delivery of the subject vehicles
rests upon the authority of the person making such a demand on the
companys behalf. Private respondent is challenging petitioners authority to
act for Saag Phils., Inc. in the corporate case pending before the RTC of
Mandaluyong, Branch 214. Taken in this light, if the supposed authority of
petitioner is found to be defective, it is as if no demand was ever made,
hence, the prosecution for estafa cannot prosper.

Note:
A prejudicial question is defined as that which arises in a case, the
resolution of which is a logical antecedent of the issue involved therein and
the cognizance of which pertains to another tribunal. Here, the case which
was lodged originally before the SEC involves facts that are intimately
related to those upon which the criminal prosecution is based.


McLeod v. NLRC
Facts:
McLeod filed a complaint for retirement benefits, vacation and sick leave
benefits, among others against Filipinas Synthetic Corporation (FILSYN), Far
Eastern Textile Mills, Inc.(FETMI), Sta. Rosa Textiles (PEGGY MILLS), Inc.,
Patricio Lim and Eric Hu. He claimed alleged that that in 1991, Filsyn sold
Peggy Mills, Inc. to FETMI as per agreement and this was renamed as Sta.
Rosa Textile with Patricio Lim as Chairman and President
Private respondents (FILSYN, FETMI and PEGGY MILLS) alleged that except
for Peggy Mills, the other respondents are not proper persons in interest
due to the lack of employer-employee relationship between them and
complainant.
LA held all respondents as jointly and solidarily liable for complainants
money claims.
All respondents appealed to the NLRC, which held that only Peggy Mills, Inc.
is liable. McLeod filed a motion for reconsideration which the NLRC denied.
McLeod thus filed a petition for certiorari before the CA assailing the
decision and resolution of the NLRC. The CA affirmed with modification the
decision of the NLRC, and included Patricio Lim as jointly and severally liable
with Peggy Mills.

Issue:
75 | P a g e

Whether or not the sale of assets in by one corporation to another amounts
to merger or consolidation

Held:
NO. What took place between PMI and SRTI was dation in payment with
lease. Here, PMI transferred its assets to SRTI to settle its obligation to SRTI.
There was no proof that PMI fraudulently transferred these assets to escape
its liability for any of its debts.
There was no merger or consolidation of PMI and SRTI.

Note:
Consolidation is the union of two or more existing corporations to form a
new corporation called the consolidated corporation. It is a combination by
agreement between two or more corporations by which their rights,
franchises, and property are united and become those of a single, new
corporation, composed generally, although not necessarily, of the
stockholders of the original corporations.
Merger, on the other hand, is a union whereby one corporation absorbs one
or more existing corporations, and the absorbing corporation survives and
continues the combined business.
The parties to a merger or consolidation are called constituent corporations.
In consolidation, all the constituents are dissolved and absorbed by the new
consolidated enterprise. In merger, all constituents, except the surviving
corporation, are dissolved. In both cases, however, there is no liquidation of
the assets of the dissolved corporations,





Spouses Nisce v. Equitable PCI Bank
Facts:
In 1994, the Equitable Banking Corporation and the PCIB were merged
under the corporate name Equitable PCI Bank.
Equitable PCI Bank as creditor-mortgagee filed a petition for extrajudicial
foreclosure of the real estate mortgage contracts executed by the spouses
Ramon and Natividad Nisce over several parcels of land as security for the
oblgation of the latter to the former.
The Nisce spouses filed a complaint for "nullity of the Suretyship
Agreement, damages and legal compensation". They alleged that in a letter
they had requested the bank to setoff the peso equivalent of their
obligation against their US dollar account with PCI Capital Asia Limited, a
fully-owned subsidiary in Hong Kong, which was accepted by the bank.
In its Answer, the Bank alleged that the spouses had no cause of action for
legal compensation since PCI Capital was a different corporation with a
separate and distinct personality.
The RTC granted the spouses Nisces plea for a writ of preliminary injunction
upon payment of a bond.
The Bank opted not to file a motion for reconsideration of the order, and
instead assailed the trial courts order before the CA via petition for
certiorari under Rule 65 of the Rules of Court. The CA nullified the assailed
order of the RTC.
MR was denied. Hence this petition.

Issue:
Whether or not legal compensation is proper in this case?

Held:
NO. when an investor has a claim against a subsidiary of another
corporation which subsequently became the acquired corporation in a
merger, the claim against the subsidiary cannot be enforced against the
surviving corporation even though the latter corporation by virtue of the
merger acquired all the shares of the absorbed corporation. This is because
the fact that a corporation owns almost all the stocks of another
corporation, taken alone, is not sufficient to justify their being treated as
one entity.




NON STOCK PROFIT CORPORATION

Litonjua v. Court of Appeals
Facts:
Antonio Litonjua is an Associate Member of Wack Wack Golf and Country
Club.
76 | P a g e

In January 1985, Wack Wack posted the monthly list of delinquent members
on its premises which included Antonio Litonjua. He proceeded to the
Cashier's Office of the club and was informed about his failure to pay his
November 1984 dues.
Antonio Litonjua was able to convince the auxiliary clerks in the Cashier's
Office to delete his name from the list of delinquent members.
Consequently, Antonio Litonjua continued to avail of the club facilities.
However, Antonio Litonjua received a letter from the General Manager of
Wack Wack, informing him of the Membership Committee's decision to
suspend him for a period of sixty days for violating Sec. 34(d) of the club's
by-laws when he availed of club privileges while listed as a delinquent
member.
In March 1985, petitioner sent a letter to Wack Wack demanding restitution
in the sum of P5M for their wrongful and arbitrary suspension.
When Wack Wack failed to respond, petitioners filed a complaint with the
SEC for the nullification of their suspension and for actual, moral and
exemplary damages.
SEC Hearing Officer rendered its decision in favor of petitioners.
SEC en banc affirmed the findings of Hearing Officer but reduced the
amount of damages awarded.
CA reversed the order of the SEC en banc and upholding the suspension of
petitioners.
Petitioners' MR was denied for lack of merit. Hence, this petition.

Issue:
Whether or not the Golf Club can suspend petitioner from the use of its
facilities

Held:
YES. A member of a country club who failed to pay his monthly dues on time
can be suspended. It was under false pretenses that Antonio Litonjua
managed to have his name deleted from the list of delinquent members the
same has no force and effect. Consequently, being a delinquent member in
the posted list and having used club facilities while posted as such, the
imposition of suspension was valid and legal.

Long v. Basa
Facts:
A religious group known as "The Church In Quezon City was organized as
"an entity of the brotherhood in Christ."

The members of the CHURCH vested upon the Board of Directors the
absolute power "to preserve and protect their faith" and to admit and expel
a member of the CHURCH.
The procedure for the expulsion of an erring or dissident member is
prescribed in Article VII (paragraph 4) of the CHURCH By-laws.
The BOD observed that certain members of the CHURCH, including
petitioners herein, exhibited conduct which was dishonorable, improper
and injurious to the character and interest of the CHURCH. The
respondents, as members of the Board of Directors, and some responsible
members of the CHURCH, advised the petitioners "to correct their ways"
and reminded them "that under the By-laws, this organization is only for
worshipping the true God, not to worship Buddha or men."

But petitioners ignored these repeated admonitions. Consequently, the
Board of Directors, during the regular meeting removed certain members
including petitioner from the list of members.
Aggrieved petitioner filed a complaint before SEC seeking the annulment of
the new membership list and the reinstatement of the original list on the
ground that the expulsion was made without prior notice and hearing.
SEC hearing officer ruled in favor of the CHURCH. SEC en banc set aside the
expulsion. Court of Appeals reinstated the SEC hearing officers decision.
Hence this present petition.

Issue:
Whether or not the expulsion of petitioners from the membership of the
CHURCH by its Board of Directors through a resolution is in accordance with
law

Held:
YES. A member of a religious corporation who commits any causes for
expulsion under its by- laws may be expelled by the board of directors
through a resolution, even without giving that erring member any notice
prior to his expulsion, if so provided for in the by laws. It may sound unusual
and objectionable as there is no requirement of prior notice but that is how
peculiar the nature of a religious corporation is vis- a vis an ordinary
corporation organized for profit. The basis of the relationship between a
religious corporation and its members is the latters adherence to a
77 | P a g e

common religious or spiritual belief. Once this basis cease, membership in
the corporation also ceases.

CLOSE CORPORATION

Manuel R. Dulay Ent., Inc. v. Court of Appeals
Facts:
Petitioner corporation through its president, Manuel Dulay, by virtue of a
Board Resolution sold the subject property to spouses Veloso.
Veloso, without the knowledge of Manuel Dulay, mortgaged the subject
property to Manuel Torres as security for a loan.
Veloso failed to pay and so the mortgage was forclosed and was sold to
Torres.
Torres filed an Affidavit of Consolidation of Ownership as neither Veloso nor
Manuel Dulay was able to redeem the subject property
Petitioner corporation filed an action for the cancellation of the Certificate
of Sheriff's Sale and TCT alleging that the sale of the subject property
between spouses Veloso and Manuel Dulay has no binding effect on
petitioner corporation as the Board Resolution which authorized the sale of
the subject property was resolved without the approval of all the members
of the board of directors.
The trial court rendered a decision in favor of private respondents.
CA affirmed.
Petitioners MR was denied.

Issue:
Whether or not a board resolution is a requirement before the president of
a close corporation can do corporate acts?



Held:
NO. In the instant case, petitioner corporation is classified as a close
corporation and consequently a board resolution authorizing the sale or
mortgage of the subject property is not necessary to bind the corporation
for the action of its president. At any rate, corporate action taken at a board
meeting without proper call or notice in a close corporation is deemed
ratified by the absent director unless the latter promptly files his written
objection with the secretary of the corporation after having knowledge of
the meeting.
Consequently, petitioner corporation is liable for the act of Manuel Dulay
and the sale of the subject property to private respondents by Manuel
Dulay is valid and binding.


DISSOLUTION/ LIQUIDATION

Mambulao Lumber Company v. PNB
Facts:
Mambulao applied a loan with PNB. As security, the plaintiff mortgaged a
parcel of land together with the buildings and improvements as well as
other machineries and equipments, situated in Mambulao, Camarines
Norte.
On 1956, PNB released portions of the loan, with the agreement that the
same will be paid in installments from 1957- 1961. Demands were made to
settle the obligation, but the plaintiff failed to settle its account. Upon
investigation, it was discovered that plaintiff had already stopped
operations about the end of 1957 or early part of 1958.
This discovery prompted PNB to move for foreclosure.
1961 or three years after, notice of public auction sale, which was to be held
on November 1961, was made, as a response the plaintiff sent letters
protesting against the foreclosures of the real estate and chattel mortgages
on the ground that they could not be effected unless the Court issued an
order for its foreclosure and that the foreclosure proceeding according to
the terms of the mortgage contracts, should be made in Manila.
Moreover, in the said letter, the plaintiff stated that if the public auction
sale was suspended and the plaintiff would be given 90 days extension, it
would settle its obligation fully, because a negotiation was on going for the
sale of its whole interest for an amount more than sufficient to settle such
obligation.
PNB construed the letter as a request for extension of the foreclosure sale.
PNB advised the Sheriff to defer the sale until December. The foreclosure
sale of the land however was held on its original date, November 1961 with
PNB as the highest bidder.
78 | P a g e

In a letter dated December 14, 1961 plaintiff sent a bank draft to PNB to
settle its obligation; in the letter plaintiff reiterated its request to have the
foreclosure sale discontinued as the indebtedness had been fully satisfied.
Plaintiff also notified the sheriff that it already paid its obligation in full.
The attorney of the bank acknowledged the remittance but stated that the
amount remitted was not enough to cover the balance of the obligation
which includes charges and other expenses.
The chattels were sold on December 1961, with PNB as the buyer, the latter
sold the same to Mariano Bundok.
The chattels were forcibly taken from the compound of the plaintiff.

Issue:
Whether the foreclosure sale was valid after the corporation stopped its
operations

Held:
NO. The amount remitted to PNB was more than sufficient to liquidate its
total obligation to the bank; moreover mortgage agreement was not
followed by the bank (place were foreclosure is to be held).
OLD RULE:
In the absence of statutory provision to the contrary, pending actions by or
against a corporation are abated upon expiration of the period allowed by
law for the liquidation of its affairs.
It is generally held, that where a statute continues the existence of a
corporation for a certain period after its dissolution for the purpose of
prosecuting and defending suits, etc., the corporation becomes defunct
upon the expiration of such period, at least in the absence of a provision to
the contrary, so that no action can afterwards be brought by or against it,
and must be dismissed. Actions pending by or against the corporation when
the period allowed by the statute expires, ordinarily abate.







National Abaca and Fibers Corporation v. Pore
Facts:
1953, Abaca filed a complaint against Pore for the recovery of a sum of
money advanced to her for the purchase of hemp. It was alleged that Pore
failed to account such.
1956, the court rendered judgment against Pore ordering her to pay Abaca
with legal interest.
Pore appealed to the CFI, in which he moved for the dismissal of the
complaint on the ground that plaintiff has no legal capacity to sue because
plaintiff corporation has already been abolished by virtue of EO 372 dated
November 24, 1950. Plaintiff objected thereto upon the ground that
pursuant to said executive order, plaintiff "shall nevertheless be continued
as a body corporate for a period of three (3) years from the effective date"
of said executive order, which was November 30, 1950, "for the purpose of
prosecuting and defending suits by or against it and of enabling the Board of
Liquidators" thereby created "gradually to settle and close its affairs", .
. . and that this case was begun on November 14, 1953, or before the
expiration of the period aforementioned. After due hearing, the court of
first instance issued an order dated August 1, 1956, directing plaintiff to
amend the complaint, within ten (10) days from notice, by including the
Board of Liquidators as co-party plaintiff, with the admonition that
otherwise the case would be dismissed.
The amendment required by the court was not made, hence the complaint
was dismissed.

Issue:
Whether an action, commenced within three (3) years after the abolition of
plaintiff, as a corporation, may be continued by the same after the
expiration of said period

Held:
NO. The rule appears to be well settled that, in the absence of statutory
provision to the contrary, pending actions by or against a corporation are
abated upon expiration of the period allowed by law for the liquidation of
its affairs.
It is generally held, that where a statute continues the existence of a
corporation for a certain period after its dissolution for the purpose
of prosecuting and defending suits, etc., the corporation becomes
defunct upon the expiration of such period, at least in the absence
79 | P a g e

of a provision to the contrary, so that no action can afterwards be
brought by or against it, and must be dismissed. Actions pending by
or against the corporation when the period allowed by the statute
expires, ordinarily abate.
. . . This time limit does not apply unless the circumstances are such
as to bring the corporation within the provision of the statute.
However, the wording of the statutes, in some jurisdictions
authorize suits after the expiration of the time limit, where the
statute provides that for the purpose of any suit brought by or
against the corporation shall continue beyond such period for a
further named period after final judgment. (Fletcher's Cyclopedia on
Corporations, Vol. 16, pp. 892-893.).
Our Corporation Law contains no provision authorizing a corporation, after
three (3) years from the expiration of its lifetime, to continue in its
corporate name actions instituted by it within said period of three (3) years.
in fact, section 77 of said law provides that the corporation shall "be
continued as a body corporate for three (3) years after the time when it
would have been . . . dissolved, for the purposed of prosecuting and
defending suits by or against it . . .", so that, thereafter, it shall no longer
enjoy corporate existence for such purpose. For this reason, section 78 of
the same law authorizes the corporation, "at any time during said three
years . . . to convey all of its property to trustees for the benefit of
members, stockholders, creditors and other interested", evidently for the
purpose, among others, of enabling said trustees to prosecute and defend
suits by or against the corporation begun before the expiration of said
period. Hence, commenting on said sections, Judge Fisher, in his work
entitled Philippines Law on Stock Corporations (1929 ed.), has the following
to say:
It is to be noted that the time during which the corporation, through
its own officers, may conduct the liquidation of its assets and sue
and be sued as a corporation is limited to three years from the time
the period of dissolution commences; but that there is no time
limited within the trustees must complete a liquidation placed in
their hands. It is provided only (Corp. Law, Sec. 78) that the
conveyance to the trustees must be made within the three-year
period. It may be found impossible to complete the work of
liquidation within the three-year period or to reduce disputed
claims to judgment. The authorities are to the effect that suits by or
against a corporation abate when it ceased to be an entity capable
of suing or being sued (7 R.C.L. Corps., Par. 750); but trustees to
whom the corporate assets have been conveyed pursuant to the
authority of section 78 may used and be sued as such in all matters
connected with the liquidation. By the terms of the statute the
effect of the conveyance is to make the trustees the legal owners of
the property conveyed, subject to the beneficial interest therein of
creditors and stockholders. (pp. 389-390; see also Sumera v.
Valencia [67 Phil. 721, 726-727).
Obviously, the complete loss of plaintiff's corporate existence after the
expiration of the period of three (3) years for the settlement of its affairs is
what impelled the President to create a Board of Liquidators, to continue
the management of such matters as may then be pending.
Gelano v. Court Of Appeals
GR NO L- 39050 February 24, 1981
De Castro, J.:

Facts:
Insular Sawmill Inc was engaged in general lumber and sawmill business. To
carry on the business, Insular leased the paraphernal property of petitioner
wife Guillermina for 1,200 a month. It was the husband Carlos who received
from the corporation case advances on account of the rentals.
Between November 1947 to December 1950, Gelano obtained from Insular
cash advances. The said sum was received by him on the agreement that
Insular could deduct the same from the monthly rentals of the leased
premises until said cash advances are fully paid.
Gelano failed to pay the entire amount; he refused to pay the balance
despite repeated demands, on the ground that said amount was for the
personal account of her husband asked for by, and given to him, without
her knowledge and consent and did not benefit the family.
80 | P a g e

Sometime from 1948- 1949, Spouses Gelano made credit purchases of
lumber materials from Insular which was used for the repair of their house.
Partial payment was made, however the spouses failed to pay the balance.
In 1952, Insular through Joseph Yu executed joint and several PNs to
accommodate and help the Carlos renew loans obtained by him from the
China Banking Corporation.
Carlos failed to pay, the bank then debited the amount from the
corporations current account with the bank. Carlos was unable to pay the
entire obligation, his wife refused to pay on the ground that she had no
knowledge about the accommodation made by the corporation in favor of
her husband.
1959 the corporation filed a case for collection against the Spouses.
While the case was pending, Insular amended its AOI, shortening its
corporate term up to December 31, 1960 only. The trial court was not
notified of the amendment shortening the corporate existence and no
substitution of party was ever made.
On November 20, 1964 and almost four (4) years after the dissolution of the
corporation, the trial court rendered a decision in favor of Insular.
After petitioners received a copy of the decision, they came to know that
the Insular Sawmill Inc. was dissolved way back on December 31, 1960.
Hence, petitioners filed a motion to dismiss the case and/or reconsideration
of the decision of the Court of Appeals on grounds that the case was
prosecuted even after dissolution of private respondent as a corporation
and that a defunct corporation cannot maintain any suit for or against it
without first complying with the requirements of the winding up of the
affairs of the corporation and the assignment of its property rights within
the required period.

Issue:
whether a corporation, whose corporate life had ceased by the expiration of
its term of existence, could still continue prosecuting and defending suits
after its dissolution and beyond the period of three years provided to wind
up its affairs, without having undertaken any step to transfer its assets to a
trustee or assignee.

Held:
The complaint in this case was filed on May 29, 1959 when private
respondent Insular Sawmill, Inc. was still existing. While the case was being
tried, the stockholders amended its Articles of Incorporation by shortening
the term of its existence from December 31, 1995 to December 31, 1960,
which was approved by the Securities and Exchange Commission.
Under Section 77 of the Corporation Law, when the corporate existence is
terminated in any legal manner, the corporation shall nevertheless continue
as a body corporate for three (3) years after the time when it would have
been dissolved, for the purpose of prosecuting and defending suits by or
against it. According to authorities, the corporation "becomes incapable of
making contracts or receiving a grant. It does not, however, cease to be a
body corporate for all purposes." In the case of Pasay Credit and Finance
Corp. vs. Isidro Lazaro and others, 46 OG (11) 5528, this Court held that "a
corporation may continue a pending 'litigation even after the lapse of the 3-
year period granted by Section 77 of Act 1459 to corporation subsequent to
their dissolution to continue its corporate existence for the purpose of
winding up their affairs and settling all the claims by and against same." We
note that the plaintiff Insular Sawmill, Inc. ceased as a corporation on
December 30, 1960 but the case at bar was instituted on May 29, 1959,
during the time when the corporation was still very much alive. Accordingly,
it is our view that "any litigation filed by or against it instituted within the
period, but which could not be terminated, must necessarily prolong that
period until the final termination of said litigation as otherwise corporations
in liquidation would lose what should justly belong to them or would be
exempt from the payment of just obligations through a mere technicality,
something that courts should prevent" (Philippine Commercial Laws by
Martin, 1962 Ed., Vol. 2, p. 1716).


Chase v. Buencamino
Facts:
Chase was the owner of a Production Manufacturing Company in the US.
Sometime in 1954 he was notified that his factory was in the path of a
proposed highway. He then contemplated of relocating his company. His
distributor Carrol informed him of a Dr. Buencamino of Manila who was
interested in establishing a manufacturing plant in the Philippines. The
Negotiations led to the establishment of American Machinery and Parts
Manufacturing Inc. (AMPARTS).
The final agreement was that Chase was to own 1/3 interest in AMPARTS,
Buencamino and Cranker as the owners of the 2/3.
81 | P a g e

Buencamino filed an application with the Central Bank for the allocation
$999,000 which would be used to purchased Chases factory; Chase,
shipped his machineries and had them installed in the AMPARTS plant in
Pasig.
The corporation began its operation with the three as its high ranking
officers. However their harmonious relationship was shortlived. Chase in
1957 tendered his letter of resignation which was accepted.
Elton W. Chase, as minority stockholder, filed a derivative suit for the
removal of the officers and directors of American Machinery & Parts
Manufacturing, Inc. on the ground of breach of trust and for the
appointment of a receiver and the dissolution and liquidation of Amparts if
necessary.
The lower court ordered Buencamino to pay to AMPARTS.

Issue:
Whether or not the defendants, more particularly defendant Dr.
Buencamino, guilty of "fraud" and/or breach of a legal obligation as would
entitle plaintiff not only to a "money judgment" but also to the dissolution
of Amparts and/or the removal of defendants Buencaminos from the
management of the said corporation.

Held:
YES, but the court cannot grant dissolution because the action was a
derivative one, for the benefit of AMPARTS and not for the personal gain of
Chase.
The court found Buencamino and Cranker guilty of committing fraud against
the corporation. But the court did not remove him from the management of
the corporation. The trial court held that Buencamino should account for
the amount of the overpricing it committed to AMPARTS.
On the issue of dissolution or receivership.The trial court held that
Amparts cannot be benefitted by its dissolution and that as Buencamino
and his group own 2/3 of Amparts, it would not be proper to oust him as
manager of Amparts. The most equitable solution would be to require him
to pay his monetary liability to Amparts.
The removal of a stockholder (in this case a majority stockholder) from the
management of the corporation and/or the dissolution of a corporation in a
suit filed by a minority stockholder is a drastic measure. It should be
resorted to only when the necessity is clear which is not the situation in the
case at bar.


Chung Ka Bio v. Intermediate Appellate Court
Facts:
The Philippine Blooming Mills Company, Inc. was incorporated on January
19, 1952, for a term of 25 years which expired on January 19,1977. On May
14, 1977, the members of its board of directors executed a deed of
assignment of all of the accounts receivables, properties, obligations and
liabilities of the old PBM in favor of Chung Siong Pek in his capacity as
treasurer of the new PBM, then in the process of reincorporation. On June
14, 1977, the new PMB was issued a certificate of incorporation by the
Securities and Exchange Commission.
On May 5, 1981, Chung Ka Bio and the other petitioners herein, all
stockholders of the old PBM, filed with the SEC a petition for liquidation
(but not for dissolution) of both the old PBM and the new PBM. The
allegation was that the former had become legally non-existent for failure
to extend its corporate life and that the latter had likewise been ipso facto
dissolved for non-use of the charter and continuous failure to operate
within 2 years from incorporation.
Issue:
Whether or not the board of directors of an already dissolved corporation
has the inherent power, without the express consent of the stockholders, to
convey all its assets to a new corporation.

Held:
Winding up is the sole activity of a dissolved corporation that does not
intend to incorporate anew. If it does, however, it is not unlawful for the old
board of directors to negotiate and transfer the assets of the dissolved
corporation to the new corporation intended to be created as long as the
stockholders have given their consent.

Clemente v. Court of Appeals
Facts:
82 | P a g e

Sociedad Anomas principal business was cockfighting and the operation
and management of cockpit.
1911, during its existence, the "Sociedad" acquired by installments the
parcel of land above described from the Friar Lands Estate of Calamba,
Laguna. Plaintiffs evidence shows that Mariano Elepao and Pablo
Clemente, now both deceased, were original stockholders of the aforesaid
"sociedad.
Pablo Clemente's shares of stocks were upon his death, later distributed and
apportioned to his heirs, in accordance with a Project of Partition.
In accordance with the aforesaid project of .partition, the "sociedad" issued
stock certificates to the aforesaid heirs of Pablo Clemente.
On the basis of their respective stocks certificates, present plaintiffs jointly
claim ownership over the above described property, asserting that their
fathers being the only known stockholders of the "sociedad" known as the
"Sociedad Popular Calamba," they, to the exclusion of all others, are entitled
to be declared owners of Lot No. 148-New.

Issue:
Whether or not the plaintiffs can claim ownership over the disputed lands.

Held:
Except in showing that they are the successors-in-interest of Elepao and
Clemente, petitioners have been unable to come up with any evidence to
substantiate their claim of ownership of the corporate asset.
If, indeed, the sociedad has long become defunct, it should behoove
petitioners, or anyone else who may have any interest in the corporation, to
take appropriate measures before a proper forum for a peremptory
settlement of its affairs. The Corporation Code provides various modes for
dissolving, liquidating or winding up, and terminating the life of the
corporation. Among the causes for such dissolution are when the corporate
term has expired or when, upon a verified complaint and after notice and
hearing, the Securities and Exchange Commission orders the dissolution of a
corporation for its continuous inactivity for at least five (5) years. The
corporation continues to be a body corporate for three (3) years after its
dissolution for purposes of prosecuting and defending suits by and against it
and for enabling it to settle and close its affairs, culminating in the
disposition and distribution of its remaining assets. It may, during the three-
year term, appoint a trustee or a receiver who may act beyond that period.
The termination of the life of a juridical entity does not by itself cause the
extinction or diminution of the rights and liabilities of such entity nor those
of its owners and creditors. If the three-year extended life has expired
without a trustee or receiver having been expressly designated by the
corporation within that period, the board of directors (or trustees) itself,
may be permitted to so continue as "trustees" by legal implication to
complete the corporate liquidation. Still in the absence of a board of
directors or trustees, those having any pecuniary interest in the assets,
including not only the shareholders but likewise the creditors of the
corporation, acting for and in its behalf, might make proper representations
with the Securities and Exchange commission, which has primary and
sufficiently broad jurisdiction in matters of this nature, for working out a
final settlement of the corporate concerns.



FOREIGN CORPORATION

Home Insurance Co. v. Eastern Shipping Lines
Facts:
S. Kajita & Co. shipped from Japan coals on board the vessel owned by
Eastern Shipping Lines, to be delivered at Manila. The shipment was
covered by Bill of Lading with Phelps Dodge Copper Products Corporation of
the Philippines, as consignee. The shipment was insured with plaintiff
against all risks under its Insurance Policy. Some of the coals discharged
were in bad order. Thus, for the loss/damage suffered by the cargo, plaintiff
paid the consignee under its insurance policy; the latter then made
demands for payment against respondent Eastern Shipping for
reimbursement which refused to pay the same.
In December 1966, the Hansa Transport Kontor shipped from Bremen,
Germany, Service Parts of Farm Equipment on board the VESSEL owned by
the defendant, N. V. Nedlloyd Lijnen, and represented in the Philippines by
its local agent, the defendant Columbian Philippines, Inc. The shipment was
insured with plaintiff company under its Cargo Policy. The packages
discharged were found to be in bad order. For the short-delivery of 1
package, petitioner paid the CONSIGNEE under its Insurance Cargo Policy,
by virtue of which plaintiff became subrogated to the rights and actions of
83 | P a g e

the CONSIGNEE. Demands were made on respondent V. Nedlloyd and
Columbian for reimbursement but they failed and refused to pay the same.
During the abovementioned transaction, petitioner was a foreign
corporation doing business in the Philippines without necessary license
prescribed by the Corporation Law. However, upon institution of the
present action, petitioner was able to obtain the said necessary license to
do business in the Philippines. Respondents however, in its answer to the
complaint, denied petitioners capacity to sue.
Trial court then dismissed the complaints in the two cases on the ground
that petitioner failed to prove its capacity to sue and further ruled that the
insurance contract involved in the case is void for having been entered into
by petitioner without the necessary license to do business in the Philippines.

Issue:
Whether or not petitioners capacity to sue has been cured by its
subsequent registration; and whether or not the contract entered into by
petitioner not having license to do business in the Philippines is void.


Held:
The Corporation Law is silent on whether or not the contract executed by a
foreign corporation with no capacity to sue is null and void ab initio. Where
a contract which is entered into by a foreign corporation without complying
with the local requirements of doing business is rendered void either by the
express terms of a statute or by statutory construction, a subsequent
compliance with the statute by the corporation will not enable it to
maintain an action on the contract. But where the statute merely prohibits
the maintenance of a suit on such contract (without expressly declaring the
contract "void"), it was held that a failure to comply with the statute
rendered the contract voidable and not void, and compliance at any time
before suit was sufficient. Thus, there is no question that the contracts are
enforceable. The requirement of registration affects only the remedy.

Antam Consolidated Inc. v. Court of Appeals
Facts:
Strokely Van Camp. Inc. and one of its subdivision, Capital City Product
Company are corporations organized and existing under the laws of the
state of Indiana, U.S.A. and were not engaged in business in the Philippines
prior to the commencement of the present suit.
Capital City and Coconut Oil Manufacturing Phil. Inc. (Comphil), now Antam
Consolidated Inc., entered into three contracts under each of which Comphil
failed to deliver the coconut oil.
After repeated demands from Comphil to pay the said amount, the latter
still refuses to pay the same. Thus, Stroke and Capital City instituted the
present action.
Petitioners filed a motion to dismiss the complaint on the ground that the
respondent, being a foreign corporation not licensed to do business in the
Philippines, has no personality to maintain the instant suit. CFI and IAC,
upon appeal to the latter, denied petitioners motion to dismiss and ruled in
favor of respondent Stroke and Capital City. Hence, present petition.
Petitioners maintain that to maintain the suit filed with the trial court, the
respondent should have secured the requisite license to do business in the
Philippines because, in fact, it is doing business here. Petitioners anchor
their argument that the respondent is a foreign corporation doing business
in the Philippines on the fact that by the respondent's own allegations, it
has participated in three transactions, either as a seller or buyer, which are
by their nature, in the pursuit of the purpose and object for which it was
organized.

Issue:
Whether or not respondent is doing business in the Philippines by virtue of
the transactions it entered into with the petitioners, thus, is required to
obtain a license to commence the present suit.

Held:
Where the three transactions indicate no intent by the foreign corporation
to engage in continuity of transactions, as the contract was modified twice
principally because of the failure of the seller to deliver, the foreign
corporation is not doing business in the Philippines and can sue the seller
even if it has no license to do business in the Philippines. Foreign
corporation not doing business in the Philippines is not required to obtain
license to do business to have capacity to sue.

RIGHT OF FIRST REFUSAL

84 | P a g e

JG Summit Holdings Inc. v. Court of Appeals
Facts:
The National Investment and Development Corporation (NIDC), a
government corporation, entered into a Joint Venture Agreement (JVA) with
Kawasaki Heavy Industries, Ltd. of Kobe, Japan (KAWASAKI) for the
construction, operation and management of the Subic National Shipyard,
Inc. (SNS) which subsequently became the Philippine Shipyard and
Engineering Corporation (PHILSECO).
One of its salient features is the grant to the parties of the right of first
refusal should either of them decide to sell, assign or transfer its interest in
the joint venture.
NIDC transferred all its rights, title and interest in PHILSECO to the Philippine
National Bank (PNB). Such interests were subsequently transferred to the
National Government. Thereafter, a trust agreement was entered into
between the National Government and the APT wherein the latter was
named the trustee of the National Government's share in PHILSECO. As a
result of a quasi-reorganization of PHILSECO to settle its huge obligations to
PNB, the National Government's shareholdings in PHILSECO increased
reducing KAWASAKI's shareholdings.
The COP and the APT deemed it best to sell the National Government's
share in PHILSECO to private entities. After a series of negotiations between
the APT and KAWASAKI, they agreed that the latter's right of first refusal
under the JVA be "exchanged" for the right to top by five percent (5%) the
highest bid for the said shares.
Subsequently, interested bidders were given copies of the JVA between
NIDC and KAWASAKI. As petitioner was declared the highest bidder, the
COP approved the sale "subject to the right of Kawasaki Heavy Industries,
Inc./[PHILYARDS] Holdings, Inc. to top JGSMI's bid by 5% as specified in the
bidding rules."
Petitioner filed with this Court a Petition for Mandamus, said petition was
referred to the Court of Appeals. The Court of Appeals denied the same for
lack of merit. Hence this present petition.

Issue:
Whether the transfer is valid

Held:
YES. agreement of co-shareholders to mutually grant this right to each
other, by itself, does not constitute a violation of the
provisions of the Constitution limiting land ownership to Filipinos and
Filipino corporations. As PHILYARDS correctly puts it, if PHILSECO still owns
land, the right of first refusal can be validly assigned to a qualified Filipino
entity in order to maintain the 60%-40% ratio. This transfer, by itself, does
not amount to a violation of the Anti-Dummy Laws, absent proof of any
fraudulent intent. The transfer could be made either to a nominee or such
other party which the holder of the right of first refusal feels it can
comfortably do business with. Alternatively, PHILSECO may divest of its
landholdings, in which case KAWASAKI, in exercising its right of first refusal,
can exceed 40% of PHILSECOs equity. In fact, it can even be said that if the
foreign shareholdings of a landholding corporation exceeds 40%, it is not
the foreign stockholders ownership of the shares which is adversely
affected but the capacity of the corporation to own land that is, the
corporation becomes disqualified to own land. This finds support
under the basic corporate law principle that the corporation and its
stockholders are separate juridical entities. In this vein, the right of
first refusal over shares pertains to the shareholders whereas the
capacity to own land pertains to the corporation. Hence, the fact that
PHILSECO owns land cannot deprive stockholders of their right of first
refusal. No law disqualifies a person from purchasing shares in a
landholding corporation even if the latter will exceed the allowed
foreign equity, what the law disqualifies is the corporation from
owning land

INTRA CORPORATE CONTROVERSY
CASES UNDER THE OLD RULE WHERE THE SEC HAD JURISDICTION OVER
INTRA CORPORATE CONTROVERSY

DMRC Enterprise v. Este Del Sole Mountain Reserve Inc.
Facts:
DMRC Enterprises made an offer in writing to Este Del Sol Mountain Resort.,
for the lease to it of three (3) units of heavy equipment.
As further conditions of the agreement, Este Del Sol Mountain Resort, was
to advance the sum of P5,000.00 per unit to be deducted from the first
collection.
85 | P a g e

As a result of the agreement DMRC proceeded to perform what was
incumbent upon it. However, despite repeated demands Este Del Sol
Mountain Resort refused to pay. Thus, DMRC Enterprise filed an action for
collection. Este del Sol Mountain Resort interposed a motion to dismiss on
the sole ground that the respondent court has no jurisdiction over the
nature of the action or suit.
The trial court granted the motion to dismiss.
Este del Sol Mountain Resort states that to compel a corporation to issue its
shares of stock in the name of subscribers or stockholders involves
controversies arising out of intracorporate affairs of the corporation and its
stockholders or subscribers. It is further argued that since DMRC Enterprise
seeks to recover from Este del Sol Mountain Resort some shares allegedly
arising out of contractual relations whereby DMRC Enterprise undertook to
render services for the Este del Sol Mountain Resort and the Este del Sol
Mountain Resort agreed to issue shares of stock to the DMRC Enterprise,
the latter is thereby made a stockholder or shareholder of the Este del Sol
Mountain Resort. Thus, the SEC is the proper forum to take cognizance of
the present controversy.
On the other hand, DMRC maintains that the complaint is simply an action
for the collection of money and delivery of personal property representing
unpaid obligations within the competence of the regular courts.

Issue:
Whether the dispute is an intra corporate controversy

Held:
NO. An action against a corporation to collect on a contractual obligation
payable in cash and partly in shares of stock without averment of fraud or
misrepresentation falls within jurisdiction of ordinary courts, not Securities
and Exchange Commission.

Malayan Integrated Industries Corporation vs. Mendoza
Facts:
Private respondents instituted a Civil Case for "Accounting and For Payment
of the Fair Value of the Shares of Bonifacio Abuyen to his heirs and
Settlement of Estate" against petitioner.
The private respondents alleged among others that "their late father,
Bonifacio Abuyen, was an incorporator of petitioner corporation and has
never received any dividend, salary and benefits from the said corporation
from the time it was incorporated until his death."
The case was assigned to the Regional Trial Court, Makati, Metro Manila.
Petitioner filed an answer asserting that the Honorable Court has no
jurisdiction over the subject-matter and over the nature of the action which
is an intracorporate affair, for the reason that the subject-matter and nature
of the action involved fall within the original, exclusive and absolute
jurisdiction of the Securities and Exchange Commission.
The trial court took cognizance of the case and concluded that it is not an
intra corporate squable or conflict for neither it arose between and among
the stockholders, members or associates. Hence this present petition.

Issue:
Which has jurisdiction over the controversy, the regular courts or the
Securities and Exchange Commission?

Held:
Respondents asking for an accounting and distribution of shares and
dividends of their father in the corporation is in the nature of intra
corporate conflicts in view of the corporations denial of said demands and
Securities and Exchange Commission has exclusive jurisdiction.

CASES UNDER THE NEW RULE THAT THE RTC EXERCISES JURISDICTION
OVER INTRACORPORATE DISPUTES

Calleja v. Panday
Facts:
Panday et al. were members of the Board of Directors and officers of St.
John Hospital, but allegedly, Calleja et. al, who were also among the
incorporators and stockholders of the said corporation, forcibly and with
the aid of armed me usurped the powers which were supposedly belonged
to Panday et. al.

Panday et.al. filed a petition for quo warranto with Damages and Prayer for
Mandatory and Prohibitory Injunction, Damages and Issuance of Temporary
Restraining Order against Calleja et.al.
86 | P a g e



RTC-Br. 58 issued an Order transferring the case to the Regional Trial Court
in Naga City, since the verified petition showed Calleja et. al appear to be
residents of Naga City, then pursuant to Section 7, Rule 66 of the 1997 Rules
of Civil Procedure, the action for quo warranto should be brought in the
Regional Trial Court exercising jurisdiction over the territorial area where
the respondents or any of the respondents resides. However, the Executive
Judge of RTC, Naga City refused to receive the case folder of the subject
case for quo warranto, stating that improper venue is not a ground for
transferring a quo warranto case to another administrative jurisdiction.
RTC-Br. 58 issued the assailed order remanding the case to the RTC-Br. 23,
Naga City which has been designated as a Special Court to try and decide
intra-corporate controversies.

ISSUE:
Whether a branch of the Regional Trial Court which has no jurisdiction to try
and decide a case has authority to remand the same to another co-equal
court in order to cure the defects on venue and jurisdiction.

HELD:
Evidently, the RTC-Br. 58 in San Jose, Camarines Sur is bereft of jurisdiction
over respondents petition for quowarranto. Based on the allegations in the
petition, the case was clearly one involving an intra-corporate dispute. The
trial court should have been aware that under R.A. No. 8799 and the
aforementioned administrative issuances of this Court, RTC-Br. 58 was
never designated as a Special Commercial Court; hence, it was never vested
with jurisdiction over cases previously cognizable by the SEC.



Orendain v. BF Homes, Inc
Facts:
BF Homes availed financial assistance from various sources to enable it to
buy properties and convert them into residential subdivisions. This resulted
in its incurring liabilities amounting to PhP 1.5B. On the other hand, during
its business operations, it was able to acquire properties and assets worth
PhP 2.4B, which, if liquidated, were more than enough to pay all its
creditors.
Despite its solvent status, BF Homes filed a Petition for Rehabilitation and
for Declaration in a State of Suspension of Payments before the SEC. In the
said petition, BF Homes prayed thatin the meantime it was continuing its
business operationsit be afforded time to pay its aforesaid obligations,
freed from various proceedings either judicially or extra-judicially against its
assets and properties. The SEC granted the petition. With Atty. Orendain
appointed as Chairman of the Management Committee.
Subsequently, a Deed of Absolute Sale

was executed by and between BF
Homesrepresented Orendainas absolute and registered owner, and the
Local Superior of the Franciscan Sisters of the Immaculate Phils., Inc.
(LSFSIPI) over a parcel of land situated at Barangay Pasong Papaya, BF
International, Municipality of Las Pias, Metro Manila.
Meanwhile, SEC appointed a new Committee of Receivers composed of the
eleven (11) members of the Board of Directors of BF Homes with Albert C.
Aguirre as the Chairman of the Committee. Consequently, receiver Orendain
was relieved of his duties and responsibilities.
BF Homes filed a Complaint with the RTC against LSFSIPI and Orendain, and
for reconveyance of the property, alleging, that the LSFSIPI transacted with
Orendain in his individual capacity and therefore, neither FBO Management,
Inc. nor Orendain had title to the property transferred. Moreover, BF Homes
averred that the selling price was grossly inadequate or insufficient
amounting to fraud and conspiracy with the LSFSIPI.
Florencio B. Orendain filed a Motion to Dismiss stating that the RTC had no
jurisdiction over the reconveyance suit, since BF Homes suit was instituted
against him as its former receiver. He also avers that it is the SEC which has
the sole power to decide the issue as to whether petitioner acted within the
scope of the vested authority.
CA affirmed the RTC it held that the action for reconveyance filed by BF
Homes was within the exclusive jurisdiction of the RTC. In the rehabilitation
case, the LSFSIPI was not a party to the said case and did not have any intra-
corporate relation with petitioner at the time of the sale. The SEC could not
acquire jurisdiction over the Franciscan Sisters; while petitioner Orendain
was sued in his individual capacity and not in his official capacity as receiver.

Issue:
87 | P a g e

Which has jurisdiction over the action for reconveyancethe RTC or SEC?

Held:
Section 5.2 of RA 8799 transferred exclusive and original jurisdiction of the
SEC over actions involving intra-corporate controversies to the courts of
general jurisdiction or the appropriate RTC. In the transition, all intra-
corporate cases pending in the SEC, which were not ripe for adjudication as
of August 8, 2000, were turned over to the RTC. Congress thereby
recognized the expertise and competence of the RTC to take cognizance of
and resolve cases involving intra-corporate controversies. Thus, "whether or
not the issue is intra-corporate, it is now the [RTC] and no longer the SEC
that takes cognizance of [and resolves cases involving intra-corporate
controversies]."
Thus, it is unequivocal that the jurisdiction to try and decide cases originally
assigned to the SEC under Section 5 of PD 902-A has been transferred to the
RTC. Juxtaposing the jurisdiction of the RTC under RA 8799 and the powers
that were retained by the SEC, it is clear that the SEC retained its
administrative, regulatory, and oversight powers over all corporations,
partnerships, and associations who are grantees of primary franchises,
and/or a license or permit issued by the Government. However, the
Securities Regulations Code (SRC) is clear that when there is a controversy
arising out of intra-corporate relations, between and among stockholders,
members or associates, and between, any, or all of them and the
corporation, it is the RTC, not SEC, which has jurisdiction over the case.
Thus, when the complaint involves "an active antagonistic assertion of a
legal right on one side and a denial thereof on the other concerning a real,
and not a mere theoretical question or issue, a cause of action involving a
delict or wrongful act or omission committed by a party in violation of the
primary right of another, or an actual controversy involving rights which are
legally demandable or enforceable, the jurisdiction over this complaint is
lodged with the RTC but not the SEC.
The passage of RA 8799 has put to rest petitioner Orendains claim that it is
the SEC and not the RTC that has jurisdiction over case. At present, the
instant petition has nothing to stand on and perforce must fail.


Yujuico v. Quiambao
Facts:
Strategic Alliance Development Corporation (STRADEC) amended its AOI to
change its principal office from Pasig City to Bayambang, Pangasinan.
STRADEC held its annual stockholders meeting in its Pasig City office as
indicated in the notices sent to the stockholders. Yujuivo was one of the
elected members of the Board of Directors; he was also elected Chairman
and President, while Bonifacio Sumbilla was elected Treasurer.
After five (5) months, Quiambao et. al filed with the Regional Trial Court
(RTC), San Carlos City, Pangasinan a Complaint against STRADEC, praying
that the election be nullified on the ground of improper venue, pursuant to
Section 51 of the Corporation Code; all ensuing transactions conducted by
the elected directors be likewise nullified; and a special stockholders
meeting be held anew and to enjoin Yujuico et al from discharging their
functions as directors and officers of STRADEC.
The Trial court granted th

Issue:
Whether or not has the power to call a special stockholders meeting
involving an intra-corporate controversy.

Held:
Upon the enactment of R.A. No. 8799, otherwise known as The Securities
Regulation Code which took effect on August 8, 2000, the jurisdiction of
the SEC over intra-corporate controversies and other cases enumerated in
Section 5 of P.D. No. 902-A has been transferred to the courts of general
jurisdiction, or the appropriate RTC.
Clearly, the RTC has the power to hear and decide the intra-corporate
controversy of the parties herein. Concomitant to said power is the
authority to issue orders necessary or incidental to the carrying out of the
powers expressly granted to it. Thus, the RTC may, in appropriate cases,
order the holding of a special meeting of stockholders or members of a
corporation involving an intra-corporate dispute under its supervision.

Reyes V. RTC of Makati
Facts:
Oscar and Rodrigo C. Reyes are two of the four children of the spouses
Pedro and Anastacia Reyes. Pedro, Anastacia, Oscar, and Rodrigo each
owned shares of stock of Zenith Insurance Corporation (Zenith), a domestic
corporation established by their family. Pedro and Anastacia died. Although
88 | P a g e

Pedro's estate was judicially partitioned among his heirs, no similar
settlement and partition appear to have been made with Anastacia's estate,
which included her shareholdings in Zenith.
Zenith and Rodrigo filed a complaint with the Securities and Exchange
Commission (SEC) against Oscar, the complaint stated that it is "a derivative
suit initiated and filed by the complainant Rodrigo C. Reyes to obtain an
accounting of the funds and assets of ZENITH INSURANCE CORPORATION
which are now or formerly in the control, custody, and/or possession of
Oscar and to determine the shares of stock of deceased spouses Pedro and
Anastacia Reyes that were arbitrarily and fraudulently appropriated by
Oscar for himself and which were not collated and taken into account in the
partition, distribution, and/or settlement of the estate of the deceased
spouses, for which he should be ordered to account for all the income from
the time he took these shares of stock, and should now deliver to his
brothers and sisters their just and respective shares."

Oscar denied the charge that he illegally acquired the shares of Anastacia
Reyes. He asserted, as a defense, that he purchased the subject shares with
his own funds from the unissued stocks of Zenith. He claimed that the
complaint is a mere nuisance or harassment suit and should, according to
the Interim Rules of Procedure for Intra-Corporate Controversies, be
dismissed; and that it is not a bona fide derivative suit as it partakes of the
nature of a petition for the settlement of estate of the deceased Anastacia
that is outside the jurisdiction of a special commercial court.

Issues:
1. Whether or not the controversy is a derivative suit.
2. Whether or not the court has jurisdiction over the controversy.

Held:
Initially, the main consideration in determining whether a dispute
constitutes an intra-corporate controversy was limited to a consideration of
the intra-corporate relationship existing between or among the parties. The
types of relationships embraced under Section 5(b), were as follows:

a) between the corporation, partnership, or association and the
public;

b) between the corporation, partnership, or association and its
stockholders, partners, members, or officers;

c) between the corporation, partnership, or association and the
State as far as its franchise, permit or license to operate is
concerned; and
d) among the stockholders, partners, or associates themselves.

The existence of any of the above intra-corporate relations was sufficient to
confer jurisdiction to the SEC, regardless of the subject matter of the
dispute. This came to be known as the relationship test.
However, in the 1984 case of DMRC Enterprises v. Esta del Sol Mountain
Reserve, Inc.,

the Court introduced the nature of the controversy test. The
Court declared in this case that it is not the mere existence of an intra-
corporate relationship that gives rise to an intra-corporate controversy; to
rely on the relationship test alone will divest the regular courts of their
jurisdiction for the sole reason that the dispute involves a corporation, its
directors, officers, or stockholders. We saw that there is no legal sense in
disregarding or minimizing the value of the nature of the transactions which
gives rise to the dispute.
Under the nature of the controversy test, the incidents of that relationship
must also be considered for the purpose of ascertaining whether the
controversy itself is intra-corporate. The controversy must not only be
rooted in the existence of an intra-corporate relationship, but must as well
pertain to the enforcement of the parties' correlative rights and obligations
under the Corporation Code and the internal and intra-corporate regulatory
rules of the corporation. If the relationship and its incidents are merely
incidental to the controversy or if there will still be conflict even if the
relationship does not exist, then no intra-corporate controversy exists.
The Court then combined the two tests and declared that jurisdiction should
be determined by considering not only the status or relationship of the
parties, but also the nature of the question under controversy. Thus under
the relationship test, the transfer of title by means of succession, though
effective and valid between the parties involved (i.e. between the
decedents estate and her heirs) does not bind the corporation and third
parties. The transfer must be registered in the books of the corporation to
make the transferee-heir a stockholder entitled recognition as such both by
the corporation and by third parties. Therefore, each of the decedents heirs
holds only an undivided interest in the shares. This interest is still inchoate
89 | P a g e

and subject to the outcome of a settlement proceeding; the right of the
heirs to specific, distributive shares of inheritance will not will not be
determined until all the debts of the estate of the decedent are paid. Insofar
as the subject shares of stock are concernedthe heir of the deceased
stockholder cannot be considered a stockholder of the corporation.
Applying the nature of the controversy test, an accounting of funds and
assets of the corporation to determine the extent and value of the
decedents shareholdings will be undertaken by the probate court and not
by a special commercial court is consistent with the probate courts limited
jurisdiction. Hence, no intra-corporate dispute exists. In other words, the
case is not considered intra-corporate controversy even if the dispute is
among stockholders if the issue is determination and distribution of
successional rights to the shareholdings of a deceased shareholder.

Unlad Resources Devt., Corp. et al. v. Renato P. Dragon
Facts:
Dragon et al. and Unlad Resources, through its Chairman Helena Z. Benitez
entered into a MOA wherein it is provided that Dragon et al., as controlling
stockholders of the Rural Bank of Noveleta shall allow Unlad Resources to
invest 4.8M in the Rural Bank in the form of additional equity. On the other
hand, Unlad Resources bound itself to invest the said amount of 4.8 million
pesos in the Rural Bank; upon signing, it was, likewise, agreed that Unlad
Resources shall subscribe to a minimum of four hundred eighty thousand
pesos (P480,000.00) common or preferred non-voting shares of stock with a
total par value of four million eight hundred thousand pesos
(P4,800,000.00) and pay up immediately one million two hundred thousand
pesos (P1,200,000.00) for said subscription; that Dragon et al., upon the
signing of the said agreement shall transfer control and management over
the Rural Bank to Unlad Resources.

According to Dragon et al, immediately after the signing of the agreement,
they complied with their obligation and transferred control of the Rural
Bank to Unlad Resources and its nominees and the Bank was renamed the
Unlad Rural Bank of Noveleta, Inc. However, Dragon et al claim that despite
repeated demands, Unlad Resources has failed and refused to comply with
their obligation under the said Memorandum of Agreement when it did not
invest four million eight hundred thousand pesos (P4,800,000.00) in the
Rural Bank in the form of additional equity and, likewise, it failed to
immediately infuse one million two hundred thousand pesos
(P1,200,000.00) as paid in capital upon signing of the Memorandum of
Agreement.

Unlad Resources President and General Manager, entered into a Contract
of Lease over the Naic, Cavite mango plantation, as a consequence of this
venture, the bank incurred expenses amounting to P475,371.57, equivalent
to 25.76% of its capital and surplus.

Unlad Rural Bank wrote Dragon et al. regarding the Central Banks approval
to retire its Development Bank of the Philippines preferred shares in the
amount of P219,000.00 and giving notice for subscription to proportionate
shares. Dragon et al. objected on the grounds that there is already a sinking
fund for the retirement of the said DBP-held preferred shares provided for
annually and that it could deprive the Rural Bank of a cheap source of fund.
Dragon et al. filed before the Regional Trial Court (RTC) for rescission of the
agreement and the return of control and management of the Rural Bank
from Unlad Resources to Dragon et al. plus damages.
Trial Court declared that the Memorandum of Agreement is rescinded,
ordered that the management be returned to Dragon et al. and enjoined
from placing the retired DBP-held preferred shares available for subscription
and the same is hereby ordered to be placed under a sinking fund.
Unlad Resources questioned the jurisdiction of the Trial Court contending
that the issues that respondents raised before the trial court are intra-
corporate in nature and are, therefore, beyond the jurisdiction of the trial
court. They point out that respondents complaint charged them with
mismanagement and alleged dissipation of the assets of the Rural
Bank. Since the complaint challenges corporate actions and decisions of the
Board of Directors and prays for the recovery of the control and
management of the Rural Bank, these matters fall outside the jurisdiction of
the trial court.

Issue:
Whether or not RTC have jurisdiction over the subject matter of the case.

Held:
Nowhere in said decree (PD 902-A) do we find even so much as an
intimation that absolute jurisdiction and control is vested in the Securities
90 | P a g e

and Exchange Commission in all matters affecting Corporations. Be that as it
may, this point has been rendered moot and academic by Republic Act
8799, also known as the Securities Regulation Code. This law, which took
effect in 2000, has transferred jurisdiction over intra-corporate disputes to
the RTC.



REGULATORY JURISDICTION

Peneyra v. Intermediate Appellate Court
Facts:
The Board of Trustees of the Corregidor College Inc. awarded the
management and operation of its canteen at a monthly rental of P80.00 to
petitioners herein who are stockholders of the said College. Subsequently,
upon instructions of Dizon, Chairman of the Board of Trustees the rental
payments of petitioners were refused, and partial demolition of the canteen
was effected.
Petitioners filed an action against Dizon for damages Petitioners filed a
motion for leave to amend the complaint so as to include Corregidor
College, Inc. as additional defendant. The trial court denied petitioners'
motion, ruling that the proposed amendment would substantially alter
petitioners' cause of action.
On December 1983, Eulogio Dizon died. The trial court dismissed
petitioners' complaint on the ground that the action for damages did not
survive the death of Dizon.
Petitioners elevated the case to the then IAC which dismissed the petition
and held that the SEC has jurisdiction over the case, the same being an
intracorporate dispute, that the amendment to include Corregidor College,
Inc. cannot be allowed and that the action for damages against Eulogio
Dizon was extinguished by his death.

Issue:
Whether or not the jurisdiction over the case pertain to the Securities and
Exchange Commission
Held:
While it is true that petitioners herein are stockholders of Corregidor
College, Inc., the. Complaint did not stem directly from such relationship,
but rather from the award to petitioners of the management and operation
of its canteen at a monthly rental of P80.00. The management of a canteen,
even if awarded to a stockholder, is outside or merely incidental to the
central operations of an educational institution. Petitioners thus
convincingly argue that the controversy is not one where petitioners are
bringing the action as stockholders but rather as operators of the canteen
under an agreement with said Board. In short, the cause of action here is for
damages arising from a violation of a contract of management operation of
the College canteen by defendant Dizon. Certainly, the present controversy
cannot qualify as an intra-controversy, its root being a contractual breach
separate and distinct from the corporate relationship between petitioners
and Corregidor College, Inc., which, it must be noted, was not even named
as a defendant in the original complaint. It was therefore patent error for
the Court of Appeals to immediately rule that the present case belongs to
the SEC just because petitioners alleged that they are stockholders of
Corregidor College, Inc. Under Section 3 of Presidential Decree 902-A, the
jurisdiction of the SEC is limited to matters intrinsically connected with the
regulation of corporations, partnerships and associations and those dealing
with the internal affairs of such entities. P.D. 902-A does not confer in the
SEC absolute jurisdiction and control over all matters affecting corporations.
To uphold the appellate court's ruling would remove without legal
imprimatur from the regular courts all controversies over matters involving
or affecting corporations regardless of the nature of the transactions which
give rise to such disputes.

Securities and Exchange Commission v. Court of Appeals
Facts:
Cualoping Securities Corporation (CUALOPING for brevity) is a stockbroker,
Fidelity Stock Transfer, Inc. (FIDELITY for brevity), on the other hand, is the
stock transfer agent of Philex Mining Corporation (PHILEX for brevity).
On or about the first half of 1988, certificates of stock of PHILEX
representing one million four hundred [thousand] (1,400,000) shares were
stolen from the premises of FIDELITY. Later, the stolen stock certificates
ended in the hands of a certain Agustin Lopez, a messenger of New World
Security Inc., an entirely different stock brokerage firm. In the first half of
1989, Agustin Lopez brought the stolen stock certificates to CUALOPING for
91 | P a g e

trading and sale with the stock exchange. When the said stocks were
brought to CUALOPING, all of the said stock certificates bore the "apparent"
indorsement (signature) in blank of the owners (the stockholders to whom
the stocks were issued by PHILEX) thereof. At the side of these
indorsements (signatures), the words "Signature Verified" apparently of
FIDELITY were stamped on each and every certificate. Further, on the words
"Signature Verified" showed the usual initials of the officers of FIDELITY.
Upon receipt of the said certificates from Agustin Lopez, CUALOPING
stamped each and every certificate with the words "Indorsement
Guaranteed," and thereafter traded the same with the stock exchange.
After the stock exchange awarded and confirmed the sale of the stocks
represented by said certificates to different buyers, the same were
delivered to FIDELITY for the cancellation of the stocks certificates and for
issuance of new certificates in the name of the new buyers. Agustin Lopez
on the other hand was paid by CUALOPING for the value of the stocks. After
acquiring knowledge of the pilferage, FIDELITY conducted an investigation
with assistance of the NBI and found that two of its employees were
involved and signed the certificates.
FIDELITY rejected the issuance of new certificates in favor of the buyers for
reasons that the signatures of the owners of the certificates were allegedly
forged and thus the cancellation and new issuance thereof cannot be
effected. FIDELITY sought an opinion on the matter from SEC. The
Commission rendered its decision and concluded that both Cualoping
Securities Corporation and Fidelity Stock Transfers, Inc. equally negligent in
the performance of their duties and that both violated Section 29(3) of the
Revised Securities Act as a consequence of which a fine was imposed.

Issue:
Whether FIDELITY violated the Revised Securities Act to warrant an
imposition of a fine under Section 29(3)

Held:
There is no question that both FIDELITY and CUALOPING have been guilty of
negligence in the conduct of their affairs involving the questioned
certificates of stock. To constitute, however, a violation of the Revised
Securities Act that can warrant an imposition of a fine under Section 29(3),
in relation to Section 46 of the Act, fraud or deceit, not mere negligence, on
the part of the offender must be established. Fraud here is akin to bad faith
which implies a conscious and intentional design to do a wrongful act for a
dishonest purpose or moral obliquity; it is unlike that of the negative idea of
negligence in that fraud or bad faith contemplates a state of mind
affirmatively operating with furtive objectives. Given the factual
circumstances found by the appellate court, neither FIDELITY nor
CUALOPING, albeit indeed remiss in the observance of due diligence, can be
held liable under the above provisions of the Revised Securities Act.

Philippine Stock Exchange v. Securities and Exchange Commission
Facts:
The Puerto Azul Land, Inc. (PALI), sought to offer its shares to the public in
order to raise funds allegedly to develop its properties and pay its loans with
several banking institutions. PALI was issued a Permit to Sell its shares to the
public by the SEC. PALI sought to course the trading of its shares through
the Philippine Stock Exchange, Inc. (PSE), for which purpose it filed with the
said stock exchange an application to list its shares, with supporting
documents attached. However, the Board of Governors of the PSE received
a letter from the heirs of Ferdinand E. Marcos, claiming that the late
President Marcos was the legal and beneficial owner of certain properties
forming part of the Puerto Azul Beach Hotel and Resort Complex which PALI
claims to be among its assets.
On the other hand, PALI claimed that the properties forming part of the
Puerto Azul Beach Hotel and Resort Complex were not claimed by PALI as its
assets. On the contrary, the resort is actually owned by Fantasia Filipina
Resort, Inc. and the Puerto Azul Country Club, entities distinct from PALI.
The Board of Governors of the PSE reached its decision to reject PALI's
application, citing the existence of serious claims, issues and circumstances
surrounding PALI's ownership over its assets that adversely affect the
suitability of listing PALI's shares in the stock exchange.
PALI wrote a letter to the SEC, bringing to the SEC's attention the action
taken by the PSE. Thereafter, the SEC rendered its Order, reversing the PSE's
decision. PSE filed a motion for reconsideration of the said order which was
however denied. Dissatisfied with this ruling, the PSE filed with the Court of
Appeals a Petition for Review. The Court of Appeals promulgated its
Resolution dismissing the PSE's Petition for Review.

Issue:
92 | P a g e

Whether or not SEC may overrule the decision of PSE in rejecting the
application of PALI
Held:
The role of the SEC in our national economy cannot be minimized. The
legislature, through the Revised Securities Act, Presidential Decree No. 902-
A, and other pertinent laws, has entrusted to it the serious responsibility of
enforcing all laws affecting corporations and other forms of associations not
otherwise vested in some other government office. This is not to say,
however, that the PSEs management prerogatives are under the absolute
control of the SEC. The PSE is, after all, a corporation authorized by its
corporate franchise to engage in its proposed and duly approved business.
One of the PSEs main concerns, as such, is still the generation of profit for
its stockholders. Moreover, the PSE has all the rights pertaining to
corporations, including the right to sue and be sued, to hold property in its
own name, to enter (or not to enter) into contracts with third persons, and
to perform all other legal acts within its allocated express or implied
powers. A corporation is but an association of individuals, allowed to
transact under an assumed corporate name, and with a distinct legal
personality. In organizing itself as a collective body, it waives no
constitutional immunities and perquisites appropriate to such body. As to its
corporate and management decisions, therefore, the state will generally not
interfere with the same. Questions of policy and of management are left to
the honest decision of the officers and directors of a corporation, and the
courts are without authority to substitute their judgment for the judgment
of the board of directors. The board is the business manager of the
corporation, and so long as it acts in good faith, its orders are not
reviewable by the courts. Thus, notwithstanding the regulatory power of
the SEC over the PSE, and the resultant authority to reverse the PSEs
decision in matters of application for listing in the market, the SEC may
exercise such power only if the PSEs judgment is attended by bad faith. It is
also alleged that the properties belong to naval and forest reserves, and
therefore beyond private dominion.



SECURITIES

ONAPAL Philippines Commodities, Inc. vs. Court of Appeals
Facts:
ONAPAL Philippines Commodities, Inc. (petitioner) was licensed as
commission merchant/broker by the SEC, to engage in commodity futures
trading.
Susan Chua (plaintiff) was invited by defendant's Account Executive to
invest in the commodity futures trading. After a series of negotiations, she
was then made to sign the Trading Contract and other documents without
making her aware of the risks involved.
Thereafter, plaintiff was informed by Miss Diaz that she had to deposit an
additional amount "to pay the difference" in prices, otherwise she will lose
her original deposit. Fearing the loss of her original deposit, plaintiff was
constrained to deposit an additional amount.
Plaintiff further testified she understood the transaction of buying and
selling as speculating in prices, and her paying the difference between gains
and losses without actual delivery of the goods to be gambling. Thus, she
stopped trading in commodity futures when she realized she was engaged
in gambling.
She was able to get only P470,000.00 out of her total deposit of
P800,000.00. Consequently she filed a complaint for the recovery of said
amount.
The trial court ruled in favor of the plaintiff. The appellate court affirmed
said ruling. Hence this present petition.

Issue:
Whether the transactions covered by the Trading Contract between the
herein parties are illegal

Held:
YES. Commodity Futures trading contract is not illegal but when the
transaction between the parties to implement the contract is in the nature
of a gambling agreement as when they did not intend actual delivery but
merely speculated on this rise or fall of the price of the commodities, then
the same will be contrary to policy.


Union Bank of the Philippines vs. Securities and Exchange Commission
Facts:
93 | P a g e

Petitioner sought the opinion of the chairman of respondent Commission as
to the applicability and coverage of the Full Material Disclosure Rule on
banks, contending that the securities issued or guaranteed by banking
institutions were exempted from the registration requirement.
The chairman informed petitioner that while the requirements of
registration do not apply to securities of banks which are exempt under
Revised Securities Act, banks with a class of securities listed for trading on
the Philippine Stock Exchange, Inc. are covered by certain rules of the same
act governing the filing of various reports with respondent Commission.
Subsequently, respondent Commission wrote petitioner, enjoining the latter
to show cause why it should not be penalized for its failure to submit a
Proxy/Information Statement in connection with its annual meeting, in
violation of respondent Commission's Full Material Disclosure Rule.
However, petitioner did not comply.
Petitioner was then assessed a fine of P50,000.00 plus P500.00 for every day
that report was not filed. The respondent appealed the said decision but the
same was denied by Commission.
Petitioner then elevated its case to the Court of Appeals which affirmed
said decision. Hence this present petition

Issue:
Whether or not petitioner is required to comply with the respondent SEC's
full disclosure rules

Held:
YES. This SEC exempts from registration the securities issued by banking or
financial institutions mentioned in the law. Nowhere does it state or even
imply that petitioner, as a listed corporation, is exempt from complying with
the reports required by the assailed RSA Implementing Rules. The
exemption from the registration requirement enjoyed petition does not
necessarily connote that [it is] exempted from the other reportorial
requirements. Having confined the exemption enjoyed by the petitioner
merely to the initial requirement of registration of securities for public
offering, and not, [to] the subsequent filing of various periodic reports,
respondent Commission, as the regulatory agency, is able to exercise its
power of supervision and control over corporations and over the securities
market as a whole. Otherwise, the objectives of the 'Full Material
Disclosure' policy would be defeated since petitioner corporation and its
dealings would be totally beyond the reach of respondent Commission and
the investing public


Fabia vs. Court of Appeals
Facts:
Maritime Training Center of the Philippines, Inc. is a domestic corporation
engaged in providing maritime courses and seminars to prospective
overseas contract workers and seamen. Petitioner Hernani N. Fabia was the
President of private respondent MTCP and was likewise a Director and
stockholder thereof. MTCP through its new president filed an affidavit-
complaint before the Office of the City Prosecutor of Manila for estafa
against Fabia alleging that on various occasions, Fabia drew cash advances
from MTCP covered by cash vouchers, which he failed to liquidate despite
repeated demands. However, the complaint was dismissed for lack of
jurisdiction for the reason that the controversy pertained to the relationship
between a corporation and a former officer thereof, hence, it was SEC
which had original and exclusive jurisdiction over the case. MTCP filed a
petition for review before the Department of Justice which was also denied.
MTCP filed a petition for certiorari before the Court of Appeals which
granted the petition. Thereafter, the Office of the City Prosecutor as
directed caused the filing of an Information for estafa against petitioner
before the Manila RTC.
Petitioner now questions the jurisdiction of the trial court arguing that the
instant case involves an intra-corporate controversy primarily cognizable by
the SEC and, as such, the public prosecutor had no authority to initially rule
in the preliminary investigation of the complaint for estafa.

Issue:
Whether the trial court has jurisdiction of the instant case.

Held:
YES. While Section 5 of PD 902-A was amended by Sec. 5.2 of RA 8799, there
is no repeal of section 6 thereof declaring that prosecution under the
Decree, or any Act, law, rules and regulations enforced and administered by
the SEC shall be without prejudice to any liability for violation of any
provision of the Revised Penal Code. Therefore, the fraudulent devices,
schemes or representations which, originally the Prosecution and
94 | P a g e

Enforcement Department of the SEC would exclusively investigate and
prosecute, are those in violation of any law or rules and regulation
administered and enforced by the SEC and shall be without prejudice to any
liability for violation of the RPC. Hence, if the fraudulent act is punished
under the RPC, like estafa under Art.315, the responsible person may be
criminally prosecuted before the regular courts in addition to proceedings
before the branches of the RTC designated by the Supreme Court to try and
decide intra-corporate controversies.

Power Homes Unlimited Corporation vs. Securities and Exchange
Commission
Facts:
Petitioner is a domestic corporation duly registered with public respondent
SEC whose primary purpose is to engage in the transaction of promoting,
acquiring, managing, leasing, obtaining options on, development, and
improvement of real estate properties for subdivision and allied purposes,
and in the purchase, sale and/or exchange of said subdivision and
properties through network marketing. Respondent Noel Manero requested
public respondent SEC to investigate petitioners business. He claimed that
he attended a seminar conducted by petitioner where the latter claimed to
sell properties that were inexistent and without any brokers license. One
Romulo E. Munsayac, Jr. also inquired from public respondent SEC whether
petitioners business involves "legitimate network marketing." After
conducting series of investigation and examination public respondent SEC
issued a Cease and Desist Order. Petitioner moved for the lifting of the CDO,
which public respondent SEC denied for lack of merit. Consequently,
petitioner filed in the Court of Appeals a Motion for the Issuance of a Writ
of Preliminary Injunction which the appellate court granted.

Issue:
Whether petitioners business constitutes an investment contract which
should be registered with public respondent SEC before its sale or offer for
sale or distribution to the public.

Held:
YES. An investment contract is defined in the Amended Implementing Rules
and Regulations of R.A. No. 8799 as a "contract, transaction or scheme
whereby a person invests his money in a common enterprise and is led to
expect profits primarily from the efforts of others." The business scheme of
petitioner in the case at bar is essentially similar. An investor enrolls in
petitioners program by paying US$234. This entitles him to recruit two (2)
investors who pay US$234 each and out of which amount he receives
US$92. A minimum recruitment of four (4) investors by these two (2)
recruits, who then recruit at least two (2) each, entitles the principal
investor to US$184 and the pyramid goes on. the business operation or the
scheme of petitioner constitutes an investment contract that is a security
under R.A. No. 8799. Thus, it must be registered with public respondent SEC
before its sale or offer for sale or distribution to the public. As petitioner
failed to register the same, its offering to the public was rightfully enjoined
by public respondent SEC.


Times Realty Corporation vs. Cesar Lao and Cynthia V. Cortez
Facts:
Herein petitioner sold to Ceasar M. Lao and Cynthia V. Cortez (respondents),
one timeshare of Laguna de Boracay payable in eight months and fully paid
by the respondents.
Subsequently, the SEC issued a resolution to the effect that petitioner was
without authority to sell securities. It further stated in the resolution/order
that the Registration Statement of petitioner became effective only on
February 11, 1998.
Petitioner sought a reconsideration of the aforesaid order but the SEC
denied the same.
Respondents wrote petitioner demanding their right and option to cancel
their Contract, as it appears that Laguna de Boracay is selling said shares
without license or authority from the SEC. Despite repeated demands,
petitioner failed and refused to refund or pay respondents.

Respondents directly filed with SEC En Banc a Complaint against petitioner
and the Members of its Board of Directors. SEC En Banc rendered a Decision
in favor of respondents, ordering petitioner to pay respondents.
Petitioner filed a Petition for Review with the CA however, CA dismissed the
same. Hence this present petition.

Issue:
Whether mere registration as a corporation already authorizes it to deal
with unregistered timeshares
95 | P a g e


Held:
NO. The provisions of B.P. Blg. 178 do not support the contention of
petitioner that its mere registration as a corporation already authorizes it to
deal with unregistered timeshares. Corporate registration is just one of
several requirements before it may deal with timeshares. Prior to fulfillment
of all the other requirements of Section 8, petitioner is absolutely
proscribed under Section 4 from dealing with unregistered timeshares
unless such securities shall have been registered and permitted to be sold as
hereinafter provided

Das könnte Ihnen auch gefallen